Chemical Pathology Flashcards

1
Q
  1. Arterial blood gas sample
    A 67-year-old woman presents to accident and emergency after having a fall. She is diagnosed with a fractured neck of femur which is fixed with a hemi- arthroplasty. She also suffers from metastatic breast cancer. Four days postoperatively, she develops shortness of breath with an increased respiratory rate of 24 breaths per minute. The doctor on call takes an arterial blood gas sample which shows the following results:
pH 7.48
PaO2 15.4kPa on 2L of oxygen 
pCO2 2.6kPa
Base excess +1
Saturations 99%

What does the blood gas show?

A Metabolic alkalosis with respiratory compensation
B Metabolic alkalosis
C Respiratory alkalosis with metabolic compensation
D Respiratory alkalosis
E None of the above

A

D
This lady has most likely suffered a pulmonary embolism manifesting
as an acute onset of shortness of breath. Acid–base questions are best approached in three steps: first, decide if the pH shows an alkalosis or an acidosis. Next look at the PaCO2 and decide if it is high or low. Carbon dioxide dissolves in water to form carbonic acid, a weak acid. Therefore, if the concentration of carbon dioxide is high, it will lower the pH. You must then decide if the PaCO2 is compounding or helping the patient’s pH – in other words, is it worsening an acidotic patient or compensating for an alkalotic patient? Finally, look at the base excess. A greater positive base excess implies a higher concentration of bicarbonate, which is a base. Unlike carbon dioxide, therefore, high levels of bicarbonate will raise the pH. In this scenario, the pH is 7.48 meaning the patient is alkalotic with a low PaCO2, implying a respiratory cause. There is no compensation as the base excess of +1 is within normal limits. Unlike respiratory compensation, metabolic compensation takes several days. Below is a table of common causes of the different acid–base abnormalities with the likely carbon dioxide and base excess values.

Another way to tackle acid–base problems is to look at the Flenley Acid–Base normogram which depicts the likely metabolic abnormality given the pH and arterial PCO2:

Note the non-linear relationship of metabolic acidosis and alkalo- sis, in contrast with the linear relationship of the respiratory based acid–base abnormality. This is because ventilation is limited at both ends of the spectrum – too low and you risk hypoxia, too high and you cannot inhale a sufficient tidal volume for meaningful gas exchange!

How well did you know this?
1
Not at all
2
3
4
5
Perfectly
2
Q
  1. Exacerbating factors for gout
    A patient presents with an acutely painful, inflamed elbow. He has decreased range of movement passively and actively and the joint is tender, erythematous and warm. PMH: HTN, chronic lower back pain for which he takes aspirin, lymphoma for which he has just completed a course of chemotherapy and psoriasis which is well controlled. He is also a heavy drinker. A joint aspirate shows weakly negative birefringent crystals confirming acute gout.

Which factor in this patient is the least likely to contribute to this attack?

A Bendroflumethiazide
B Chemotherapy
C Alcohol
D Psoriasis
E Aspirin
A

D

Although all of these factors can contribute to hyperuricaeamia, well controlled psoriasis (D) in this patient is unlikely to contribute to this attack of gout. Gout may be acute or chronic and is caused by hyperuricaemia. Hyperuricaemia is caused either by increased urate production or decreased urate excretion.

Uric acid is a product of purine metabolism and is produced in three main ways – metabolism of endogenous purines, exogenous dietary nucleic acid and de novo production. De novo production involves metabolizing purines to eventually produce hypoxanthine and xanthine. The rate limiting enzyme in this pathway is called phosphoribosyl pyrophosphate aminotransferase (PAT) which is under negative feedback by guanine and adenlyl monophosphate. The metabolism of exogenous and endogenous purines, however, is the predominant pathway for
uric acid production. The serum concentration of urate is dependent
on sex, temperature and pH. A patient with acute gout does not necessarily have an increased urate concentration, therefore making serum urate levels an inaccurate method of diagnosis. The diagnosis of acute gout, which most commonly affects the first metatarsophalangeal joint (‘podagra’) is best made by observing weakly negatively birefringent crystals in an aspirate of the affected joint. This test is performed with polarized light – urate crystals are rhomboid and illuminate weakly when polarized light is shone perpendicular to the orientation of the crystal (hence negative birefringence). This is in contrast with pseu- dogout which has positively birefringent, spindly crystals – these illuminate best when the polarized light is aligned with the crystals. X-ray of the affected joint shows soft tissue inflammation early on, but as the disease progresses, well defined ‘punched out’ lesions in the juxta-articular bone appear with a late loss of joint space. There is no sclerotic reaction. Treatment is with a non-steroidal antiiflammatory (e.g. diclofenac) in the acute phase or colchicine.

Aspirin (E) is avoided because it directly competes for urate acid excretion in the nephron therefore worsening hyperuricaemia. After the acute attack settles, long term xanthine oxidase inhibitors (the enzyme responsible for the final production of urate) can be inhibited by allopurinol. Alternatively, but less commonly, uricosuric drugs such as probenecid may be used (e.g. prevention of cidofovir nephropathy). Finally rasburicase, recombinant urate oxidase, is a newer pharmacological tx in the setting of chemo to prevent hyperuricaeamia.
Thiazide diuretics such as bendroflumethiazide (A) act by inhibiting NaCl transport in the distal convoluted tubule. They are contraindicated in gout as they increase uric acid concentration and are a well known precipitant of gout. Other diuretics do not have this property and therefore this patient should have his antihypertensive medication reviewed. Other side effects of thiazides include hyperglyacaemia, hypercalcaemia and increased serum lipid concentrations.

Alcohol (C) increases urate levels in two ways – first it increases adenosine triphosphate turnover thus activating the salvage pathway producing more urate. It also decreases urate excretion in the kidney as it increases organic acids which compete for urate excretion in the nephron (much like aspirin).

Chemotherapy (B) involves the destruction of malignant cells, which release all of their intracellular contents into the blood stream including purines. Widespread malignancy treated with chemotherapy can dramatically increase urate concentration. Therefore some patients undergoing chemotherapy are given prophylactic allopurinal to prevent this side effect as well as being encouraged to drink plenty of fluid to essentially dilute the urate produced.

Psoriasis (D) is a dermatological condition characterized by discrete patches of epithelial hyperproliferation. There are different types including flexural, extensor, guttate, erythrodermic and pustulopalmar. Some special clinical signs associated with this condition often asked about include Koebner’s phenomenon (appearance of psoriatic plaques at sites of injury) and Auspitz’s sign (dots of bleeding when a plaque is scratched off representing reticular dermis clubbing with capillary dilatation). Severe psoriasis results in T-cell mediated hyperproliferation and eventual breakdown of cells releasing their intracellular contents resulting in hyperuricaemia in much the same mechanism as chemotherapy. The treatment for psoriasis includes phototherapy with ultraviolet light, topical agents including tar and oral tablets including antiproliferatives.

How well did you know this?
1
Not at all
2
3
4
5
Perfectly
3
Q
11. Anion gap
A patient has the following blood results; calculate the anion gap:
Na 143mmol/L
K 4mmol/L
Cl 107mmol/L
HCO3 25mmol/L
PO4 1mmol/L
Glucose 8mmol/L
Urea 7mmol/L
A 14 mmol/L
B 15 mmol/L
C 16 mmol/L
D 17 mmol/L
E Not enough information
A

A

The anion gap is calculated using the following equation:
Anion gap = [Na+] + [K+] − [HCO3] − [Cl−]

It is a method of assessing the contribution of unmeasured anions in metabolic acidosis. The normal range varies between laboratories but the upper limit is usually between 10 and 18mmol/L. It is helpful to estimate the unmeasured anions such as phosphate, ketones and lactate which are difficult to measure normally.
Metabolic acidosis in the setting of a raised anion gap implies there is an increase production or reduced excretion of fixed or organic acids. The acid produced is buffered by bicarbonate thus increasing the anion gap. Causes include raised lactate (e.g. shock, infection or tissue ischaemia), urate (renal failure), ketones (DM) or drugs (methanol, aspirin). Furthermore there are two types of lactic acidosis – type A and type B. Type A is the most commonly associated with shock. Hypoperfusion of the tissues reduces the capacity of cells to continue aerobic respiration which leads to the formation of lactate via anaerobic respiration. Physiologically lactate concentration is around 1mM but can rise up to 10mM in extreme situations. It can also be falsely raised when replacing fluids which contain lactate (e.g. Hartmann’s solution – a common surgical fluid used to treat hypovolaemia). This is particu- larly important when dealing with suspected bowel ischaemia where fluid resuscitation is a vital initial management step. Lactate is often used to distinguish the presence of ischaemia which could be falsely elevated if using this fluid!. Type B lactic acidosis occurs in the absence of significant oxygen delivery problems and usually occurs secondary to drugs. Common culprits include metformin in a patient with renal fail- ure, paracetamol overdose, ethanol or methanol poisoning or acute liver failure. A useful and often quoted mnemonic to remember the causes of metabolic acidosis with a raised anion gap is MUDPILES: Methanol, Uraemia, Diabetic ketoacidosis, Propylene glycol, Isoniazid, Lactic acidosis, Ethylene glycol, Salicylates.

Metabolic acidosis with a normal anion gap implies the loss of bicarbonate or ingestion of hydrogen ions. The loss of bicarbonate is compensated for by chloride thus normalizing the anion gap. This is why this type of acidosis is sometimes called hyperchloraemic acidosis. Alternatively excessive chloride load (e.g. ammonium chloride ingestion) can cause acidosis where bicarbonate concentration reduces to compensate. The causes of this type of acidosis are generally due to problems either in the kidneys, GI tract or secondary to drugs. In the kidneys, failure of acid secretion is the main problem. This may be due to an intrinsic problem in the tubules (called renal tubular acidosis (RTA)) or secondary to drugs manipulating the acid transport systems. There are four types of RTA: type I is caused by the failure of acid secretion in the distal convoluted tubule. There is an inability to acidify urine despite systemic acidosis. Type II is caused by a bicarbonate leak in the proximal tubule which may be an isolated defect or associated with a generalized tubulopathy (Fanconi’s syndrome). In RTA type II there is an ability to acidify the urine during systemic acidosis because the kidney retains some bicarbonate transport function. There is often hypokalaemia due to the increased osmotic diuretic effect in the tubule caused by excessive bicarbonate, therefore increasing flow rate to the distal tubule. Type III RTA is a rare combination of type I and type II RTA. Type IV RTA is always due to an intrinsic problem in the tubules. There is lack of effective function of aldosterone which may be due to the lack of renin release (e.g. renal failure with parenchymal loss in the juxtaglomerular apparatus), hypoadrenalism (e.g. autoimmune disease or tuberculosis), renal resistance to aldosterone or drugs (e.g. ACE inhibitor, non-steroidal anti-inflammatory drugs, potassium sparing diuretics).

GI loss of bicarbonate is the other main cause of metabolic acidosis with a normal anion gap. Diarrhoea caused by any pathology can lead to this problem. It is particularly associated in the setting of VIPoma (vasoactive intestinal peptide–oma). Also known as Verner Morrison syndrome, this rare disease is due to a non-beta islet cell tumour, usually in the pancreas. It causes profound diarrhoea, hypoka- laemia, achlorhydia and flushing. Note vomiting causes hypochloraemic alkalosis due to the loss of hydrogen chloride in the stomach. Other gastrointestinal causes include pancreatic or biliary fistulae, ileostomy or ureterosigmoidostomy.

One method to distinguish the different types of normal gap metabolic acidosis is the use of the urinary anion gap (UAG). The formula for this is:
Urinary anion gap = [Na+] + [K+] – [Cl-]

The UAG is a rough estimate of the bicarbonate concentration in the urine – the more negative the number, the higher the ammonium concentration and vice versa. This therefore helps distinguish the
cause of the normal gap metabolic acidosis. If the bowel is responsible through bicarbonate loss, it would be sensible to assume the kidneys will try to compensate by increasing the ammonium excretion which
is exchanged for hydrogen ions. The opposite is true for a loss of acid through the kidneys. A useful aide memoire is the word ‘neGUTive’. The negative urinary anion gap implies the gut is the culprit of the acidosis.

How well did you know this?
1
Not at all
2
3
4
5
Perfectly
4
Q
  1. Estimated plasma osmolarity
    A patient has the following blood results:
Na 143mmol/L
K 4mmol/L
Cl 107mmol/L
HCO3 25mmol/L
PO4 1mmol/L
Glucose 8mmol/L
Urea 7mmol/L
What is the estimated plasma osmolarity?
A 309
B 279
C 426
D 294
E Not enough information
A

A
Estimated plasma osmolarity = {[Na+] + [K+]}x2 + glucose+ urea

The estimation of osmolarity is an approximation of the laboratory plasma osmolality which is always higher. The difference between osmolarity and osmolality is the quantity of solvent one is referring to – the former describes the osmoles of solute in 1kg, whereas the latter describes the same solute in 1L of solvent. Sodium and potassium are the main plasma cations, they are doubled to take into account the equal concentration of total anions present to maintain electrical neu- trality. Glucose and urea are the other main osmolites even though urea has very little osmotic effect in the plasma. It is a very small molecule that can pass easily through cell membranes without affecting osmotic pressure.

Estimating osmolarity is useful when calculating the osmolar gap. This is the difference between the estimated osmolarity and the laboratory osmolality. The difference is usually <10mmol/L. If the gap is greater than 10mmol/L one must consider the presence of additional solutes to account for this gap. These solutes include: ethanol, methanol, ethylene glycol and acetone. This measurement is often used as a screening tool to detect toxins in the blood. A patient presenting unwell with a raised anion gap metabolic acidosis with a raised osmolar gap must be investigated for methanol, ethanol or ethylene glycol poisoning. A raised osmolar gap is also seen in lactic acidosis, diabetic ketoacidosis and in chronic renal failure.

Finally there is sometimes an apparent gap in patients with hyperlipidaemia or hyperproteinaemia. Here the measured electrolytes are underestimated as the increased fat or protein displaces a certain volume of the tested plasma but the concentration reading given is for the total solvent present. This gives a falsely low osmolality measurement and therefore plasma triglycerides, cholesterol and protein should be taken into account when performing this calculation. Nowadays, some laboratories take this well known effect into account, making the calculation much more accurate.

How well did you know this?
1
Not at all
2
3
4
5
Perfectly
5
Q
  1. Biochemical abnormalities in chronic renal failure

A 67 year old man with chronic renal failure presents with fatigue. He has been on haemodialysis 3/week for a decade.
PMH: DM, HTN and gout. He has been increasingly tired the last few weeks although he cannot explain why. He has been attending his dialysis appointments and is compliant with his medications. The GP takes some bloods to investigate. Which of the following is NOT a common association with CRF?

A Acidosis
B Anaemia
C Hyperkalaemia
D Hypocalcaemia
E Hypophosphataemia
A

E

Patients with chronic renal failure normally suffer from hyperphosphataemia, not hypophosphataemia (E). This is due to renal impairment of calcium metabolism which is under the control of parathyroid hormone (PTH) and vitamin D. In the evolving stages of chronic renal failure, a secondary hyperparathyroidism exists to compensate for the inability of the kidney to retain calcium and excrete phosphate. Therefore hypocalcaemia (D) is associated with chronic renal failure. This stimulates a physiological secretion of PTH by the parathyroid glands in an attempt to retain calcium. PTH is also responsible for excreting phosphate in the kidney, which is impaired due to the failure.

Hyperphosphataemia also increases PTH levels as part of a negative feedback loop designed to maintain its homeostasis. Patients with chronic renal failure usually take phosphate binders (e.g. Sevelamer) which act to reduce phosphate absorption. This reduces PTH production which also reduces bone resorption thus improving renal osteodystrophy, a complex metabolic bone pathology associated with chronic renal failure. It is also important to reduce phosphate concentration to reduce ectopic calcification – if this precipitates in the tubules, this may reduce what little function there is left.

Hyperkalaemia (B) is associated with chronic renal failure and is important as it can be potentially fatal. Hyperkalaemia changes cardiac membrane excitability making it more prone to arrhythmias; other indications include. Resistant severe hyperkalaemia (>7mmol/L) is an indication for emergency renal dialysis refractory pulmonary oedema, severe metabolic acidosis (pH <7.2 or base excess <10), uraemic encephalopathy or uraemic pericarditis. Tx: Hyperkalaemia is with calcium gluconate to stabilize the heart muscle, insulin and dextrose to reduce potassium levels as well as salbutamol or calcium resonium.

Acidosis is also a consequence of chronic renal failure. This is due directly to the failure of acid–base handling within the renal tubules and indirectly due to hyperkalaemia causing a shift of hydrogen ions into the plasma. Acidosis impairs metabolic functions by reducing the efficacy of enzymes as well as worsening renal osteodystrophy by essentially dissolving bone. There is usually respiratory compensation with hypocapnia.

Anaemia (B) is common with chronic renal failure and is normocytic normochromic in nature. It is due to the lack of erythropoietin production in the renal parenchyma. Tx: Recombinant human erythropoietin - acts as a replacement to stimulate erythropoiesis in bone marrow. Impt to exclude IDA which would produce a microcytic hypochromic picture as this is easily treatable with iron.

How well did you know this?
1
Not at all
2
3
4
5
Perfectly
6
Q
  1. Thyroid function tests
    A 45yo woman presents feeling tired all of the time. She has been investigated for anaemia which reveals macrocytosis. She denies drinking excessively. She has recently moved house and the GP notices she has a croaky voice, peaches and cream complexion and a slowed reaction to his questions. He examines her and elicits slow relaxing ankle reflexes. He suspects hypothyroidism and orders some thyroid function tests.

Which of the following results are consistent with primary hypothyroidism?

A Low TSH, raised free T4 and T3
B Low or normal TSH with low free T4 and T3
C Raised TSH with normal free T4 and T3
D Normal or raised TSH with raised T4 and T3
E None of the above

A

E
Thyroid function tests are relatively easy to interpret with a basic understanding of the hypothalamic–pituitary–thyroid axis of thyroid hormone control. The pituitary produces TSH (thyroid stimulating hormone) which is released from the anterior pituitary. It is under the control of the hypothalamus which releases thyroid releasing hormone (TRH) which signals to anterior pituitary cells to release TSH. TSH travels in the bloodstream and acts on thyrocytes in the thyroid gland to stimulate production of T4 and T3 hormone. Specifically TSH con- trols the rate of iodide uptake required for thyroid hormone production, thyroid peroxidase activity, iodotyrosine reuptake into the thyrocyte from colloid and iodotyrosine cleavage to form mature hormone. T4 is the main circulatory hormone produced in about a 10:1 ratio compared with T3. However, free T3 has greater efficacy; in fact circulating T4 is converted into T3 within cells which then binds to its hormone receptor. TSH release is under negative feedback control of T4. In primary hypothyroidism, the thyroid does not have the ability to produce suf- ficient T4 or T3 to inhibit further TSH release. Therefore the biochemical abnormality found in primary hypothyroidism is a raised TSH with low T4 and T3, which is not one of the answer options (E).
A low TSH with raised free T4 and T3 (A) is seen in primary hyper- thyroidism, the most common cause of which is Graves’ disease. This
is an autoimmune condition where stimulating antibodies bind to the TSH receptor to stimulate thyroid hormone production. The exces-
sive T4 concentration negatively feedbacks onto the hypothalamus and pituitary to reduce TSH release. The other causes of this biochemical abnormality include multinodular goitre with functional tissue, toxic nodule (also known as Plummer’s disease), transient thyroiditis and
De Quervain’s thyroiditis. Graves’ disease is unique in that it features extrathyroid features including pretibial myxoedema, exophthalmos and thyroid acropachy. Radioisotope scanning, a method using radioactive iodine to measure uptake in the thyroid gland, shows increased uptake throughout the gland.
A low or normal TSH with low free T4 and T3 (B) is frequently seen
in patients with other non-thyroid illness. This is also known as sick euthyroid syndrome where the patient is unwell with another illness causing thyroid abnormalities. The cause is unclear but the role of inflammatory cytokines and reduced peripheral deiodination of T4 has been implicated. Another important differential for this combination of biochemical abnormalities is secondary hypothyroidism i.e. pitui- tary dysfunction causing low TSH and low thyroid hormones. This differential is serious as the associated hypoadrenalism could be fatal. A pituitary tumour must be excluded by imaging (MRI brain) and endocrinological stimulation tests (i.e. short synacthen test) to exclude Addison’s disease. Another explanation for these results not applicable in this patient is recently treated hyperthyroidism. There is sometimes a residual suppression of TSH following hyperthyroid treatment for up to 1 year, and if they are clinically hypothyroid replacement therapy should be prescribed.
A raised TSH with normal T4 and T3 (C) normally means the patient is suffering from subclinical hypothyroidism. This is an important finding as patients may have subtle symptoms and improve with treatment as well as possibly reducing deaths from cardiac events. People with TSH levels >10 /L, positive thyroid antibodies, previously treated Graves’ disease or other organ specific autoimmunity (e.g. diabetes mellitus type I, myas- thenia gravis) should be treated as they are at high risk of progression to clinical hypothyroidism. Other less common causes of this biochemical configuration include amiodarone therapy, recovery from sick euthyroid disease and thyroxine malabsorption in patients taking thyroxine therapy due to small bowel disease, cholestyramine or iron therapy.
A normal or raised TSH with raised T3 and T4 (D) is a rare disorder and usually means there is an artefact with the test. The results imply cen- tral hyperthyroidism with the hypothalamus inappropriately excreting excessive TSH stimulating the thyroid gland to overproduce T4 and T3. Rarely it can be caused by amiodarone therapy, thyroid receptor muta- tions, intermittent thyroxine overdose, or familial dysalbuminaemic hyperthyroxinaemia. This last condition is a rare abnormality of albu- min which results in increased binding affinity of albumin for T4. This interferes with the assay and shows a normal TSH and T3 with appar- ently increased T4.

How well did you know this?
1
Not at all
2
3
4
5
Perfectly
7
Q
  1. Inherited metabolic disorders
    42 year old woman presents to maternity in labour. It is her first child and she delivers a baby boy at 42 weeks gestation. During the neonatal period, the child develops feeding difficulty with hypotonia and jaundice. O/E: conjugated hyperbilirubinaemia. The mother thinks this has started shortly after she has started feeding the child with milk. After a few months, the child develops cataracts. On testing the urine, there is positive Fehling’s and Benedict’s reagent tests with a negative glucose oxidase strip test. The milk is eliminated from the child’s diet and immediately some of the symptoms improve.

What is the diagnosis?

A Fructose intolerance
B Galactosaemia
C Galactokinase deficiency
D Urea cycle disorder
E Tyrosinaemia
A

B
This neonate, born with cataracts, poor feeding, lethargy, conjugated hyperbilirubinaemia with hepatomegaly and reducing sugars in the urine after starting milk, is likely to have galactosaemia (B). This is a rare autosomal recessive inherited condition most commonly due to a mutation in the galactose-1-phosphate uridyltransferase gene on chromosome 9p13. It results in excessive galactose concentrations when milk, which contains glucose and galactose, is introduced into the baby’s diet.
Galactose can enter the metabolic pathway through a number of steps. It must first be phosphorylated to allow its conversion into glucose- 1-phosphate which eventually become glucose-6-phosphate to finally enter the metabolic cycle. Galactose-1-phosphate uridyltransferase con- verts galactose-1-phosphate into UDP galactose. This is the most common enzyme to be defective in galactosaemia. It is unclear exactly why the build up of galactose is so harmful, however one of the by products of its metabolism (galactitol produced by aldolase on galactose-1- phosphate) is responsible for cataract formation. The collection of GI symptoms, hepatomegaly and cataracts on starting milk is very suggestive of this disease. Children with this disease are also more susceptible to sepsis with Escherichia coli. The Fehling’s and Benedict’s reagent tests are positive because galactose is a reducing sugar, the other important one being glucose which was excluded using glucose specific sticks. The investigation of choice is a red cell galactose-1-phosphate uridyltransferase level although this condition is sometimes screened for during the neonatal period in certain parts of the world. Treatment is to exclude milk from the child’s diet as well as eliminating other sources of galactose.

Galactokinase (C) deficiency is another cause of galactosaemia but much less common. It is due to a defective galactokinase gene on 17q24. Its function is to phosphorylate galactose to galactose-1-phosphate. Unlike classical galactosaemia as described above, severe symptoms in early life are less common. Instead, excess galactitol formation results in early cataract formation in homozygous infants. Treatment is similar to those with classical galactosaemia.

Fructose intolerance (A) is caused by fructose-1-phosphate aldolase deficiency which normally converts fructose-1-phosphate to dihydroacetone phosphate and glyceraldehyde. These products are further metabolized and can enter either glycolytic or gluconeogenesis path- ways depending on the energy state of the cell. The explanation is made more complicated by the fact that there are three isoenzymes of fructose-1-phosphate aldolase (A, B and C) of which B is expressed exclusively in the liver, kidney and intestine as well as metabolizing three different reactions. Aldolase B can produce triose phosphate com- pounds which are central to the glycolytic pathway, but this can also be reversed making it important in gluconeogenesis. A deficiency therefore explains the hypoglycaemia experienced by these patients. Furthermore, the reduced fructose metabolism increases its blood levels which consequently changes the ATP:ADP ratio. This increases purine metabolism resulting in excess uric acid production which competes for excretion in the kidney with lactic acid. The result is lactic acidosis, hyperuricaemia and hypoglycaemia. These is also severe hepatic dysfunction, the pathophysiology of which is relatively less well understood.

Tyrosinaemia (E) is another autosomal recessive inherited disorder of metabolism which has three subtypes – types I, II and III. Type I is the hereditary form which has a specifically high incidence in Quebec, Canada and is characterized by a defect in fumarylacetoacetate hydrolase. In its most severe form it presents with failure to thrive in the first few months, bloody stool, lethargy and jaundice. A distinctive cabbage-like odour is characteristic. On examination there is hepatomegaly with signs of liver failure and subsequent survival for less than 12 months if untreated. The investigation of choice is urinary succinylacetone and treatment is to restrict dietary tyrosine and phenylalanine and to treat the liver failure, sometimes with a transplant.

Urea cycle disorders (D) normally present with a non-infective encephalopathy, failure to thrive and hyperventilation in the neonatal period progressing to neurological symptoms associated with protein intake. The inability to metabolize urea leads to hyperammonaemia. A blood level above 300μM/L is associated with encephalopathy. There are also associated increases in plasma amino acids, urine amino acids and organic acids. Enzyme studies are required to differentiate it from one of the ten potential defects responsible for this group of diseases. Tx: use benzoate or phenylacetate or extracorporeal dialysis to remove ammonia and a low protein diet to prevent its build up.

How well did you know this?
1
Not at all
2
3
4
5
Perfectly
8
Q
  1. Neonatal jaundice
    A 2 week old neonate born at term with no gestational complications develops uncongutated jaundice. This was following a difficult birth where instrumentation was required after excessive delay in the second stage of labour. On examination, the neonate looks well in a normal flexed position with visible jaundice most noticeable in the soft palate. There are no abnormal facies but there is a visible large caput succedaneum with bruising. Urine dipstick is normal with no markers of infection present in the blood. What is the most likely cause of the jaundice?
A Urinary tract infection
B Bruising
C Haemolysis
D Crigler–Najjar syndrome
E Gilbert’s disease
A

B
This child, with a large amount of bruising (B), most probably developed unconjugated jaundice from the excess breakdown products of erythro- cytes. The difficult labour requiring instrumentation has led to a large collection of bruising in the scalp which is broken down and leads to unconjugated jaundice. Neonates are susceptible to jaundice for many different reasons – reduced erythrocyte half life with increased haemo- globin levels, reduced transport in the liver (reduced ligandin is respon- sible for this) and increased enterohepatic circulation. Investigation of this is to rule out other causes including urinary tract infection, other haemolytic anaemias and congenital hypothyroidism which is normally tested for by the heel prick Guthrie test. Treatment is usually via pho- totherapy which uses light at 450nm wavelength to solubilize (NOT conjugate) the excess bilirubin for excretion through the kidneys. This prevents passage of bilirubin through the immature blood–brain bar- rier which can then deposit into the basal ganglia causing kernicterus. Another method of treatment includes exchange transfusion. Other causes of jaundice include haemolysis (C), which may be con- genital or acquired. Congenital causes include G6PD deficiency which can cause severe unconjugated jaundice. The mutation in this enzyme reduces erythrocyte ability to withstand oxidative stress which can
be triggered by numerous drugs (classically anti-malarials) and fava beans (hence the alternative name for this condition is favism). Other causes of haemolysis include ABO incompability where blood type O mothers sometimes express IgG anti-A-haemolysins which can cross the placental barrier resulting in haemolysis. Treatment is supportive. Rhesus haemolytic disease is serious but fortunately rare with the implementation of anti-D immunization after significant events. In this situation, a mother has anti-D antibodies which cross the placental barrier resulting in profound haemolysis, hydrops and hepatospleno- megaly. This requires previous sensitization of the mother to rhesus D antigen either by previous pregnancy or blood transfusion. Therefore all pregnant women who are rhesus D negative receive prophylactic immunoglobulins during significant events in pregnancy which may release fetal blood into the maternal circulation, e.g. abortion. The immunoglobulins effectively neutralize the fetal blood and prevent an immune response from developing. Failure to do this will risk the next rhesus positive fetus.

Urinary tract infection (A) is a common cause of unconjugated jaundice in the neonate and must be excluded because if left untreated it can lead to complicated urinary tract infection involving the kidneys and urosepsis. Sepsis in neonates does not always present with fever but instead an inability to regulate body temperature. The most common pathogen is group B streptococcus, a common commensal in the vaginal tract of the mother.
Crigler–Najar syndrome (D) is caused by a genetic defect in glucoro-
nyl transferase which is responsible for transporting bilirubin into the hepatocyte. There are two types – type I is characterized by a complete absence of this enzyme, type II is characterized by a partial reduction
of this enzyme. Type I presents with severe neonatal jaundice with ker- nicterus, phototherapy can reduce the levels by half and liver transplan- tation is the only cure. Phenobarbitone is used only in type II Crigler– Najjar syndrome. This disease is different from Gilbert’s disease (E) which is relatively common but also causes a mild unconjugated hyper- bilirubinaemia. The main defect is in biliribuin uridinediphosphate- glucuronyltransferase (UGT1A1) which is the enzyme responsible for conjugating bilirubin and is reduced by about 30 per cent in Gilbert’s disease. It does not cause liver damage and is relatively benign. Precipitating factors include stress, fasting, fever and dehydration. Investigations aim to prove an unconjugated jaundice without hae- molysis and normal plasma bile acids. There is no bilirubinuria and no increase in urobilinogen either.

How well did you know this?
1
Not at all
2
3
4
5
Perfectly
9
Q
  1. Vitamin D deficiency
    A 54-year-old man with a past history of alcohol abuse, recurrent severe epigastric pain with flatulence and steatorrhoea presents after a fall whilst out drink- ing with his friends. He had fallen onto his hip, has severe pain and inability to weight bear. On examination, his right lower limb is shortened and externally rotated. His liver function tests were normal apart from a raised alkaline phos- phatase. A fractured neck of femur is diagnosed and is fixed that night. As part of a routine follow up, the fracture liaison nurse suspects vitamin D deficiency and orders a full set of vitamin D levels. What set of results would you expect in this man given his history?

A Low 25-hydroxycholecalciferol, low 1,25-dihydroxycholecalciferol, low parathyroid hormone
B Low 25-hydroxycholecalciferol, high 1,25-dihydroxycholecalciferol, high parathyroid hormone
C High 25-hydroxycholecalciferol, low 1,25-dihydroxycholecalciferol, high parathyroid hormone
D High 25-hydroxycholecalciferol, high 1,25-dihydroxycholecalciferol, high parathyroid hormone
E High 25-hydroxycholecalciferol, low 1,25-dihydroxycholecalciferol, low parathyroid hormone

A

B
This man is highly likely to have osteomalacia given the history of chronic alcohol abuse and episodes consistent with chronic pancreatitis. This is significant because the pancreas is responsible for emulsification and digestion of fats which facilitate fat soluble vitamin absorption including vitamins A, D, E and K. The reduced vitamin D absorption has led to osteomalacia, the pathological syndrome caused by vitamin D deficiency after epiphyseal closure. If vitamin D deficiency occurred before epiphyseal closure, the patient would suffer from rickets.
Vitamin D metabolism involves the skin, liver and kidneys as well as the bones and gastrointestinal tract. Sources of vitamin D include sunlight exposure and diet. Sunlight converts 7-dehydrocholesterol into cholecalciferol (vitamin D3). The latter product is what is consumed in the diet. This is then hydroxylated in the liver to form 25-hydroxycholecalciferol. This is then transported to the kidneys where the final hydroxylation by 1 alpha hydroxylase converts 25-hydroxy-cholecalciferol to 1,25-dihydroxycholecalciferol. This final step is stimulated by parathyroid hormone.
Therefore, this man has low 25-hydroxy-cholecalciferol levels due
to reduced absorption of dietary vitamin D, but has a high level of 1,25-dihydroxycholecalciferol because of the reactive secondary hyperparathyroidism which converts any remaining 25-hydroxy-cholecalciferol to the activated form, hence the high levels (B). Answer (A), where there are low levels of both forms of vitamin D, could also be present in this situation but there would be a high PTH level making this answer incorrect.
A high 25-hydroxy-cholecalciferol, low 1,25-dihydroxy-cholecalciferol, high parathyroid hormone (C) would occur in patients with chronic renal failure where there is loss of parenchymal tissue to hydroxylyze 25-hydroxy-cholecalciferol to its final activated form. There is a secondary or tertiary hyperparathyroidism depending on the stage of renal failure. Secondary hyperparathyroidism occurs early on when the kidneys retain phosphate and appropriately stimulate PTH secretion. As the renal failure continues, the gland secretes PTH autonomously despite normal or high calcium levels.
A high 25-hydroxy-cholecalciferol, high 1,25-dihydroxy-cholecalciferol, high parathyroid hormone (D) would occur in patients with vitamin D resistance where there is normal production of vitamin D but there is reduced activity due to the inability to detect vitamin D. There are two types – type 2 vitamin D dependent rickets is autosomal recessive and is caused by an end organ resistance whereas type 1 is caused by a congenital lack of 1 alpha hydroxylase giving a similar biochemical profile to that seen in chronic renal failure. Parathyroid hormone levels are high despite high vitamin D levels because PTH is under negative feedback control from calcium and phosphate levels, not vitamin D levels.

Finally a high 25-hydroxy-cholecalciferol, low 1,25-dihydroxy-cholecalciferol, low parathyroid hormone may be seen in hypoparathyroidism of which the most common cause is post-surgical intervention. There is a low PTH level and therefore low stimulation of 1 alpha hydroxylase in the kidney to covert 25-hydroxy-cholecalciferol to 1,25-dihydroxy- cholecalciferol thus explaining their levels.

How well did you know this?
1
Not at all
2
3
4
5
Perfectly
10
Q
  1. Raised alkaline phosphatase
    Which of the following is not a cause of raised ALP levels?
A Pregnancy
B Paget’s
C Congestive heart failure
D Obstructive jaundice
E Myeloma
A

E
Alkaline phosphatase (ALP) is an enzyme responsible for removing phosphate groups from various molecules. It is produced in the liver, bile duct, kidney, bone and placenta. It is commonly requested as part of the liver function test panel and is used diagnostically in the approach to various conditions.
Of these answers, only myeloma does not classically cause a raised ALP. ALP is caused by osteoblast activation whereas in myeloma there is direct osteoclast activation through the release of various cytokines.
This means although there are areas of lysis on X-rays, there is little osteoblast response leading to a normal alkaline phosphatase level. This may be complicated by a fracture which will stimulate osteoblast activ- ity leading to a raised ALP in the setting of myeloma. Paget’s disease, a syndrome characterized by abnormal remodelling, normally has a very elevated level of ALP caused by increased but disorganized remodelling of the bone. On X-rays there are patches of lucency and sclerosis. There are normally no calcium or phosphate abnormalities making it differ- ent from metastatic prostatic cancer which also gives a patchy sclerotic X-ray but would raise the calcium levels.
There are various isomers of ALP which are not distinguishable on a standard liver function assay without electrophoresis. In the third tri- mester of pregnancy (A), placental ALP is produced leading to raised levels if one were to measure them at this time. Another isoenzyme is found in the liver and bile ducts where it is used to distinguish between an obstructive and hepatic picture in liver disease. Here, obstruction or damage to the bile ducts cause a disproportionately raised ALP com- pared with the AST and ALT. Another way of distinguishing whether an isolated raised ALP is originating in the liver is to look at the GGT
– this often rises with bile duct injury whereas it would be normal if the ALP were of bone or placental origin.
Finally congestive cardiac failure can cause a mildly raised ALP and may be due to reduced forward flow of blood causing congestion in the liver and release of ALP into the systemic circulation. The causes of a raised ALP are many and can be categorized into the following:
1 Liver related – cholestasis, hepatitis, fatty liver, tumour
2 Drugs – phenytoin, erythromycin, carbamezepine, verapamil
3 Bones
–> Bone disease – Paget’s disease, renal osteodystrophy, fracture
–> Non-bone disease – vitamin D deficiency, malignancy, secondary hyperparathyroidism
4 Cancer (different from metastases to bones) – breast, colon cancer and Hodgkin’s lymphoma

How well did you know this?
1
Not at all
2
3
4
5
Perfectly
11
Q
  1. Paradoxical aciduria
    A 19-year-old female student presents to the GP with low mood, lethargy and muscle weakness. She is anxious that she is putting on weight and admits to purging after meals to keep her weight under control for several months. She has a past history of depression and is taking citalopram. On examination, her body mass index is 18, she is clinically dehydrated with signs of anaemia including conjunctival pallor. She has bilateral parotidomegaly and the GP also notices erosions of the incisors. He orders some blood tests which reveal the following:
Hb 9.5
White cells 7.8
Platelets 345
Na 143
K 3.1
Urea 8.5
Creatinine 64
Arterial pH 7.49

Urinalysis is normal except for acidic urine. The cause of this patient’s acidic urine is:

A Acute renal failure
B Renal tubular acidosis
C Citalopram
D Anaemia
E Physiological
A

E

This is a difficult question but the answer can be deduced with a basic knowledge of electrolyte physiology. This patient suffers from bulimia nervosa as characterized by the use of characteristic purging after meals to keep her weight under control. The main abnormalities in the investigations reveal a hypokalaemia with arterial alkalosis and paradoxical aciduria. The alkalosis is likely to be due to excessive purging leading to a loss of hydrogen ions. The hypokalaemia is secondary to the metabolic alkalosis as potassium and hydrogen are transported across cell membranes by the same transporter. The reduction of plasma hydrogen ions leads to increased potassium uptake (excretion in urine) leading to hypokalaemia. As part of a normal homeostatic mechanism, potassium is exchanged for hydrogen ions in the distal convoluted tubule of the nephron, resulting in an apparent paradoxical aciduria.

Acute renal failure (A) tends to give hyperkalaemia and metabolic acidosis. This is due to the failure of homeostatic mechanism, the causes of which are classically defined as pre-renal, renal or post- renal. Pre-renal failure is caused by a reduction in GFR. This may be due to reduced blood flow or reduced perfusion pressure. Common causes include hypovolaemia or hypotension from shock. Intrinsic renal failure has a wide aetiology including drugs, inflammation and infection. Post-renal failure is caused by obstruction anywhere from the collecting ducts distally. classically presents in elderly men with prostatic disease with urinary retention relieved by catheterization.

Citalopram (C) is a selective serotonin reuptake inhibitor (SSRI) used in the treatment of depression. Some SSRIs cause hyponatraemia, but not usually hypokalaemia.

Renal tubular acidosis (B) generally causes a lack of ability to acidify urine and hyperkalaemia. The exception is type II renal tubular acidosis with a bicarbonate leak in the proximal convoluted tubule where hypokalaemia is common but the urine is only acidified during systemic metabolic acidosis. This is not the case in this patient.

Finally anaemia (D) does not usually cause electrolyte abnormalities.

How well did you know this?
1
Not at all
2
3
4
5
Perfectly
12
Q
  1. Nutritional deficiency
    A 44-year-old African man is seen by a volunteer doctor in his village with skin changes around the neck. There are erythematous and pigmented areas around the neck in a necklace-like distribution. His family is also complaining of him becoming more forgetful and unable to perform normal daily tasks. This is made particularly distressing given his increase in bowel movements, although he cannot remember how many times he goes. He and his family, like many of the villagers, eat almost exclusively maize, and the doctor has treated several cases of kwashiorkor in the local area. What is the nutritional deficiency most likely to explain his symptoms?
A Tocopherol
B Riboflavin
C Retinol
D Vitamin B3
E Ascorbate
A
D
This man with poor diet, dermatitis, dementia and diarrhoea most likely has a niacin deficiency leading to pellagra. The other name for niacin is vitamin B3 (D).The rash he describes is also known as Casal’s necklace – a distinctive erythematous, pigmented rash in the necklace distribution named after Gaspar Casal, a Spanish physician practising in the early 1700s. Niacin is essential for most cellular processes but only usually affects those with severe malnutrition because tryptophan can also be converted into niacin, therefore a dual deficiency is required for the full syndrome to develop. The disease is remembered by the four Ds – dementia, diarrhoea, dermatitis and death. The neurological symptoms do not exclusively manifest as dementia – other symptoms also include depression, anxiety, tremor, delusions, psychosis and even coma. The diarrhoea occurs in about half of patients furthering the malnutrition problem. Dermatitis can affect the mouth, lips, hands, arms, legs and feet. The causes are primary niacin deficiency due to poor nutrition – this is the most likely case in this question given the maize diet and the suggestion of protein malnutrition by the presence of kwashiorkor in the local population. Secondary niacin deficiency may be secondary to malabsorptive problems including prolonged diarrhoea, inflammatory bowel disease and liver cirrhosis. Iatrogenic causes are well described – implicated drugs include isoniazid and azathioprine. Treatment is with niacin replacement therapy and treatment of underlying disease if it is secondary.
Tocopherol (A) is also known as vitamin E, its deficiency causes haemo- lytic anaemia, spinocerebellar degeneration and peripheral neuropathy. It is rare in humans. It is one of the fat soluble vitamins (the others being A, D and K) and is important in normal reproduction, muscular devel- opment and resistance to red cell haemolysis. It is stored in the liver, adipose tissue, muscle, pituitary gland, testes and adrenals. Its levels are directly measured in the plasma. Recently in the HOPE (Heart Outcomes Prevention Evaulation) study, vitamin E was found to have no evidence of benefit in preventing the development of cardiovascular disease.
Riboflavin deficiency (B), also known as vitamin B2, causes ariboflavino- sis. Symptoms include dry mucous membranes affecting the mouth, eyes and genitalia along with a normocytic normochromic anaemia. It is usu- ally associated with protein and energy malnutrition or alcoholism and is normally found in legumes, pulses and animal products. Riboflavin
is an essential constituent in two molecules – flavin mononucleotide
and flavin adenine dinucleotide (FAD). These molecules readily accept and donate electrons making them ideal coenzymes in redox metabolic reactions. Riboflavin is absorbed in the proximal small intestine, its defi- ciency can be tested for by assaying erythrocyte levels, or assaying the activity of erythrocyte glutathione reductase which requires FAD for its activity. Treatment of this deficiency is daily supplementation.
Retinol (C), also known as vitamin A, is another fat soluble vitamin whose function is necessary for normal epithelial tissue growth, poly- saccharide synthesis and the formation of visual pigment, rhodopsin. Vitamin A deficiency can cause dry skin and hair as well as xeroph- thalmia (drying of the cornea with ulceration). Rarely, Bitot’s spots can develop and are seen on the conjunctiva and represent an accumulation of keratin. Vitamin A deficiency can also cause night blindness due to rhodopsin abnormalities as well as a distinctive skin rash called pityria- sis rubra pilaris. Treatment is with a balanced diet and supplementation. This is not without caution – vitamin A can be toxic: there are reports of Arctic explorers eating polar bear liver who developed headache, diarrhoea and dizziness. Vitamin A consumption, especially in liver, is also cautioned in pregnant women as it may be teratogenic.
Ascorbate (E) is also known as vitamin C which, if deficient, causes scurvy. The features of scurvy include anaemia, bleeding gums and induration of the calf and leg muscles. This is due to ascorbate’s role in the formation of collagen including that of bone, cartilage, teeth and intercellular substance of capillaries. This explains the defective ossi- fication and bleeding tendency. Unsurprisingly, wound healing is also poor. Vitamin C also improves the efficacy of desferrioxime, an iron chelator used in states of iron overload, which may be due to vitamin C’s antioxidant action.
How well did you know this?
1
Not at all
2
3
4
5
Perfectly
13
Q
  1. Therapeutic drug monitoring
    A 51-year-woman with epilepsy is admitted after suffering a seizure following non-compliance with her phenytoin. She admits to having problems at home and was finding it difficult to continue to take her medication regularly. She is restarted on phenytoin. How many half lives does it normally take for a drug to reach its steady state?
A 1–2 half lives
B 3–5 half lives
C 10–11 half lives
D 50–60 half lives
E 100–150 half lives
A

B
Usually, drugs take between 4 and 5 half lives to reach a steady state. The half life is the time it takes for the plasma concentration of the drug to halve. Drugs such as phenytoin are monitored because under- dosing will lead to no effect but overdosing will lead to toxicity. Most drugs have a wide therapeutic window – that is the difference between the minimum effective concentration and minimum toxic concentration. Drugs with narrow therapeutic windows may be suitable for drug moni- toring to optimize treatment.
This figure can be calculated relatively simply. Let us consider we give a patient a single dose of a drug with a half life of 24 hours. This means 50 per cent of the medication will be eliminated in 24 hours, but 50 per cent will remain. On day 2, 24 hours after the first dose, we give anoth- er dose. On day 3 there is now 75 per cent of the original doses of drug in the patient’s circulation – the original dose which has been in the system for two half lives and therefore is at 25 per cent, and the second dose which has been in the system for one half life and is therefore at 50 per cent of the original dose – giving 75 per cent. Continuing this daily, the amount of drug in steady state by day 4 is 93.75 per cent, by day 5 it is around 97 per cent. This is for a drug with a half life of 24 hours (this is approximately phenytoin’s half life); but this holds true for any half life except those drugs with very short half lives.
This also explains why loading doses are used. If, say, a drug has a half life of 1 week, then it would take up to 5 weeks for the patient to be within the therapeutic range. Therefore, loading doses are used to increase the initial blood concentration and reduce the time needed to reach steady state.
Drugs which require therapeutic drug monitoring include:
- Antibiotics, e.g. gentamicin, vancomycin
- Anticonvulsants, e.g. phenytoin, lamotrigine
- Immunosuppressives, e.g. methotrexate, mycophenolate, tacrolimus
- Lithium
- Digoxin
Unfortunately, determining drug efficacy is more complicated than simply measuring its plasma concentration. The efficacy depends on both pharmacokinetic and pharmacodynamic factors. Pharmacokinetic factors relate to the absorption, distribution, metabolism and excretion of the drug. Absorptive factors include water/fat solubility of the drug or specific transport mechanisms across the mucosal lining of the gut, e.g. grapefruit juice increases ciclosporin bioavailability. Distributive factors take into account the water solubility and fat solubility of the drug as well as the amount of fat or water the patient has. A useful method of measurement of this concept is the volume of distribution which describes the volume of water required to completely account for the administered drug at the given plasma concentration. If the drug has a high fat solubility, the concentration in the plasma will
be relatively low, therefore it would require a high volume at the
given concentration to account for the drug given. Metabolism fac- tors include pharmacogenetic factors, e.g. thiopurine methyltransferase mutation affects the administration of azathioprine as reduced levels are more likely to lead to toxicity. These factors also include the phase I and phase II type reactions which are involved in oxidation/reduction and solubilization of the drug, respectively. Excretive factors are most- ly to do with renal function; some drugs (e.g. digoxin) can accumulate in renal failure.

The pharmacodynamic factors that must be considered include whether the drug is in the active form when administered. Some drugs are required to be metabolized before they have their therapeutic effect, e.g. azathiopurine (metabolized to mercaptopurine), enalapril (metabolized to enalaprilat) or carbimazole (metabolized to methimazole) or the drug is active but its metabolic products are also active, e.g. codeine and tramadol. Another important pharmacodynamic factor to consider is the degree of drug bound to protein. Traditionally drug levels are quoted as a total drug level which includes both bound and unbound drug but only the unbound drug is active. Drugs which are highly protein bound have an altered therapeutic effect in low protein states or if another drug has a higher protein affinity therefore displacing the former drug and increasing the proportional unbound active drug. Phenytoin is important to remember as it is highly protein bound (90–94 per cent). Others include mycophenolate and carbamazepine.

How well did you know this?
1
Not at all
2
3
4
5
Perfectly
14
Q
  1. Hypoglycaemia
    A 67-year-old Indian man presents with irritability, sweating and tremor which progresses to stupor. The admitting doctor sends for a laboratory glucose which comes back at 2.2mmol/L. The patient is resuscitated and given intravenous glucose. A history reveals that he does not suffer from diabetes, and his past medical history is remarkable only for vitiligo. On direct questioning he admits to feeling increasingly more tired, particularly after returning recently from India. His family arrive after which the doctor notices the patient’s unusually darker tan compared with his children. Further investigations reveal the patient has low insulin and low C peptide concentrations.

What is the most likely diagnosis?

A Pituitary failure
B Addison’s disease
C Alcohol induced
D Glycogen storage disease
E Medium chain acyl-CoA dehydrogenase deficiency (MCADD)
A

B
This patient, presenting with hypoglycaemia, tiredness and hyperpig- mentation with an associated autoimmune history of vitiligo, most probably has adrenal failure (Addison’s disease (B)). The adrenal glands are responsible for producing cortisol, aldosterone and sex hormones. Adrenal failure is potentially lethal due to the lack of cortisol, which is an important stress hormone as well as an important gluconeogenesis stimulant at times of hypoglycaemia. An important worldwide cause is tuberculosis but in the developed world, autoimmunity is more likely. Autoimmune conditions often segregate as in this man with vitiligo,
an autoimmune disease causing destruction of melanin in the skin. The patient has a tan as a by product of the lack of negative feedback in the hypothalamic–pituitary–adrenal axis. The hypothalamus releases cortisol releasing hormone (CRH) to the anterior pituitary which in turn releases ACTH (adrenocorticotropic hormone). ACTH is produced from its precur- sor molecule POMC (pro-opiomelanocortin) which, when cleaved, also produces MSH (melanocyte stimulating hormone). This accounts for the increased tanning seen in patients with Addison’s.
In patients with hypoglycaemia, a plasma insulin and C peptide is diag- nostically useful to elucidate the cause. Insulin is the main endogenous hypoglycaemic and is released from beta cells in the pancreas. C peptide is a by product of insulin production and therefore has a direct correlation with endogenous insulin production. Causes of raised insulin and C pep- tide concentrations are few and include islet cell hyperplasia (e.g. persis- tent hyperinsulinaemic hypoglycaemia of infancy, Beckwith Weidemann syndrome) or insulinoma. If insulin were exogenously administered, then the C peptide level would be low because endogenous production would be appropriately suppressed.
All of the answers given can cause hypoglycaemia with low insulin and C peptide levels. Pituitary failure (A) with TSH and ACTH failure can cause hypoglycaemia. In this patient other symptoms and signs sug- gesting pituitary failure would manifest, e.g. sex hormone deficiency leading to loss of libido, menopause in women; lack of growth hormone leads to muscle atrophy, abdominal obesity; lack of dopaminergic inhi- bition to prolactin leads to galactorrhoea, amenorrhoea and infertility; lack of TSH leads to hypothyroidism. Alcohol induced (C) hypoglycae- mia occurs due to the increased production of cytosolic NADH from ethanol metabolism into acetaldehyde. NADH inhibits gluconeogenesis resulting in hypoglycaemia. Chronically, chronic alcoholism leads to malnutrition thus reducing the hepatic glycogen stores. Glycogen stor- age disease (D), more specifically glycogen storage disease type I (Von Gierke’s disease) is caused by a mutation in the glucose-6-phosphatase enzyme. Phosphorylated glucose cannot cross cell membranes and therefore the lack of this enzyme essentially traps glucose from being transported. Patients present with stunted growth, hepatomegaly and have hypoglycaemia, lactic acidosis, high urate and high triglycerides (GLUT). Finally medium chain acyl-CoA dehydrogenase deficiency (MCADD) is caused by a genetic defect in fatty acid beta oxidation. This is important in ketone body formation in hypoglycaemia, which the brain must use to preserve function as it cannot utilize fats directly in states of neuroglycopenia. This mutation leads to hypoketotic hypoglycaemia often with hepatomegaly and cardiomyopathy.

How well did you know this?
1
Not at all
2
3
4
5
Perfectly
15
Q
  1. Acute pancreatitis
    A 56-year-old presents with sudden onset, severe epigastric pain which radiates through to the back. The pain is relieved only partly by sitting forward and is associated with nausea. The admitting doctor suspects pancreatitis and sends for a serum amylase which is greatly raised. A diagnosis of acute pancreatitis is made. The following results come back following a blood test:
Haemoglobin 14.5g/dL
White cells 14.2
Na 148
K 4.6
Urea 14
Creatinine 123
Calcium 2.98 (corrected) Cholesterol 5.5
Albumin 35g/L
Glucose 8.8mmol/L

Which biochemical abnormality is not likely to be a consequence of acute pancreatitis?

A Raised white cells
B Raised sodium
C Raised urea and creatinine
D Raised calcium
E Raised glucose
A

D
Hypercalcaemia is not a common consequence of acute pancreatitis, indeed hypercalcaenia is one of the causes of acute pancreatitis. Other causes of pancreatitis can be remembered by the well known mnemonic ‘GET SMASHED’:
- Gallstones
- Ethanol
- Trauma
- Steroids
- Mumps
- Autoimmune (polyarteritis nodosa)
- Scorpion venom (Trinidadian scorpion)
- Hypercalcaemia/Hypertriglyceridaemia/Hypothermia
- Endoscopic retrograde cholangiopancreatogram
- Drugs (including thiazides, azathioprine, valproate, oestrogens)
Corrected calcium is used instead of calcium because the latter is dependent on albumin concentration which binds 40 per cent of plasma calcium and is normally quoted by laboratory studies. The ionized non- bound calcium is the important measurement clinicians are usually interested in; therefore the corrected value is used which takes into account albumin concentration. If the laboratory has not quoted a corrected calcium, one can calculate the corrected value by subtracting 0.1mmol/L from the calcium concentration for every 4g/L the albumin is below 40g/L.
The mechanism of aetiology related to hypercalcaemia is unknown. Some theorize that hypercalcaemia results in small intraductal stones in the pancreas causing blockage. Others believe hypercalcaemia directly increases pancreatic exogenous enzyme output or direct activation
of trypsinogen. Pancreatitis is a potentially life-threatening disease with progression to systemic inflammatory response syndrome (SIRS) and multiorgan failure is a well recognized complication. Scoring systems which help to predict severity do exist, perhaps the most easily remembered is the modified Glasgow scoring system:

PaO2 <8kPa
Age >55 years
Neutrophilia – white blood cells >15×109/L
Calcium <2 mmol/L
Renalfunction urea>16mmol/L
Enzymes – LDH >600iu/L or AST >200ui/L
Albumin <32 g/L
Sugar >10 mmol/L
Scoring three or more of these criteria within 48 hours of admission should prompt early intensive care unit referral. Inspecting this list,
this patient’s other biochemical abnormalities can be explained from
the inflammatory response to the pancreatitis. A raised white cell count (A) is due to the response of necrotic tissue in the pancreas which is being degraded by the inappropriate activation of trypsin, a powerful protease enzyme. A raised white cell count can also be secondary to
the SIRS response as well as infection of the necrotic tissue. The raised sodium (B) and raised urea and creatinine (C) are likely to be secondary to dehydration which is multifactorial – nausea and vomiting and third space sequestration of fluid from the inflamed pancreas. Acute renal failure in pancreatitis is a devastating complication – one study found the risk factors for developing acute renal failure were previous renal disease, hypoxaemia and abdominal compartment syndrome. A raised glucose (E) is due to the pancreatic endocrine dysfunction where glucose monitoring and insulin release are impaired leading to hyperglycaemia. Hypocalcaemia is a complication of pancreatitis and is due to the fat saponification from the released enzymes.

How well did you know this?
1
Not at all
2
3
4
5
Perfectly
16
Q
  1. Treatment of hyperkalaemia
    A 76-year-old man presents following a fall and is diagnosed with a pubic ramus fracture which is treated conservatively. He has a background of chronic renal failure and over the weekend starts to feel palpitations and lightheadedness. An ECG is performed which shows tenting of the T waves, suggestive of hyperkalaemia. A blood test is performed which confirms the diagnosis.

Which of the following treatments does not lower plasma potassium levels?

A Calcium resonium
B Sodium bicarbonate
C Calcium gluconate
D Insulin
E Salbutamol
A

C
Hyperkalaemia over 6.5mmol/L is a medical emergency. High extracellular potassium levels increase cardiac excitability lowers the threshold of fatal dysrhythmia. Classical electrocardiographic changes include tall tented T waves, small P waves, widened QRS complexes which eventually become sinusoidal and can degenerate into ventricular fibrillation. Ten millilitres of 10 per cent calcium gluconate is the first line medication given to anyone with hyperkalaemia. It does not change the plasma potassium levels but stabilizes the myocardium to help prevent fatal dysyhythmia. It does so by increasing the threshold potential making the myocardium less excitable.

Calcium resonium (A) can be given orally or per rectum and reduces the plasma potassium levels over the longer term (around hours). This is therefore not helpful in the acute situation this patient is in, but may be considered once the potassium level is controlled. It binds potassium within the gut to increase excretion of ingested potassium therefore lowering overall potassium absorption. Its side effects unsurprisingly include gastrointestinal upset, including nausea and vomiting.

Insulin (D) along with dextrose is the main treatment to reduce potas- sium concentration acutely. Insulin drives potassium into cells along with glucose. Insulin must not be given alone as one could precipitate hypoglycaemia, the mechanism of action is within 20–30 minutes.
Nebulized salbutamol (E) is an example of a beta-2 receptor ago-
nist which reduces potassium plasma concentration by activating the sodium–potassium–ATPase pump. This ubiquitous enzyme uses energy to transfer sodium and potassium to the extracellular and intracellular spaces respectively. In one recent study, it was shown that more lipophilic beta 2 agonists such as formeterol were more efficacious at reducing potassium plasma levels.
Sodium bicarbonate (B) does not directly lower plasma potassium levels, but instead neutralizes any excess acid in the blood. Bicarbonate reacts with hydrogen ions to produce carbon dioxide and water by increasing the bicarbonate levels, excess hydrogen ions are used in this reaction which raises the pH. Hydrogen and potassium compete at the cell membrane for entry into the cell; if hydrogen ion concentration decreases, a relative abundance of potassium is present making it more likely to enter the cell. This therefore lowers potassium levels, hence sodium bicarbonate indirectly can affect potassium levels.
How well did you know this?
1
Not at all
2
3
4
5
Perfectly
17
Q
  1. Myocardial infarction
    A 54-year-old man is admitted for an elective shoulder repair. The day before his surgery he develops acute onset central crushing chest pain radiating to his left arm and up the jaw. He is also sweaty and feels nauseous. He has a past medical history of coronary artery bypass grafting and angina, and his father died from a heart attack aged 46. An electrocardiogram is performed which shows acute ST elevation in the inferior leads. He is diagnosed with acute coronary syndrome and treated appropriately. His surgery is delayed, but he presents with the same symptoms 2 days later with further ST changes in the lateral leads. Which cardiac enzyme is most useful to confirm re-infarction?
A Troponin I
B Troponin T
C Aspartate transaminase
D Creatine kinase muscle brain (CK MB)
E Lactate dehydrogenase
A

D
This question is difficult as it requires both knowledge of the relative sensitivities of cardiac enzymes and their relative timelines at which they stay raised after a recent infarction. CK MB (D) is the heart iso- enzyme creatine kinase which rises about 6–12 hours post-infarction and it usually peaks in concentration 24 hours later. It then reduces to normal within 48–72 hours. It is very sensitive and is diagnostic if it is >6 per cent of total creatine kinase or the CK MB mass is >99 percentile of normal. It is very useful in detecting re-infarction because of its sen- sitivity and rapid return to normal levels compared with troponin I and T (A and B). Troponin is the most sensitive and specific test for MI and is traditionally taken 12 hours post-infarction. Troponin I is a better marker of myocardial infarction compared with troponin T (Trop I: sensitivity and specificity of 90 per cent at 8 hours and 95 per cent, respectively, trop T 84% at 8 hours and 81%, respectively). However, troponin levels take up to 10 days to normalize, making their use in re-infarction soon after a primary infarct, limited. Another reason troponin is not the correct answer is that they are not strictly speaking cardiac enzymes, but rather a structural protein in the contractility mechanism. Interestingly, troponin T is also elevated in chronic kidney disease without troponin I elevation, for reasons unknown.
AST (C) rises around 24 hours after an infarct and remains raised for 48 hours but is less sensitive and specific. It is also raised in liver disease, skeletal muscle damage (particularly in crush injury) and haemolysis. Similarly, LDH (E) rises around 48 hours after myocardial infarction and remains elevated for up to a week. It is also not very specific – it can be raised in liver disease, haemolysis, pulmonary embolism and tumour necrosis. For a summary of cardiac enzyme changes with time see the figure below.

How well did you know this?
1
Not at all
2
3
4
5
Perfectly
18
Q
  1. Hyponatraemia
    A 55-year-old man with severe learning difficulties presents with shortness of breath on exertion, fever and a productive cough of rusty red sputum. On exami- nation, there is increased bronchial breathing in the lower right zone with inspir- atory crackles. The patient is clinically euvolaemic, and urine dipstick is normal. CXR demonstrates right lower zone consolidation with the presence of air bronchograms. He is on carbemezepine for epilepsy and risperidone. Blood tests reveal the following:
Hb 13.4
White cell count 12.8 
C reactive protein 23 
Na 123
K 4.7
Urea 6
Creatinine 62

What is the most likely cause of hyponatraemia?

A Pneumonia
B Carbamezepine
C Risperidone
D Syndrome of inappropriate antidiuretic hormone (SIADH)
E Cerebral salt wasting syndrome
A

B

This patient’s hyponatraemia is most likely secondary to Carbamezepine therapy (B), a well documented side effect of this anti-epileptic medication. Carbamezepine stimulates the production of ADH.
It is also one of the ‘terrible 3 Cs’ which cause aplastic anaemia, the other two being carbimazole and chloramphenicol. Any patient with signs of infection or bleeding must be taken very seriously as fulmi- nant sepsis may ensue without prompt treatment. This patient, how- ever, has mounted a white cell response with a normal platelet count therefore making aplastic anaemia unlikely.

Pneumonia (A) does not normally cause a sodium abnormality on its own. Less commonly, Legionnaire’s disease caused by the bacterium Legionella pneumophilia can have extrapulmonary features including hyponatraemia, deranged liver function tests and lymphopenia. This is unlikely to be the case as this organism often colonizes water tanks in places with air conditioning and has a prodromal phase of dry cough with flu-like symptoms. The alternative indirect pulmonary cause of hyponatraemia is lung cancer producing a SIADH; the tumour pre- disposes the patient to pneumonia by obstructing the normal ciliary clearance of the bronchi. It is unlikely in this patient given the lack of smoking history or cachexia.
Risperidone (C) is an atypical antipsychotic and only very rarely causes hyponatraemia. More common side effects include gastrointestinal disturbance and dry mouth. SIADH (D) is the excessive production of anti-diuretic hormone (also called vasopressin) from the posterior pituitary. Its release is stimulated physiologically by osmoreceptors responding to an increased plasma osmolality, as well as baroreceptors responding to decreased intravascular volume. Vasopressin activates vasopressin 2 receptors in the renal collecting duct principal cells, which in turn activate adenylate cyclase to increase intracellular cyclic AMP levels. This is turn increases aquaporin 2 gene transcription and the protein inserts into the apical membrane of the cells allowing free water influx to nor- malize increased plasma osmolality. SIADH occurs when there is excessive production of vasopressin leading to a euvolaemic hyponatraemia. It is a diagnosis of exclusion and requires two criteria in the blood, two criteria in the urine and three exclusion criteria and can be remembered as ‘two low in the blood, two high in the urine, three exclusions everywhere else’.

1 Two low in the blood – hyponatraemia and hypo-osmolality
2 Two high in the urine – high urinary sodium >20mmol/L and high
urinary osmolality
3 Three exclusions – NO renal/adrenal/thyroid/cardiac disease, NO hypovolaemia, NO contributing drugs.
Cerebral salt wasting (CSW) syndrome (E) occurs after head injury or neurosurgical procedures where a natriuretic substance produced in the brain leads to sodium and chloride loss in the kidneys, reducing intra- vascular volume and leading to water retention. There is therefore a baroreceptor-mediated stimulus to vasopressin production. It resembles SIADH in that both are hyponatraemic disorders seen after head injury with high urinary sodium, urinary osmalility and vasopressin levels. The difference is the primary event in CSW is high renal sodium chloride loss, not high vasopressin release.

How well did you know this?
1
Not at all
2
3
4
5
Perfectly
19
Q
  1. Hypercalcaemia

A patient with end stage renal failure presents with depression. He is on haemodialysis three times a week but feels it is not working anymore and is getting more tired lately. He says he has lost his appetite and consequently feels rather constipated too. He feels his mind is deteriorating and there is little worth in attending dialysis anymore. His doctor wants to exclude a reversible cause of his depression and orders some blood tests. The doctor finds the patient has a raised corrected calcium, normal phosphate levels and high parathyroid hormone levels.

What is the diagnosis?

A Primary hyperparathyroidism
B Secondary hyperparathyroidism
C Tertiary hyperparathyroidism
D Pseudohypoparathyroidism
E Pseudopseudohypoparathyroidism
A

C

This patient has tertiary hyperparathyroidism (C) given the presence of elevated calcium levels with high parathyroid levels in the presence of chronic renal failure. Plasma calcium levels are controlled via parathy- roid hormone (PTH) which is produced in the parathyroid glands situ- ated within the thyroid gland. Reduced ionized calcium concentration is detected by the parathyroid glands leading to a release of PTH which circulates in the blood stream. PTH increases calcium resorption from the kidneys whilst increasing phosphate excretion. PTH also stimulates 1-alpha hydroxylation of 25-vitamin D to make 1,25-vitamin D. Finally, PTH increases bone resorption of calcium via osteoclast activation.
The sum effects of increased PTH levels are to increase plasma calcium concentration and to reduce phosphate concentration. PTH has an indi- rect, but very important, mechanism via 1,25-vitamin D which acts to increase gut absorption of calcium.
Tertiary hyperparathyroidism (C) is seen in the setting of chronic renal failure and chronic secondary hyperparathyroidism leads to hyperplastic or adenomatous change in the parathyroid glands resulting in autono- mous PTH secretion. The causes of calcium homeostasis dysregulation are multifactorial including tubular dysfunction leading to calcium leak, inability to excrete phosphate leading to increased PTH levels and parenchymal loss resulting in lower activated vitamin D levels. As a result tertiary hyperparathyroidism gives a raised calcium with a very raised PTH, with normal or low phosphate. Serum alkaline phos- phatase is also raised due to the osteoblast and osteoclast activity (note, osteoblasts produce alkaline phosphatase. This is why there is
a normal alkaline phosphatase in myeloma, as it directly stimulates
the osteoclasts). Treatment of tertiary hyperparathyroidism is subto-
tal parathyroidectomy. Tertiary hyperparathyroidism is differentiated from primary hyperparathyroidism (A) by the presence of chronic renal failure but is otherwise difficult to distinguish biochemically. Primary hyperparathyroidism is most commonly caused by a solitary adenoma in the parathyroid gland. Surgeons sometimes use sestamibi technetium scintigraphy to locate the offending adenoma prior to surgical removal.

Secondary hyperparathyroidism (B) occurs where there is an appro- priately increased PTH level responding to low calcium levels. This is commonly due to chronic renal failure or vitamin D deficiency but can be seen in any pathology resulting in reduced calcium or vitamin D absorption or hyperphosphataemia.
Pseudohypoparathyroidism (also known as Albright’s osteodystrophy) results from a PTH receptor insensitivity in the proximal convoluted tubule of the nephron. As a result, calcium resorption and phosphate excretion fail despite high PTH levels. Furthermore, other physical signs associated with this condition include short height, short 4th and 5th metacarpals, reduced intelligence, basal ganglia calcification, and endocrinopathies including diabetes mellitus, obesity, hypogonadism and hypothyroidism. Type 1 pseudohypoparathyroidism
is inherited in an autosomal dominant manner where the renal ade- nylate cyclase G protein S alpha subunit is deficient, thus halting
the intracellular messaging system activated by PTH. Patients with pseudopseudohypoparathyroidism (E) have similar physical features to pseudohypoparathyroidism but with no biochemical abnormalities of calcium present. This condition is a result of genetic imprinting where the phenotype expressed is dependent on not just what mutation is inherited but also from whom. In other words, inheriting the pseudo- hypoparathyroidism mutation from one’s mother leads to pseudohypo- parathyroidism, but inheriting it from one’s father leads to pseudo- pseudohypoparathyroidism. At the molecular level, this is signalled
by differential methylation of genes thus providing a molecular off switch controlling its expression. Another example of genetic imprint- ing occurs in Prade–Willi syndrome and Angelman’s syndrome, caused by a microdeletion on chromosome 15.
How well did you know this?
1
Not at all
2
3
4
5
Perfectly
20
Q
  1. Biochemical abnormalities of metabolic bone disease
    An 86-year-old woman presents to accident and emergenc1y after a fall. She is a frequent faller but was unable to weight bear after the most recent incident. She has a history of rheumatoid arthritis which is controlled with low dose prednisolone. On examination her right leg is clinically shortened and externally rotated and a pelvic X-ray confirms the presence of a fractured neck of femur. The patient’s hip is fixed the next day. Her day one postoperative bloods show the following:
Corrected calcium normal 
Phosphate normal
Alkaline phosphatase raised 
Parathyroid hormone level normal 
Vitamin D level low

What is the most likely diagnosis?

A Normal
B Osteoporosis
C Paget’s disease
D Osteomalacia
E Malignancy
A

B
Osteoporosis (B) is a common disease which affects women more than men. It is pathologically associated with a reduction in bone density but normal mineralization of bone. There are usually no biochemical abnormalities and therefore all of the parameters measured here should be normal. Given the nature of the fracture, the raised alkaline phos- phatase is likely to be due to the fracture where osteoblast and osteo- clast activation for remodelling and bone healing is required for bone union. Note osteoblasts produce alkaline phosphatase, not osteoclasts. The activation of the two is usually simultaneous, therefore any bone remodelling will lead to a rise in alkaline phosphatase concentration. An important exception is in myeloma where bone lysis occurs with no rise in alkalaline phosphatase because osteoclasts are directly activated without osteoblast activity. Recently the National Institute of Clinical Excellence (NICE) have published guidelines regarding osteoporosis and its management. The risk factors of osteoporosis include:
1 Genetic factors: woman, age, Caucasion/Asian, family history
2 Nutritional factors: excessive alcohol and caffeine, low body weight
3 Life style factors: inactivity, smoking
4 Hormonal factors: nulliparous women, late menarche/early menopause, oophorectomy, post menopausal women, amenorrhoea
5 Iatrogenic factors: thyroxine replacement, steroids
The four risk factors NICE highlight are a low BMI (<22kg/m2), chronic medical conditions including Crohn’s disease, rheumatoid arthritis and ankylosing spondylitis, any condition resulting in prolonged immobil- ity and untreated premature menopause. Investigation of osteoporosis
is with a DEXA scan (dual energy X-ray absorptiometry) which uses X-rays of the spine and pelvis to calculate and compare the bone min- eral density of the subject with a healthy control. This is reported as a
T score; the score is a negative number and represents the number of standard deviations away the subject is from a fit and healthy 30 year old. 0 to −1 is normal, −1 to −2.5 is osteopenic whereas −2.5 or less is osteoporosis. This is different from the Z score which compares the sub- ject’s bone mineral density with an age-related control.
Osteomalacia (D) is caused by vitamin D deficiency which this patient may have a degree of. However, the question asks which is the most likely and given the fragility fracture, her age and previous risk factors, osteoporosis is a much more likely answer. Osteomalacia is the only condition where bone mineralization is reduced: in normal and osteo- porotic patients demineralized bone (osteoid) accounts for about 25 per cent of bone mass whereas in osteomalacia it can approach 100 per cent. Biochemically, there is low calcium, low phosphate, high alkaline phos- phatase and high PTH with low vitamin D levels. The calcium–phosphate product (that is calcium and phosphate concentration multiplied togeth- er) is diagnostically less than 2.4 where the normal value is 3. Subclinical vitamin D deficiency is becoming increasingly common, particularly in children because of reduced sunshine exposure.
Paget’s disease (C) is caused by excessive and abnormal bone remodel- ling with a normal calcium and phosphate but markedly raised alkaline phosphatase, reflecting high osteoblastic activity. There is also increased urinary hydroxyproline which reflects osteoclast activity. Clinical features include pain, bony deformities, sensorineuronal deafness from vestibulocochlear nerve compression through the internal auditory canal or conductive deafness from osteosclerotic changes of the ossicles of the ear. Fractures are a complication of Paget’s because of the disorganized remodelling of bone and patients undergoing surgery in Pagetic bone often need cross-matched blood due to the highly vascular nature of the bone. A serious complication of Paget’s is osteosarcoma which occurs in 1 per cent of patients.
Malignancy usually presents with a raised calcium, phosphate, alkaline phosphatase and reduced PTH level. It is a common cause of hypercal- caemia and must be differentiated from primary hyperparathyroidism, the other common differential. The most common primary sites include breast, kidney, lung, thyroid, colon, ovary and prostate. Prostate can- cer, characteristically, produces osteosclerotic changes with metastases whereas the others have an osteolytic appearance. Prostate cancer often spreads to the vertebrae first due to the venous drainage it shares with the vertebrae.

How well did you know this?
1
Not at all
2
3
4
5
Perfectly
21
Q
  1. Vitamin deficiency tests
    A 59-year-old man presents with a fall and haematemesis after a heavy night drinking at the local pub. This is his third admission in a month with alcohol related problems. He has stopped vomiting, and on examination he is haemodynamically stable. He has digital clubbing, spider naevi and gynaecomastia. He is admitted for neurological observations overnight as he hit his head. The doctors notice the patient suffers from complex ophthalmoplegia, confusion and ataxia. Given his neurological symptoms which test would confirm the associated vitamin deficiency?
A Red cell folate
B Red blood cell transketolase
C Red blood cell glutathione reductase
D Red blood cell aspartate aminotransferase activity
E Carbohydrate deficient transferrin
A

B

This patient suffers from chronic alcohol abuse with signs of chronic liver disease. He also exhibits the classical triad of Wernicke’s encephalopathy caused by a thiamine (vitamin B1) deficiency. The test for this is measuring red blood cell transketolase activity (B). Red cell transke- tolase is a thiamine pyrophosphate requiring enzyme which catalyzes reactions in the pentose phosphate pathway essential for regenerating NADPH in erythrocytes. The test measures enzyme activity by adding thiamine pyrophosphate to a sample of haemolyzed red blood cells and measuring the effluent substances. By calculating the amount of product made and substrates consumed, one is able to calculate the increase of enzyme activity after thiamine addition. A marked increase in activity implies a thiamine deficiency as the other substrate (ribose
5 phosphate) is supplied in excess. Thiamine deficiency has a number of clinical sequelae including Wernicke’s encephalopathy, a reversible neurological manifestation characterized pathologically by haemor- rhage in the mammillary bodies. If left untreated, this may progress to Korsakoff’s syndrome, an irreversible neurological disease character- ized by severe memory loss, confabulation, lack of insight and apathy. Thiamine deficiency can also lead to wet beriberi syndrome leading to a high output cardiac failure.
Folate (vitamin B9) is required for cell reproduction and DNA and
RNA synthesis. It is particularly important in infancy and pregnancy where cell turnover is high and provides the rationale behind folate supplementation of pregnant women up to 12 weeks’ gestation where organogenesis is at its peak. Folic acid is found in high levels in green leafy vegetable, nuts, yeast and liver. Body stores last up to 4 months, therefore deficiency is not common given the fortification of foods.
If one does become deficient, however, features include megaloblastic anaemia, diarrhoea, peripheral neuropathy and glossitis (classically giving a beefy tongue).

Red blood cell glutathione reductase (C) assay tests for riboflavin deficiency. Riboflavin (also known as vitamin B2) is named after its structure – it contains a ribose sugar with a flavin ring moiety which gives it its striking yellow colour. Riboflavin is important in energy metabolism including fats, ketone bodies, proteins and carbohydrates. The assay relies on glutathione reductase (GR), an important enzyme which regenerates glutathione which acts as a buffer against oxidative damage in erythrocytes. GR activity is reliant on riboflavin; GR activ- ity is measured in vitro before and after addition of riboflavin. An increased level of GR activity implies its activity is being limited by riboflavin deficiency. Clinically, riboflavin deficiency causes glossitis, mouth ulceration and dry skin. It is almost always associated with other vitamin deficiencies including iron. Treatment is with vitamin replacement.
Red cell aspartate aminotransferase (AST) activity (D) tests for pyridoxine levels (also known as vitamin B6). This vitamin is important in neurotransmitter synthesis, histamine synthesis, haemoglobin function and amino acid metabolism – this last function is exploited in the laboratory to test for deficiency. The enzyme activity is tested before and after the addition of pyridoxine, in a similar manner to the glutathione reductase and transketolase assays. Pyridoxine is found in meats, whole grain products and vegetables. It is absorbed in the jejunum and ileum and is water soluble. Deficiency causes a seborrhoeic dermatitis-like rash, angular cheilitis and neurological symptoms including confusion and neuropathy. Treatment is with replacement. Importantly, those on isoni- azid for tuberculosis infection should be supplemented with pyridoxine to prevent these symptoms.
Carbohydrate deficient transferrin (E) is used in the detection of alco- hol abuse. Transferrin, normally involved in plasma iron transport, has bound carbohydrate moieties making it a glycoprotein. People who abuse alcohol have a reduction in these bound carbohydrates therefore increasing their carbohydrate deficient transferrin. The test is around 70 per cent sensitive but about 95 per cent specific for alcohol abuse. Other tests for detecting increased alcohol consumption include the presence of a macrocytic anaemia, raised gamma glutamyl transferase as well as alanine aminotransferase and AST.

How well did you know this?
1
Not at all
2
3
4
5
Perfectly
22
Q
  1. Hyperkalaemia
    A 75yo man presents with acute onset abdominal pain. The patient has not passed stools for 3 days and looks unwell. PMH: bowel cancer which was treated with an abdominoperineal resection and chemo 6 years ago. O/E: a large parastomal mass which is tender and irreducible. ABG shows metabolic acidosis with a rasied lactate. The on-call doctor immediately starts normal saline fluids and prepares the patient for theatre. A strangulated hernia is diagnosed by the registrar and an emergency laparotomy is performed to resect the ischaemic bowel. One day postoperatively the patient has the following blood results:
Hb 13.2
WCC 10.9
Platelets 234
Na 145
K 6.3
pH 7.38
Urea and creatinine normal

What is the most likely cause of hyperkalaemia?

A Acute kidney injury
B Tissue injury
C Resolving metabolic acidosis
D Adrenal failure from metastases
E Overhydration from intravenous fluids
A

B

The most likely cause of this patient’s hyperkalaemia is secondary to tissue injury. Potassium is the principle intracellular cation whereas sodium is the principle extracellular cation. Na–K exchange pumps require a continuous supply of adenosine triphosphate (ATP) to supply the energy required to maintain the transcellular gradient. In iscliaemic conditions, where oxygen supply is limited, ATP production fails to meet demand via aerobic respiration alone. Therefore ATP is also generated via anaerobic respiration. This can only occur for a limited period as the anaerobic pathway is both less efficient and produces lactic acid, thereby reducing the local pH and reducing the efficiency of enzymatic activity. This patient has had a significant amount of infarcted bowel removed with a raised lactate implying anaerobic metabolism has both occurred and ultimately failed leading to cell necrosis. The cells are then unable to maintain the Na–K transporter activity leading to potassium release in the blood stream. Furthermore, surgery itself causing direct cell damage increases the intracellular potassium leak into the plasma.

Acute kidney injury (A) is not likely in this patient given the normal creatinine and urea, although this patient is at high risk of pre-renal failure. Bowel obstruction and infarction leads to so-called third space losses of fluids which can be up to litres in magnitude. The third space is within the bowel lumen where resorption of secretions has stopped owing to disrupted transport mechanisms. Acute renal failure would classically give a sharp rise in urea and creatinine and, if serious, leads to a hyperkalaemia with metabolic acidosis. This patient needs intravenous fluids with careful monitoring of input and output as well as monitoring electrolytes. Hyperkalaemia is important as it alters cardiac membrane stability making arrhythmias more likely. The classical electrocardiographic features of hyperkalaemia include tall tented T waves, small P waves, widened QRS complexes, ST depression and QT narrowing. If severe, a sinusoidal pattern emerges at which point the patient needs urgent treatment to prevent a fatal dysrhythmia.

The patient’s metabolic acidosis (C) has resolved and usually the potas- sium abnormality associated with this resolves too. There is a close link between potassium concentration and pH – a lower pH is associ- ated with hyperkalaemia as both K+ and H+ compete with each other for exchange with sodium. Thus an increased pH means increased H+ concentration making it more available for exchange with sodium
thus leaving K+ in the extracellular space. Once the metabolic acidosis resolves, this competition no longer exists and normal potassium homeostasis resumes.

Adrenal failure (D) from metastases could lead to Addison’s disease, a destruction of the zona glomerulosa and fasciculata resulting in lack of aldosterone and cortisol production. Addison’s disease classically presents biochemically with hyponatraemia, hyperkalaemia and hyper- calcaemia. Aldosterone acts in the distal convoluted tube and collecting duct, its intracellular receptor (aldosterone mineralocorticoid receptor) acts with specific hormone response elements on the DNA to regulate gene transcription including N+/K+ pumps. The aldosterone receptor is also activated by cortisol which is produced in much higher concentra- tions physiologically. To prevent over-stimulation of the receptor, how- ever, a deactivating enzyme (11 beta hydroxysteroid dehydrogenase) co-localizes with the receptor to locally inactivate cortisol’s effect. In hypercorisolaemia (Cushing’s) this mechanism is overcome, resulting in excess aldosterone-like effects thus explaining the hypertension in these patients. Interestingly, licorice, which contains glycyrrhizic acid, inhibits this deactivating enzyme explaining its association with hypertension when eaten in excess. Adrenal metastases, although possible in this patient, are unlikely given the biochemical disparity and lack of clinical information about metastatic disease.

Overhydration with intravenous fluids (E) can cause hyperkalaemia, but the on-call doctor in this question prescribed normal saline, which alone contains 154mmol/L sodium chloride only. Given the well known side effect of tissue injury postoperatively, some clinicians routinely omit potassium in the first postoperative bag of fluids to prevent hyperkalaemia. Hartmann’s has a more physiological biochemical profile and contains 5mmol/L potassium as well as 29mmol/L lactate. Patients on this fluid for maintenance fluid therapy can have falsely high lactates when arterial blood samples are analyzed.

How well did you know this?
1
Not at all
2
3
4
5
Perfectly
23
Q
  1. Hypernatraemia
    A 54 year old with a background of hypertension, presents to the GP with a 2 week history of diarrhoea. He has been travelling in South East Asia recently and developed symptoms of diarrhoea 3 weeks ago. He went to the local doctor whilst in China who prescribed tetracycline, but his symptoms have persisted and only improved slightly. His past medical history includes an undisplaced parietal skull fracture he sustained when he was 10. He takes no other medications.

The GP orders blood tests which show the following:

Na 148
K 4.8
Urea 13
Creatinine 112

What is the most likely cause of his hypernatraemia?

A Conn’s syndrome
B Nephrogenic diabetes insipidus
C Cranial diabetes insipidus
D Tetracycline
E Dehydration
A
E
The most likely cause of hypernatraemia in this man is dehydration (E). Gastroenteritis with diarrhoea for 3 weeks causes a high rate of free water loss resulting in increased concentration of sodium in the extra- cellular compartment. Sodium and intravascular volume are closely linked and controlled by the renin angiotensin system and antidiuretic hormone. A reduction in renal blood flow through loss of intravascular volume results in increased renin secretion from the juxtaglomerular apparatus in the kidneys. Renin converts angiotensinogen to angio- tensin I which in turn is converted to angiotensin II by angiotensin converting enzyme (which is constitutively expressed in the lungs). Angiotensin II increases the release of aldosterone from the zona glo- merulosa in the adrenal cortex which acts to increase sodium retention. Retained sodium increases plasma osmolality which stimulates antidiu- retic hormone (ADH) release from the posterior pituitary. ADH acts to increase free water retention, the net result being an increased intravas- cular volume with a normal osmolality.
Diabetes insipidus (DI) is caused by lack of ADH action. Craniogenic DI (C) implies a lack of production of ADH from the posterior pituitary whereas nephrogenic DI (B) implies a lack of sensitivity to ADH.
Craniogenic DI classically follows head injury where over 80 per cent of the descending neurones from the paraventricular and supraoptic nuclei in the hypothalamus need to be destroyed to produce clinical symptoms. It is rare and probably would have manifested earlier with polydipsia and polyuria in this patient given the head injury was at the age of 10.
Nephrogenic DI is a result of renal resistance to ADH and has numer- ous aetiologies. Many intrinsic renal pathologies including intersti- tial nephritis, polycystic kidneys, sarcoid or amyloid can cause this. However, remember nephrogenic DI means a resistance to ADH action despite normal or high levels. This does not necessarily mean there
is an intrinsic kidney problem – any cause of prolonged polyuria can cause solute washout in the renal medulla reducing the action of ADH. Another important cause of nephrogenic DI is drugs. The two classical drugs associated with this are lithium and demeclocycline. The latter is sometimes used therapeutically in patients with the syndrome of inap- propriate ADH (SIADH). Here the excess ADH production is counter- acted by the demeclocycline which inhibits the renal response to ADH. Although demeclocycline is a type of tetracycline, prescribed tetracy- cline (D) (rather confusingly) is a separate drug which is not associ- ated with nephrogenic DI. Thus the treatment this man has received is unlikely to have caused the hypernatraemia.
Conn’s syndrome (A) is caused by an aldosterone secreting tumour leading to a hypertensive, hypokalaemic, metabolic alkalosis. It very rarely causes hypernatraemia. The causes of this disease include Adrenal adenoma, Bilateral nodular hyperplasia, Carcinoma of the adrenals or a Defective gene (glucocorticoid remediable aldosteronism, GRA). Adrenal ademona is by far the most common and presents with resistant hyper- tension and weakness (due to hypokalaemia). GRA is caused by a chimeric gene of aldosterone synthase with the 11 beta hydroxylase-1 promoter, resulting in an ACTH sensitive secretion of aldosterone. ACTH is under the negative feedback control of glucocorticoids. Exogenous administration of dexamethasone reduces ACTH levels thus reducing aldosterone expression, treating the disease!
How well did you know this?
1
Not at all
2
3
4
5
Perfectly
24
Q
  1. Water deprivation test
    A 42-year-old woman with persistent polyuria and polydipsia is admitted for a water deprivation test. At the beginning of the test her weight, urine volume and osmolality and serum osmolality are measured and hourly thereafter for 8 hours. After 8 hours, she is given intramuscular desmopressin but drinks 3L of water before going to bed. Her blood is taken again the next morning (16 hours after beginning the test) and the results are as follows:

What is the most likely diagnosis?

A Nephrogenic diabetes insipidus
B Craniogenic diabetes insipidus
C Psychogenic polydipsia
D Invalid test
E Normal
A

C
This patient is most likely suffering from psychogenic polydipsia, an uncommon condition where excessive water drinking occurs without the physiological stimulus to drink. It was classically described in patients with schizophrenia but also occurs in children. Chronic psychogenic polydipsia can result in mineral washout of the renal interstitium result- ing in a physiological inability to concentrate urine, in other words a form of nephrogenic diabetes insipidus.

The water deprivation test is a seldom used test nowadays but is useful to understand when considering these clinical problems. The test begins with the patient being completely deprived of water for 8 hours in which time the patient’s weight, blood and urine osmolality and urine volume are measured. A weight loss of more than 5 per cent in adults is an indication to stop the test. After 8 hours, 2μg of desmopressin (a synthetic analogue of vasopressin) is given. The same measurements are taken for the next 8 hours. After the desmopressin is given the patient is allowed to drink up to 1.5 times the total urine output for the first
8 hours. In this patient’s case she had produced 2200 mL of urine, but drank 3000 mL of water. This therefore is acceptable and did not nullify the test making (D) an incorrect answer.
The patient’s urine osmolality increased above 800 mOsmol/kg after
8 hours of water deprivation, indicating vasopressin action is functioning to appropriately retain water therefore concentrating the urine. A further 8 hours later, despite drinking 3 L of fluid, the patient’s urine is still very concentrated implying the administered desmopressin and endogenous vasopressin are functioning. In patients with craniogenic DI (B), the administration of desmopressin provides the water retention signal that the kidneys are failing to concentrate the urine. The typical result for patients with craniogenic DI is a dilute urine (<300 mOsmol/kg)
after the first 8 hours, but concentrated urine after the desmopressin administration (>800 mOsmol/kg). Nephrogenic DI (A) would not respond to desmopressin and would likely leave the patient with dilute urine (<300mOsmol/kg) before and after desmopressin administration. Finally, suggesting this patient is normal given the symptomatic polyuria and polydispia is unlikely. Any patient presenting with these symptoms must be investigated with a blood glucose measurement to explore the possibility of diabetes mellitus.

How well did you know this?
1
Not at all
2
3
4
5
Perfectly
25
Q
  1. Acute abdominal pain
    A 24-year-old previously fit and well woman presents with sudden onset abdominal pain the night after a party where she drank five units of alcohol. She complains of central abdominal pain, with nausea and vomiting. She also finds it difficult to control her bladder. On examination, she is tachycardic, hypertensive and is beginning to become confused. On looking back at her previous admissions, the doctor notices she has had similar episodes after drinking. This was also true for when she started the oral contraceptive pill and when she had tuberculosis which was treated with standard antibiotic treatments. She is also seeing a neurologist for peripheral neuropathy of unknown cause. The admitting doctor, an Imperial college graduate, suggests the possibility of acute intermittent porphyria.

What enzyme deficiency is responsible for this disease?

A Porphobilinogen deaminase
B Uroporphyrinogen synthase
C Coproporphyrinogen oxidase
D Protoporphyrinogen oxidase
E Uroporphyrinogen decarboxylase
A

A
PBG deaminase deficiency (A) causes acute intermittent porphyria, which this patient suffers from. The porphyrias are a group of seven disorders caused by enzyme activity reduction in the haem biosynthetic pathway. Haem is manufactured in both the liver and bone marrow where branched chain amino acids together with succinyl CoA and glycine are needed. The first step involves 5 aminolevulinic acid (ALA) synthesis by ALA synthase. This is the rate limiting step which is under negative feedback from haem itself.
A simplified schema of haem production is provided with the products depicted on the left, and the enzyme responsible along with the type of porphyria caused if it was deficient on the right.

The features of porphyria can be generally classified into neurological, cutaneous and microcytic anaemia. The exact combination of symp- toms depends on where in the haem pathway the deficiency occurs. Neurological symptoms, including peripheral neuropathy, autonomic neuropathy and psychiatric features, are caused by the increase of por- phyrin precursors 5 ALA and prophobilinogen (PBG). Cutaneous symp- toms are due to photosensitive porphyrins which are produced later on in the sequence. Finally microcytic anaemia occurs due to the deficiency of haem production.
Acute intermittent porphyria (AIP) presents without cutaneous symp- toms, this is because the enzyme deficiency is further upstream from the photosensitive porphyrins which cause the cutaneous symptoms. Instead neurological symptoms of the peripheral, autonomic and psy- chiatric systems predominate, as in this patient. The symptoms cluster in attacks if toxins induce ALA synthase or PBG deaminase activity. These include alcohol, the oral contraceptive pill and certain anti- biotics including rifampicin and pyrazinamide (two commonly used anti-tuberculosis drugs). Other common precipitants include surgery, infection and starvation. Investigations classically show urine which becomes brown or black upon standing in light as well as reduced erythrocyte PBG deaminase levels. Note there is no increase of faecal porphyrins in AIP. Treatment is to avoid precipitants as well as dex- trose infusion and haem arginate intravenously which both inhibit ALA synthase activity.

Uroporphyrinogen synthase (B) results in congenital erythropoeitic porphyria which is one of the rarest inborn errors of metabolism. It
is caused by a mutation on chromosome 10q26 and is inherited in an autosomal recessive fashion. Symptoms include vesicles, bullae and excessive lanugo hair as well as mutilating deformities of the limbs
and face. Urine is classically burgundy red as well as patients having erythrodontia – red stained teeth. Treatment is to avoid sunlight and symptomatically treat the anaemia. Coproporphyrinogen oxidase (C) causes hereditary coproporphyria and is another rare type of porphyria. The symptoms are predominantly neuro-visceral. Diagnosis is confirmed with increased faecal and urinary coproporphyrinogen.
Protoporphyrinogen oxidase deficiency (D) causes variegate porphyria which is caused by an autosomal dominant mutation of chromosome 14. It is relatively rare in the world except in South Africa where its incidence is as high as one in 300 (most probably due to the founder effect from early settlers). Attacks feature neuro-cutaneous features, although not necessarily together at the same time. It is almost always precipitated by drugs making it difficult to distinguish from AIP. In var- iegate porphyria, however, there is increased faecal protoporphyria as well as positive plasma fluorescence scanning.
Uroporphyrinogen decarboxylase (E) causes porphyria cutanea tarda and can be inherited in an autosomal dominant manner. It is characterized by cutaneous features including bullous reactions to light, hyperpig- mentation, as well as liver disease. Non-inherited causes include alco- hol, iron, infections (hepatitis C and HIV) and systemic lupus erythema- tosus (SLE). Investigations reveal abnormal liver function tests, raised ferritin (always) and increased urinary uroporphyrinogen. This gives a characteristic pink red fluorescence when illuminated with a Wood’s lamp. Treatment is to avoid precipitants as well as chloroquine which complexes with porphyrins and promotes uroporphyrin release from the liver.
How well did you know this?
1
Not at all
2
3
4
5
Perfectly
26
Q

Acid–base balance

A Metabolic acidosis
B Metabolic acidosis with respiratory compensation
C Metabolic alkalosis
D Metabolic alkalosis with respiratory compensation
E Respiratory acidosis
F Respiratory acidosis with metabolic compensation
G Respiratory alkalosis
H Respiratory alkalosis with metabolic compensation
I Mixed metabolic and respiratory acidosis

1)

pH 7.31 (7.35–7.45)
pO2 7.6 (10.6–13kPa)
pCO2 8.2 (4.7–6.0kPa)
HCO3 26 (22–28mmol/L)

A

1) E

Respiratory acidosis (E) is defined by low pH (acidosis) together with a high pCO2, due to CO2 retention secondary to a pulmonary, neuromuscular or physical causes. There is no metabolic compensation in this case, suggesting this is an acute pathology; a compensatory metabolic rise in HCO3 from the kidneys can take hours or days. This patient is also hypoxic with a low pO2. Causes of an acute respiratory acidosis include an acute exacerbation of asthma, foreign body obstruction & cardiac arrest.

Metabolic acidosis (A) occurs when pH is reduced due to low HCO3. If there is no respiratory compensation, pCO2 will be normal or elevated.

Metabolic alkalosis (C) occurs when pH is increased as a result of raised HCO3. If there is no respiratory compensation, pCO2 will be normal or low.

Respiratory acidosis with metabolic compensation (F) is defined as a low pH as a consequence of high pCO2. There is a raised HCO3 concentration in order to raise pH back towards normal.

Respiratory alkalosis with metabolic compensation (H) is defined as a high pH due to low pCO2. There is a reduced HCO3 concentration in order to lower pH back towards normal.

How well did you know this?
1
Not at all
2
3
4
5
Perfectly
27
Q

Acid–base balance

A Metabolic acidosis
B Metabolic acidosis with respiratory compensation
C Metabolic alkalosis
D Metabolic alkalosis with respiratory compensation
E Respiratory acidosis
F Respiratory acidosis with metabolic compensation
G Respiratory alkalosis
H Respiratory alkalosis with metabolic compensation
I Mixed metabolic and respiratory acidosis

2)

pH 7.36 (7.35–7.45)
pO2 14.2 (10.6–13kPa)
pCO2 4.1 (4.7–6.0kPa)
HCO3 14 (22–28mmol/L)

A

2) B

Metabolic acidosis with respiratory compensation (B) occurs when pH is low (acidosis) and HCO3 is low with concurrent respiratory compensation by decreasing pCO2. The anion gap can differentiate between causes of metabolic acidosis (anion gap = [Na++ K+] – [Cl−+ HCO3−]; normal range between 10 and 18mmol/L). Causes of a raised anion gap can be remembered by the mnemonic MUDPILES: Methanol/metformin, Uraemia, Diabetic ketoacidosis, Paraldehyde, Iron, Lactate, Ethanol and Salicylates. Causes of a normal anion gap include diarrhoea, Addison’s disease and renal tubular acidosis.

Metabolic acidosis (A) occurs when pH is reduced due to low HCO3. If there is no respiratory compensation, pCO2 will be normal or elevated.

Metabolic alkalosis (C) occurs when pH is increased as a result of raised HCO3. If there is no respiratory compensation, pCO2 will be normal or low.

Respiratory acidosis with metabolic compensation (F) is defined as a low pH as a consequence of high pCO2. There is a raised HCO3 concentration in order to raise pH back towards normal.

Respiratory alkalosis with metabolic compensation (H) is defined as a high pH due to low pCO2. There is a reduced HCO3 concentration in order to lower pH back towards normal.

How well did you know this?
1
Not at all
2
3
4
5
Perfectly
28
Q

Acid–base balance

A Metabolic acidosis
B Metabolic acidosis with respiratory compensation
C Metabolic alkalosis
D Metabolic alkalosis with respiratory compensation
E Respiratory acidosis
F Respiratory acidosis with metabolic compensation
G Respiratory alkalosis
H Respiratory alkalosis with metabolic compensation
I Mixed metabolic and respiratory acidosis

3)

pH 7.45 (7.35–7.45)
pO2 10.2 (10.6–13kPa)
pCO2 7.2 (4.7–6.0kPa)
HCO3 32 (22–28mmol/L)

A

3) D

Metabolic alkalosis with respiratory compensation (D) occurs when pH is high (alkalosis) and HCO3 is high with a compensatory reduction in respiratory effort that increases pCO2. As respiratory effort is reduced there is the possibility of the patient becoming hypoxic. Causes of metabolic alkalosis include vomiting, potassium depletion secondary to diuretic use, burns and sodium bicarbonate ingestion. Respiratory compensation increase serum CO2 concentration, which reduces pH back towards normal.

Metabolic acidosis (A) occurs when pH is reduced due to low HCO3. If there is no respiratory compensation, pCO2 will be normal or elevated.

Metabolic alkalosis (C) occurs when pH is increased as a result of raised HCO3. If there is no respiratory compensation, pCO2 will be normal or low.

Respiratory acidosis with metabolic compensation (F) is defined as a low pH as a consequence of high pCO2. There is a raised HCO3 concentration in order to raise pH back towards normal.

Respiratory alkalosis with metabolic compensation (H) is defined as a high pH due to low pCO2. There is a reduced HCO3 concentration in order to lower pH back towards normal.

How well did you know this?
1
Not at all
2
3
4
5
Perfectly
29
Q

Acid–base balance

A Metabolic acidosis
B Metabolic acidosis with respiratory compensation
C Metabolic alkalosis
D Metabolic alkalosis with respiratory compensation
E Respiratory acidosis
F Respiratory acidosis with metabolic compensation
G Respiratory alkalosis
H Respiratory alkalosis with metabolic compensation
I Mixed metabolic and respiratory acidosis

4)

pH 7.30 (7.35–7.45)
pO2 8.2 (10.6–13kPa)
pCO2 7.2 (4.7–6.0kPa)
HCO3 19 (22–28mmol/L)

A

4) I

Mixed metabolic and respiratory acidosis (I) occurs when there is a low pH and a simultaneous high pCO2 and low HCO3. In the case of a mixed metabolic and respiratory acidosis, the metabolic acidosis component may be due to conditions such as uraemia, ketones produced as a result of diabetes mellitus or renal tubular acidosis. The respiratory acidosis component may be due to any cause of respiratory failure. Hence, this mixed picture may occur in a COPD patient with concurrent diabetes mellitus.

Metabolic acidosis (A) occurs when pH is reduced due to low HCO3. If there is no respiratory compensation, pCO2 will be normal or elevated.

Metabolic alkalosis (C) occurs when pH is increased as a result of raised HCO3. If there is no respiratory compensation, pCO2 will be normal or low.

Respiratory acidosis with metabolic compensation (F) is defined as a low pH as a consequence of high pCO2. There is a raised HCO3 concentration in order to raise pH back towards normal.

Respiratory alkalosis with metabolic compensation (H) is defined as a high pH due to low pCO2. There is a reduced HCO3 concentration in order to lower pH back towards normal.

How well did you know this?
1
Not at all
2
3
4
5
Perfectly
30
Q

Acid–base balance

A Metabolic acidosis
B Metabolic acidosis with respiratory compensation
C Metabolic alkalosis
D Metabolic alkalosis with respiratory compensation
E Respiratory acidosis
F Respiratory acidosis with metabolic compensation
G Respiratory alkalosis
H Respiratory alkalosis with metabolic compensation
I Mixed metabolic and respiratory acidosis

5)

pH 7.49 (7.35–7.45)
pO2 13.6 (10.6–13kPa)
pCO2 4.1 (4.7–6.0kPa)
HCO3 23 (22–28mmol/L)

A

5) G

Respiratory alkalosis (G) is biochemically defined by a raised pH (alkalosis) and reduced pCO2. As previously mentioned, metabolic compensation can take hours or days to occur. The primary pathology causing respiratory alkalosis is hyperventilation which causes increased CO2 to be lost via the lungs. Causes of hyperventilation may be due to central nervous system disease (eg. stroke). Other causes of hyperventilation include anxiety (panic attack), pulmonary embolism and drugs (salicylates).

Metabolic acidosis (A) occurs when pH is reduced due to low HCO3. If there is no respiratory compensation, pCO2 will be normal or elevated.

Metabolic alkalosis (C) occurs when pH is increased as a result of raised HCO3. If there is no respiratory compensation, pCO2 will be normal or low.

Respiratory acidosis with metabolic compensation (F) is defined as a low pH as a consequence of high pCO2. There is a raised HCO3 concentration in order to raise pH back towards normal.

Respiratory alkalosis with metabolic compensation (H) is defined as a high pH due to low pCO2. There is a reduced HCO3 concentration in order to lower pH back towards normal.

How well did you know this?
1
Not at all
2
3
4
5
Perfectly
31
Q

Calcium handling

A Primary hyperparathyroidism
B Secondary hyperparathyroidism
C Tertiary hyperparathyroidism
D Pseudohypoparathyroidism
E Primary hypoparathyroidism
F Osteoporosis
G Osteomalacia
H Paget’s disease
I Familial benign hypercalcaemia

1)

Ca 2.4 (2.2–2.6 mmol/L)
PTH 4.2 (0.8–8.5pmol/L)
ALP 250 (30–150u/L)
PO4 1.1 (0.8–1.2mmol/L) 
Vitamin D 76 (60–105nmol/L)
A

1)H 2)A 3)G 4)I 5)E

Paget’s disease (H) is associated with impaired bone remodelling. New bone is larger but weak and prone to fracture. The pathogenesis has been postulated to be linked to paramyxovirus. All calcium blood studies will be normal apart from ALP, which will be raised. Paget’s disease is associated with extreme bone pain, bowing and chalk-stick fractures. Bossing of the skull may lead to CN VIII palsy and hence hearing loss. X-ray findings = lytic and sclerotic lesions.

Primary hyperparathyroidism (A) is caused by a parathyroid adenoma or parathyroid chief cell hyperplasia that leads to increased PTH production. Primary hyperparathyroidism leads to hypercalcaemia due to
a raised PTH level. PTH achieves this by activating osteoclastic bone resorption (increasing blood ALP), stimulating calcium reabsorption in the kidney (with concurrent excretion of phosphate) and potentiating the action of the enzyme 1-alpha-hydroxylase in the kidney, which acts on 25-hydroxyvitamin D3 to produce 1,25-dihydroxyvitamin D3 (calcitriol), which increases gut absorption of calcium.

Osteomalacia (G; rickets in children) results from insufficient bone mineralization, secondary to vitamin D or phosphate deficiency. Low vitamin D causes hypocalcaemia, due to reduced 1,25-dihydoxyvitamin D3 production, and hence reduced reabsorption of calcium from the gut. Low blood calcium levels cause an increase in production of PTH in an attempt to normalize calcium. Therefore, calcium levels will either be low or inappropriately normal. Increased bone resorption will cause ALP levels to rise.

Familial benign hypercalcaemia (I) is a genetic condition leading to raised blood calcium levels. The disease results from a mutation in the calcium receptor located on the parathyroid glands and kidneys. This receptor defect therefore leads to underestimation of calcium, causing an increased production of PTH, despite the raised calcium levels. It is important to distinguish these patients from hyperparathyroid patients as the management of these conditions differs. Receptor failure in the kidneys reduces calcium excretion, leading to a hypocalcuric state.

Primary hypoparathyroidism (E) is defined as dysfunction of the para- thyroid glands leading to reduced production of PTH. As a result, the actions of PTH are blunted leading to reduced bone resorption as well as renal and gut calcium reabsorption. As a consequence there is hypocalcaemia and hyperphosphataemia. Other causes of hypocalcaemia include pseudoparathyroidism, vitamin D deficiency, renal disease (unable to make 1,25-dihydroxyvitamin D3), magnesium deficiency (magnesium required for PTH rise) and post-surgical (neck surgery may damage parathyroid glands).

Secondary hyperparathyroidism (B) is defined as the release of PTH as a consequence of hypocalcaemia that arises due to non-parathyroid pathology. The most common cause is chronic renal failure.

Tertiary hyperparathyroidism (C) results from hyperplasia of the parathyroid glands after a long period of secondary hyperparathyroidism. Autonomous production of PTH causes hypercalcaemia.

Pseudohypoparathyroidism (D) is a genetic condition in which there is resistance to PTH. As a result patients have high PTH and phosphate levels but are hypocalcaemic.

Osteoporosis (F) results in reduced bone density and all calcium studies are normal. RF = Menopause, alcohol, drugs (e.g goserelin) and steroids.

32
Q

Calcium handling

A Primary hyperparathyroidism
B Secondary hyperparathyroidism
C Tertiary hyperparathyroidism
D Pseudohypoparathyroidism
E Primary hypoparathyroidism
F Osteoporosis
G Osteomalacia
H Paget’s disease
I Familial benign hypercalcaemia

2)

Ca 3.1 (2.2–2.6 mmol/L)
PTH 10.5 (0.8–8.5pmol/L)
ALP 165 (30–150u/L)
PO4 0.6 (0.8–1.2mmol/L) 
Vitamin D 78 (60–105nmol/L)
A

1)H 2)A 3)G 4)I 5)E

Paget’s disease (H) is associated with impaired bone remodelling. New bone is larger but weak and prone to fracture. The pathogenesis has been postulated to be linked to paramyxovirus. All calcium blood studies will be normal apart from ALP, which will be raised. Paget’s disease is associated with extreme bone pain, bowing and chalk-stick fractures. Bossing of the skull may lead to CN VIII palsy and hence hearing loss. X-ray findings = lytic and sclerotic lesions.

Primary hyperparathyroidism (A) is caused by a parathyroid adenoma or parathyroid chief cell hyperplasia that leads to increased PTH production. Primary hyperparathyroidism leads to hypercalcaemia due to a raised PTH level. PTH achieves this by activating osteoclastic bone resorption (increasing blood ALP), stimulating calcium reabsorption in the kidney (with concurrent excretion of phosphate) and potentiating the action of the enzyme 1-alpha-hydroxylase in the kidney, which acts on 25-hydroxyvitamin D3 to produce 1,25-dihydroxyvitamin D3 (calcitriol), which increases gut absorption of calcium.

Osteomalacia (G; rickets in children) results from insufficient bone mineralization, secondary to vitamin D or phosphate deficiency. Low vitamin D causes hypocalcaemia, due to reduced 1,25-dihydoxyvitamin D3 production, and hence reduced reabsorption of calcium from the gut. Low blood calcium levels cause an increase in production of PTH in an attempt to normalize calcium. Therefore, calcium levels will either be low or inappropriately normal. Increased bone resorption will cause ALP levels to rise.

Familial benign hypercalcaemia (I) is a genetic condition leading to raised blood calcium levels. The disease results from a mutation in the calcium receptor located on the parathyroid glands and kidneys. This receptor defect therefore leads to underestimation of calcium, causing an increased production of PTH, despite the raised calcium levels. It is important to distinguish these patients from hyperparathyroid patients as the management of these conditions differs. Receptor failure in the kidneys reduces calcium excretion, leading to a hypocalcuric state.

Primary hypoparathyroidism (E) is defined as dysfunction of the para- thyroid glands leading to reduced production of PTH. As a result, the actions of PTH are blunted leading to reduced bone resorption as well as renal and gut calcium reabsorption. As a consequence there is hypocalcaemia and hyperphosphataemia. Other causes of hypocalcaemia include pseudoparathyroidism, vitamin D deficiency, renal disease (unable to make 1,25-dihydroxyvitamin D3), magnesium deficiency (magnesium required for PTH rise) and post-surgical (neck surgery may damage parathyroid glands).

Secondary hyperparathyroidism (B) is defined as the release of PTH as a consequence of hypocalcaemia that arises due to non-parathyroid pathology. The most common cause is chronic renal failure.

Tertiary hyperparathyroidism (C) results from hyperplasia of the parathyroid glands after a long period of secondary hyperparathyroidism. Autonomous production of PTH causes hypercalcaemia.

Pseudohypoparathyroidism (D) is a genetic condition in which there is resistance to PTH. As a result patients have high PTH and phosphate levels but are hypocalcaemic.

Osteoporosis (F) results in reduced bone density and all calcium studies are normal. RF = Menopause, alcohol, drugs (e.g goserelin) and steroids.

33
Q

Calcium handling

A Primary hyperparathyroidism
B Secondary hyperparathyroidism
C Tertiary hyperparathyroidism
D Pseudohypoparathyroidism
E Primary hypoparathyroidism
F Osteoporosis
G Osteomalacia
H Paget’s disease
I Familial benign hypercalcaemia

3)

Ca 2.1 (2.2–2.6 mmol/L)
PTH 10.4 (0.8–8.5pmol/L)
ALP 190 (30–150u/L)
PO4 0.69 (0.8–1.2mmol/L) 
Vitamin D 41 (60–105nmol/L)
A

1)H 2)A 3)G 4)I 5)E

Paget’s disease (H) is associated with impaired bone remodelling. New bone is larger but weak and prone to fracture. The pathogenesis has been postulated to be linked to paramyxovirus. All calcium blood studies will be normal apart from ALP, which will be raised. Paget’s disease is associated with extreme bone pain, bowing and chalk-stick fractures. Bossing of the skull may lead to CN VIII palsy and hence hearing loss. X-ray findings = lytic and sclerotic lesions.

Primary hyperparathyroidism (A) is caused by a parathyroid adenoma or parathyroid chief cell hyperplasia that leads to increased PTH production. Primary hyperparathyroidism leads to hypercalcaemia due to a raised PTH level. PTH achieves this by activating osteoclastic bone resorption (increasing blood ALP), stimulating calcium reabsorption in the kidney (with concurrent excretion of phosphate) and potentiating the action of the enzyme 1-alpha-hydroxylase in the kidney, which acts on 25-hydroxyvitamin D3 to produce 1,25-dihydroxyvitamin D3 (calcitriol), which increases gut absorption of calcium.

Osteomalacia (G; rickets in children) results from insufficient bone mineralization, secondary to vitamin D or phosphate deficiency. Low vitamin D causes hypocalcaemia, due to reduced 1,25-dihydoxyvitamin D3 production, and hence reduced reabsorption of calcium from the gut. Low blood calcium levels cause an increase in production of PTH in an attempt to normalize calcium. Therefore, calcium levels will either be low or inappropriately normal. Increased bone resorption will cause ALP levels to rise.

Familial benign hypercalcaemia (I) is a genetic condition leading to raised blood calcium levels. The disease results from a mutation in the calcium receptor located on the parathyroid glands and kidneys. This receptor defect therefore leads to underestimation of calcium, causing an increased production of PTH, despite the raised calcium levels. It is important to distinguish these patients from hyperparathyroid patients as the management of these conditions differs. Receptor failure in the kidneys reduces calcium excretion, leading to a hypocalcuric state.

Primary hypoparathyroidism (E) is defined as dysfunction of the para- thyroid glands leading to reduced production of PTH. As a result, the actions of PTH are blunted leading to reduced bone resorption as well as renal and gut calcium reabsorption. As a consequence there is hypocalcaemia and hyperphosphataemia. Other causes of hypocalcaemia include pseudoparathyroidism, vitamin D deficiency, renal disease (unable to make 1,25-dihydroxyvitamin D3), magnesium deficiency (magnesium required for PTH rise) and post-surgical (neck surgery may damage parathyroid glands).

Secondary hyperparathyroidism (B) is defined as the release of PTH as a consequence of hypocalcaemia that arises due to non-parathyroid pathology. The most common cause is chronic renal failure.

Tertiary hyperparathyroidism (C) results from hyperplasia of the parathyroid glands after a long period of secondary hyperparathyroidism. Autonomous production of PTH causes hypercalcaemia.

Pseudohypoparathyroidism (D) is a genetic condition in which there is resistance to PTH. As a result patients have high PTH and phosphate levels but are hypocalcaemic.

Osteoporosis (F) results in reduced bone density and all calcium studies are normal. RF = Menopause, alcohol, drugs (e.g goserelin) and steroids.

34
Q

Calcium handling

A Primary hyperparathyroidism
B Secondary hyperparathyroidism
C Tertiary hyperparathyroidism
D Pseudohypoparathyroidism
E Primary hypoparathyroidism
F Osteoporosis
G Osteomalacia
H Paget’s disease
I Familial benign hypercalcaemia

4)

Ca 1.8 (2.2–2.6 mmol/L)
PTH 9.6 (0.8–8.5pmol/L)
ALP 50 (30–150u/L)
PO4 1.9 (0.8–1.2mmol/L) 
Vitamin D 82 (60–105nmol/L)
A

1)H 2)A 3)G 4)I 5)E

Paget’s disease (H) is associated with impaired bone remodelling. New bone is larger but weak and prone to fracture. The pathogenesis has been postulated to be linked to paramyxovirus. All calcium blood studies will be normal apart from ALP, which will be raised. Paget’s disease is associated with extreme bone pain, bowing and chalk-stick fractures. Bossing of the skull may lead to CN VIII palsy and hence hearing loss. X-ray findings = lytic and sclerotic lesions.

Primary hyperparathyroidism (A) is caused by a parathyroid adenoma or parathyroid chief cell hyperplasia that leads to increased PTH production. Primary hyperparathyroidism leads to hypercalcaemia due to a raised PTH level. PTH achieves this by activating osteoclastic bone resorption (increasing blood ALP), stimulating calcium reabsorption in the kidney (with concurrent excretion of phosphate) and potentiating the action of the enzyme 1-alpha-hydroxylase in the kidney, which acts on 25-hydroxyvitamin D3 to produce 1,25-dihydroxyvitamin D3 (calcitriol), which increases gut absorption of calcium.

Osteomalacia (G; rickets in children) results from insufficient bone mineralization, secondary to vitamin D or phosphate deficiency. Low vitamin D causes hypocalcaemia, due to reduced 1,25-dihydoxyvitamin D3 production, and hence reduced reabsorption of calcium from the gut. Low blood calcium levels cause an increase in production of PTH in an attempt to normalize calcium. Therefore, calcium levels will either be low or inappropriately normal. Increased bone resorption will cause ALP levels to rise.

Familial benign hypercalcaemia (I) is a genetic condition leading to raised blood calcium levels. The disease results from a mutation in the calcium receptor located on the parathyroid glands and kidneys. This receptor defect therefore leads to underestimation of calcium, causing an increased production of PTH, despite the raised calcium levels. It is important to distinguish these patients from hyperparathyroid patients as the management of these conditions differs. Receptor failure in the kidneys reduces calcium excretion, leading to a hypocalcuric state.

Primary hypoparathyroidism (E) is defined as dysfunction of the para- thyroid glands leading to reduced production of PTH. As a result, the actions of PTH are blunted leading to reduced bone resorption as well as renal and gut calcium reabsorption. As a consequence there is hypocalcaemia and hyperphosphataemia. Other causes of hypocalcaemia include pseudoparathyroidism, vitamin D deficiency, renal disease (unable to make 1,25-dihydroxyvitamin D3), magnesium deficiency (magnesium required for PTH rise) and post-surgical (neck surgery may damage parathyroid glands).

Secondary hyperparathyroidism (B) is defined as the release of PTH as a consequence of hypocalcaemia that arises due to non-parathyroid pathology. The most common cause is chronic renal failure.

Tertiary hyperparathyroidism (C) results from hyperplasia of the parathyroid glands after a long period of secondary hyperparathyroidism. Autonomous production of PTH causes hypercalcaemia.

Pseudohypoparathyroidism (D) is a genetic condition in which there is resistance to PTH. As a result patients have high PTH and phosphate levels but are hypocalcaemic.

Osteoporosis (F) results in reduced bone density and all calcium studies are normal. RF = Menopause, alcohol, drugs (e.g goserelin) and steroids.

35
Q

Calcium handling

A Primary hyperparathyroidism
B Secondary hyperparathyroidism
C Tertiary hyperparathyroidism
D Pseudohypoparathyroidism
E Primary hypoparathyroidism
F Osteoporosis
G Osteomalacia
H Paget’s disease
I Familial benign hypercalcaemia

5)

Ca 1.8 (2.2–2.6 mmol/L)
PTH 0.69 (0.8–8.5pmol/L)
ALP 89 (30–150u/L)
PO4 1.5 (0.8–1.2mmol/L) 
Vitamin D 76 (60–105nmol/L)
A

1)H 2)A 3)G 4)I 5)E

Paget’s disease (H) is associated with impaired bone remodelling. New bone is larger but weak and prone to fracture. The pathogenesis has been postulated to be linked to paramyxovirus. All calcium blood studies will be normal apart from ALP, which will be raised. Paget’s disease is associated with extreme bone pain, bowing and chalk-stick fractures. Bossing of the skull may lead to CN VIII palsy and hence hearing loss. X-ray findings = lytic and sclerotic lesions.

Primary hyperparathyroidism (A) is caused by a parathyroid adenoma or parathyroid chief cell hyperplasia that leads to increased PTH production. Primary hyperparathyroidism leads to hypercalcaemia due to a raised PTH level. PTH achieves this by activating osteoclastic bone resorption (increasing blood ALP), stimulating calcium reabsorption in the kidney (with concurrent excretion of phosphate) and potentiating the action of the enzyme 1-alpha-hydroxylase in the kidney, which acts on 25-hydroxyvitamin D3 to produce 1,25-dihydroxyvitamin D3 (calcitriol), which increases gut absorption of calcium.

Osteomalacia (G; rickets in children) results from insufficient bone mineralization, secondary to vitamin D or phosphate deficiency. Low vitamin D causes hypocalcaemia, due to reduced 1,25-dihydoxyvitamin D3 production, and hence reduced reabsorption of calcium from the gut. Low blood calcium levels cause an increase in production of PTH in an attempt to normalize calcium. Therefore, calcium levels will either be low or inappropriately normal. Increased bone resorption will cause ALP levels to rise.

Familial benign hypercalcaemia (I) is a genetic condition leading to raised blood calcium levels. The disease results from a mutation in the calcium receptor located on the parathyroid glands and kidneys. This receptor defect therefore leads to underestimation of calcium, causing an increased production of PTH, despite the raised calcium levels. It is important to distinguish these patients from hyperparathyroid patients as the management of these conditions differs. Receptor failure in the kidneys reduces calcium excretion, leading to a hypocalcuric state.

Primary hypoparathyroidism (E) is defined as dysfunction of the para- thyroid glands leading to reduced production of PTH. As a result, the actions of PTH are blunted leading to reduced bone resorption as well as renal and gut calcium reabsorption. As a consequence there is hypocalcaemia and hyperphosphataemia. Other causes of hypocalcaemia include pseudoparathyroidism, vitamin D deficiency, renal disease (unable to make 1,25-dihydroxyvitamin D3), magnesium deficiency (magnesium required for PTH rise) and post-surgical (neck surgery may damage parathyroid glands).

Secondary hyperparathyroidism (B) is defined as the release of PTH as a consequence of hypocalcaemia that arises due to non-parathyroid pathology. The most common cause is chronic renal failure.

Tertiary hyperparathyroidism (C) results from hyperplasia of the parathyroid glands after a long period of secondary hyperparathyroidism. Autonomous production of PTH causes hypercalcaemia.

Pseudohypoparathyroidism (D) is a genetic condition in which there is resistance to PTH. As a result patients have high PTH and phosphate levels but are hypocalcaemic.

Osteoporosis (F) results in reduced bone density and all calcium studies are normal. RF = Menopause, alcohol, drugs (e.g goserelin) and steroids.

36
Q

Endocrine chemical pathology

A Prolactinoma
B Grave’s disease
C Addison’s disease
D Schmidst’s syndrome
E Acromegaly
F Conn’s syndrome
G Kallman’s syndrome
H Secondary hypoaldosteronism 
I De Quervain’s thyroiditis

1) 38-year-old woman is referred by her GP to the Endocrine Clinic for further tests after experiencing fatigue and orthostatic hypotension. After a positive short synACTHen test, a long synACTHen test reveals a cortisol of 750 nmol/L after 24 hours.

A

1)C 2)F 3)A 4)B 5)E

Addison’s disease (C) is caused by primary adrenal insufficiency result- ing in a reduced production of cortisol and aldosterone. It is diagnosed using the synACTHen test. In the short synACTHen test, baseline plasma cortisol is measured at 0 minutes, the patient is given 250μg of synthetic ACTH at 30 minutes and plasma cortisol is rechecked at 60 minutes; if the final plasma cortisol is <550nmol/L, a defect in corti- sol production exists. The long synACTHen test distinguishes between primary and secondary adrenal insufficiency. A 1mg dose of synthetic ACTH is administered; after 24 hours, a cortisol level of <900nmol/L signifies a primary defect. Due to reduced mineralocorticoid production, blood tests will also reveal a hyponatraemia and hyperkalaemia.

Conn’s syndrome (F) is defined as primary hyperaldosteronism second- ary to an aldosterone-producing adrenal adenoma. As a result of the high aldosterone levels produced there will be an increased excretion of potassium and reabsorption of sodium, leading to hypokalaemia and hypernatraemia. The increased delivery of sodium to the juxtaglomerular apparatus causes renin levels to be reduced. Plasma aldosterone will either be raised or inappropriately normal (as ACTH is suppressed, aldosterone should physiologically be reduced).

A prolactinoma (A) is a prolactin-producing tumour and is the most prevalent pituitary tumour. Prolactinomas are classified according to size: microprolactinoma <10mm diameter and macroprolactinoma >10mm diameter. The clinical consequences of prolactinoma are divided into, first, those that occur as a result of increased prolactin production and, second, effects due to the mass effect of the tumour. Hormonal effects of prolactin include amenorrhoea, galactorrhoea and gynaeco- mastia in males. Mass effects of the tumour can lead to compression of pituitary cells producing other hormones such as thyroid stimulating hormone, growth hormone and ACTH.

Grave’s disease (B) is an autoimmune condition resulting in the produc- tion of TSH-receptor antibodies, leading to elevated levels of T3 and T4. TSH levels will therefore be suppressed as a result of negative feedback. Clinical features will include exophthalmos, pretibial myxoedema, diffuse thyroid enlargement as well as other systemic features of hyper- thyroiditis (tremor, excess sweating, heat intolerance and unintentional weight loss). There is a strong association with other autoimmune con- ditions such as vitiligo and type 1 diabetes mellitus.

Acromegaly (E) is caused by the increased secretion of growth hormone as a result of a pituitary adenoma (rarely there may be ectopic production). Serum growth hormone levels are not a useful marker of acromegaly due to its pulsatile release from the pituitary. The diagnostic test for acromegaly is the oral glucose tolerance test with synchronous growth hormone measurement: 75 mg of glucose is administered to the patient; if growth hormone levels are not suppressed to below 2 mU/L, a diagnosis of acromegaly is made.

Schmidst’s syndrome (D), also known as autoimmune polyendocrine syndrome type 2, is associated with Addison’s disease, hypothyroidism and type 1 diabetes mellitus.

Kallman’s syndrome (G) is a genetic disorder that results in hypogonadotropic hypogonadism. As a consequence there is reduced production of LH and FSH in the pituitary. Anosmia is an associated feature.

Secondary hyporaldosteronism (H) is defined by a defect in the pituitary gland which results in reduced ACTH production, and hence reduced cortisol and aldosterone. The long synACTHen test will reveal a cortisol of >900nmol/L as there is a delayed rise in production in the adrenal glands.

De Quervain’s thyroiditis (I) is a post virus induced thyroiditis which initially presents as hyperthyroidism because thyroxine from colloid enters the circulation. Hypothyroidism then ensues for a period as thyroxine stores are depleted.

37
Q

Endocrine chemical pathology

A Prolactinoma
B Grave’s disease
C Addison’s disease
D Schmidst’s syndrome
E Acromegaly
F Conn’s syndrome
G Kallman’s syndrome
H Secondary hypoaldosteronism 
I De Quervain’s thyroiditis

2) A 48-year-old man visits his GP complaining of muscle pain and weakness. He is found to have raised blood pressure. Blood tests reveal Na 149 (135–145mmol/L) and K 3.1 (3.5–5.0mmol/L).

A

1)C 2)F 3)A 4)B 5)E

Addison’s disease (C) is caused by primary adrenal insufficiency result- ing in a reduced production of cortisol and aldosterone. It is diagnosed using the synACTHen test. In the short synACTHen test, baseline plasma cortisol is measured at 0 minutes, the patient is given 250μg of synthetic ACTH at 30 minutes and plasma cortisol is rechecked at 60 minutes; if the final plasma cortisol is <550nmol/L, a defect in cortisol production exists. The long synACTHen test distinguishes between primary and secondary adrenal insufficiency. A 1mg dose of synthetic ACTH is administered; after 24 hours, a cortisol level of <900nmol/L signifies a primary defect. Due to reduced mineralocorticoid production, blood tests will also reveal a hyponatraemia and hyperkalaemia.

Conn’s syndrome (F) is defined as primary hyperaldosteronism secondary to an aldosterone-producing adrenal adenoma. As a result of the high aldosterone levels produced there will be an increased excretion of potassium and reabsorption of sodium, leading to hypokalaemia and hypernatraemia. The increased delivery of sodium to the juxtaglomerular apparatus causes renin levels to be reduced. Plasma aldosterone will either be raised or inappropriately normal (as ACTH is suppressed, aldosterone should physiologically be reduced).

A prolactinoma (A) is a prolactin-producing tumour and is the most prevalent pituitary tumour. Prolactinomas are classified according to size: microprolactinoma <10mm diameter and macroprolactinoma >10mm diameter. The clinical consequences of prolactinoma are divided into, first, those that occur as a result of increased prolactin production and, second, effects due to the mass effect of the tumour. Hormonal effects of prolactin include amenorrhoea, galactorrhoea and gynaecomastia in males. Mass effects of the tumour can lead to compression of pituitary cells producing other hormones such as thyroid stimulating hormone, growth hormone and ACTH.

Grave’s disease (B) is an autoimmune condition resulting in the production of TSH-receptor antibodies, leading to elevated levels of T3 and T4. TSH levels will therefore be suppressed as a result of negative feedback. Clinical features will include exophthalmos, pretibial myxoedema, diffuse thyroid enlargement as well as other systemic features of hyper- thyroiditis (tremor, excess sweating, heat intolerance and unintentional weight loss). There is a strong association with other autoimmune con- ditions such as vitiligo and type 1 diabetes mellitus.

Acromegaly (E) is caused by the increased secretion of growth hormone as a result of a pituitary adenoma (rarely there may be ectopic production). Serum growth hormone levels are not a useful marker of acromegaly due to its pulsatile release from the pituitary. The diagnostic test for acromegaly is the oral glucose tolerance test with synchronous growth hormone measurement: 75 mg of glucose is administered to the patient; if growth hormone levels are not suppressed to below 2 mU/L, a diagnosis of acromegaly is made.

Schmidst’s syndrome (D), also known as autoimmune polyendocrine syndrome type 2, is associated with Addison’s disease, hypothyroidism and type 1 diabetes mellitus.

Kallman’s syndrome (G) is a genetic disorder that results in hypogonadotropic hypogonadism. As a consequence there is reduced production of LH and FSH in the pituitary. Anosmia is an associated feature.

Secondary hyporaldosteronism (H) is defined by a defect in the pituitary gland which results in reduced ACTH production, and hence reduced cortisol and aldosterone. The long synACTHen test will reveal a cortisol of >900nmol/L as there is a delayed rise in production in the adrenal glands.

De Quervain’s thyroiditis (I) is a post virus induced thyroiditis which initially presents as hyperthyroidism because thyroxine from colloid enters the circulation. Hypothyroidism then ensues for a period as thyroxine stores are depleted.

38
Q

Endocrine chemical pathology

A Prolactinoma
B Grave’s disease
C Addison’s disease
D Schmidst’s syndrome
E Acromegaly
F Conn’s syndrome
G Kallman’s syndrome
H Secondary hypoaldosteronism 
I De Quervain’s thyroiditis

3) A 39-year-old woman sees an endocrinologist due to recent onset galactorrhoea. She denies recent child birth. TFTs are found to be normal.

A

1)C 2)F 3)A 4)B 5)E

Addison’s disease (C) is caused by primary adrenal insufficiency result- ing in a reduced production of cortisol and aldosterone. It is diagnosed using the synACTHen test. In the short synACTHen test, baseline plasma cortisol is measured at 0 minutes, the patient is given 250μg of synthetic ACTH at 30 minutes and plasma cortisol is rechecked at 60 minutes; if the final plasma cortisol is <550nmol/L, a defect in cortisol production exists. The long synACTHen test distinguishes between primary and secondary adrenal insufficiency. A 1mg dose of synthetic ACTH is administered; after 24 hours, a cortisol level of <900nmol/L signifies a primary defect. Due to reduced mineralocorticoid production, blood tests will also reveal a hyponatraemia and hyperkalaemia.

Conn’s syndrome (F) is defined as primary hyperaldosteronism secondary to an aldosterone-producing adrenal adenoma. As a result of the high aldosterone levels produced there will be an increased excretion of potassium and reabsorption of sodium, leading to hypokalaemia and hypernatraemia. The increased delivery of sodium to the juxtaglomerular apparatus causes renin levels to be reduced. Plasma aldosterone will either be raised or inappropriately normal (as ACTH is suppressed, aldosterone should physiologically be reduced).

A prolactinoma (A) is a prolactin-producing tumour and is the most prevalent pituitary tumour. Prolactinomas are classified according to size: microprolactinoma <10mm diameter and macroprolactinoma >10mm diameter. The clinical consequences of prolactinoma are divided into, first, those that occur as a result of increased prolactin production and, second, effects due to the mass effect of the tumour. Hormonal effects of prolactin include amenorrhoea, galactorrhoea and gynaecomastia in males. Mass effects of the tumour can lead to compression of pituitary cells producing other hormones such as thyroid stimulating hormone, growth hormone and ACTH.

Grave’s disease (B) is an autoimmune condition resulting in the production of TSH-receptor antibodies, leading to elevated levels of T3 and T4. TSH levels will therefore be suppressed as a result of negative feedback. Clinical features will include exophthalmos, pretibial myxoedema, diffuse thyroid enlargement as well as other systemic features of hyper- thyroiditis (tremor, excess sweating, heat intolerance and unintentional weight loss). There is a strong association with other autoimmune con- ditions such as vitiligo and type 1 diabetes mellitus.

Acromegaly (E) is caused by the increased secretion of growth hormone as a result of a pituitary adenoma (rarely there may be ectopic production). Serum growth hormone levels are not a useful marker of acromegaly due to its pulsatile release from the pituitary. The diagnostic test for acromegaly is the oral glucose tolerance test with synchronous growth hormone measurement: 75 mg of glucose is administered to the patient; if growth hormone levels are not suppressed to below 2 mU/L, a diagnosis of acromegaly is made.

Schmidst’s syndrome (D), also known as autoimmune polyendocrine syndrome type 2, is associated with Addison’s disease, hypothyroidism and type 1 diabetes mellitus.

Kallman’s syndrome (G) is a genetic disorder that results in hypogonadotropic hypogonadism. As a consequence there is reduced production of LH and FSH in the pituitary. Anosmia is an associated feature.

Secondary hyporaldosteronism (H) is defined by a defect in the pituitary gland which results in reduced ACTH production, and hence reduced cortisol and aldosterone. The long synACTHen test will reveal a cortisol of >900nmol/L as there is a delayed rise in production in the adrenal glands.

De Quervain’s thyroiditis (I) is a post virus induced thyroiditis which initially presents as hyperthyroidism because thyroxine from colloid enters the circulation. Hypothyroidism then ensues for a period as thyroxine stores are depleted.

39
Q

Endocrine chemical pathology

A Prolactinoma
B Grave’s disease
C Addison’s disease
D Schmidst’s syndrome
E Acromegaly
F Conn’s syndrome
G Kallman’s syndrome
H Secondary hypoaldosteronism 
I De Quervain’s thyroiditis

4) A 46-year-old man is seen by his GP after experiencing tremors, heat intolerance and weight loss. His wife complained that his eyes were bulging. Blood tests reveal T3 (1.2–3.0nmol/L), T4 (70–140nmol/L), TSH (0.5–5.7mIU/L).

A

1)C 2)F 3)A 4)B 5)E

Addison’s disease (C) is caused by primary adrenal insufficiency result- ing in a reduced production of cortisol and aldosterone. It is diagnosed using the synACTHen test. In the short synACTHen test, baseline plasma cortisol is measured at 0 minutes, the patient is given 250μg of synthetic ACTH at 30 minutes and plasma cortisol is rechecked at 60 minutes; if the final plasma cortisol is <550nmol/L, a defect in cortisol production exists. The long synACTHen test distinguishes between primary and secondary adrenal insufficiency. A 1mg dose of synthetic ACTH is administered; after 24 hours, a cortisol level of <900nmol/L signifies a primary defect. Due to reduced mineralocorticoid production, blood tests will also reveal a hyponatraemia and hyperkalaemia.

Conn’s syndrome (F) is defined as primary hyperaldosteronism secondary to an aldosterone-producing adrenal adenoma. As a result of the high aldosterone levels produced there will be an increased excretion of potassium and reabsorption of sodium, leading to hypokalaemia and hypernatraemia. The increased delivery of sodium to the juxtaglomerular apparatus causes renin levels to be reduced. Plasma aldosterone will either be raised or inappropriately normal (as ACTH is suppressed, aldosterone should physiologically be reduced).

A prolactinoma (A) is a prolactin-producing tumour and is the most prevalent pituitary tumour. Prolactinomas are classified according to size: microprolactinoma <10mm diameter and macroprolactinoma >10mm diameter. The clinical consequences of prolactinoma are divided into, first, those that occur as a result of increased prolactin production and, second, effects due to the mass effect of the tumour. Hormonal effects of prolactin include amenorrhoea, galactorrhoea and gynaecomastia in males. Mass effects of the tumour can lead to compression of pituitary cells producing other hormones such as thyroid stimulating hormone, growth hormone and ACTH.

Grave’s disease (B) is an autoimmune condition resulting in the production of TSH-receptor antibodies, leading to elevated levels of T3 and T4. TSH levels will therefore be suppressed as a result of negative feedback. Clinical features will include exophthalmos, pretibial myxoedema, diffuse thyroid enlargement as well as other systemic features of hyper- thyroiditis (tremor, excess sweating, heat intolerance and unintentional weight loss). There is a strong association with other autoimmune con- ditions such as vitiligo and type 1 diabetes mellitus.

Acromegaly (E) is caused by the increased secretion of growth hormone as a result of a pituitary adenoma (rarely there may be ectopic production). Serum growth hormone levels are not a useful marker of acromegaly due to its pulsatile release from the pituitary. The diagnostic test for acromegaly is the oral glucose tolerance test with synchronous growth hormone measurement: 75 mg of glucose is administered to the patient; if growth hormone levels are not suppressed to below 2 mU/L, a diagnosis of acromegaly is made.

Schmidst’s syndrome (D), also known as autoimmune polyendocrine syndrome type 2, is associated with Addison’s disease, hypothyroidism and type 1 diabetes mellitus.

Kallman’s syndrome (G) is a genetic disorder that results in hypogonadotropic hypogonadism. As a consequence there is reduced production of LH and FSH in the pituitary. Anosmia is an associated feature.

Secondary hyporaldosteronism (H) is defined by a defect in the pituitary gland which results in reduced ACTH production, and hence reduced cortisol and aldosterone. The long synACTHen test will reveal a cortisol of >900nmol/L as there is a delayed rise in production in the adrenal glands.

De Quervain’s thyroiditis (I) is a post virus induced thyroiditis which initially presents as hyperthyroidism because thyroxine from colloid enters the circulation. Hypothyroidism then ensues for a period as thyroxine stores are depleted.

40
Q

Endocrine chemical pathology

A Prolactinoma
B Grave’s disease
C Addison’s disease
D Schmidst’s syndrome
E Acromegaly
F Conn’s syndrome
G Kallman’s syndrome
H Secondary hypoaldosteronism 
I De Quervain’s thyroiditis

5) A 45-year-old woman is referred to an endocrinologist due to the appearance of enlarged hands and feet as well as a protruding jaw. After conducting an oral glucose tolerance test, growth hormone levels are found to be 5 mU/L (<2 mU/L).

A

1)C 2)F 3)A 4)B 5)E

Addison’s disease (C) is caused by primary adrenal insufficiency result- ing in a reduced production of cortisol and aldosterone. It is diagnosed using the synACTHen test. In the short synACTHen test, baseline plasma cortisol is measured at 0 minutes, the patient is given 250μg of synthetic ACTH at 30 minutes and plasma cortisol is rechecked at 60 minutes; if the final plasma cortisol is <550nmol/L, a defect in cortisol production exists. The long synACTHen test distinguishes between primary and secondary adrenal insufficiency. A 1mg dose of synthetic ACTH is administered; after 24 hours, a cortisol level of <900nmol/L signifies a primary defect. Due to reduced mineralocorticoid production, blood tests will also reveal a hyponatraemia and hyperkalaemia.

Conn’s syndrome (F) is defined as primary hyperaldosteronism secondary to an aldosterone-producing adrenal adenoma. As a result of the high aldosterone levels produced there will be an increased excretion of potassium and reabsorption of sodium, leading to hypokalaemia and hypernatraemia. The increased delivery of sodium to the juxtaglomerular apparatus causes renin levels to be reduced. Plasma aldosterone will either be raised or inappropriately normal (as ACTH is suppressed, aldosterone should physiologically be reduced).

A prolactinoma (A) is a prolactin-producing tumour and is the most prevalent pituitary tumour. Prolactinomas are classified according to size: microprolactinoma <10mm diameter and macroprolactinoma >10mm diameter. The clinical consequences of prolactinoma are divided into, first, those that occur as a result of increased prolactin production and, second, effects due to the mass effect of the tumour. Hormonal effects of prolactin include amenorrhoea, galactorrhoea and gynaecomastia in males. Mass effects of the tumour can lead to compression of pituitary cells producing other hormones such as thyroid stimulating hormone, growth hormone and ACTH.

Grave’s disease (B) is an autoimmune condition resulting in the production of TSH-receptor antibodies, leading to elevated levels of T3 and T4. TSH levels will therefore be suppressed as a result of negative feedback. Clinical features will include exophthalmos, pretibial myxoedema, diffuse thyroid enlargement as well as other systemic features of hyper- thyroiditis (tremor, excess sweating, heat intolerance and unintentional weight loss). There is a strong association with other autoimmune con- ditions such as vitiligo and type 1 diabetes mellitus.

Acromegaly (E) is caused by the increased secretion of growth hormone as a result of a pituitary adenoma (rarely there may be ectopic production). Serum growth hormone levels are not a useful marker of acromegaly due to its pulsatile release from the pituitary. The diagnostic test for acromegaly is the oral glucose tolerance test with synchronous growth hormone measurement: 75 mg of glucose is administered to the patient; if growth hormone levels are not suppressed to below 2 mU/L, a diagnosis of acromegaly is made.

Schmidst’s syndrome (D), also known as autoimmune polyendocrine syndrome type 2, is associated with Addison’s disease, hypothyroidism and type 1 diabetes mellitus.

Kallman’s syndrome (G) is a genetic disorder that results in hypogonadotropic hypogonadism. As a consequence there is reduced production of LH and FSH in the pituitary. Anosmia is an associated feature.

Secondary hyporaldosteronism (H) is defined by a defect in the pituitary gland which results in reduced ACTH production, and hence reduced cortisol and aldosterone. The long synACTHen test will reveal a cortisol of >900nmol/L as there is a delayed rise in production in the adrenal glands.

De Quervain’s thyroiditis (I) is a post virus induced thyroiditis which initially presents as hyperthyroidism because thyroxine from colloid enters the circulation. Hypothyroidism then ensues for a period as thyroxine stores are depleted.

41
Q

Inborn errors of metabolism

A Phenylketonuria (PKU)
B Peroxisomal disorders
C Maple syrup urine disease
D Short-chain acyl-coenzyme A dehydrogenase (SCAD) deficiency
E Von Gierke’s disease 
F Fabry’s disease
G Urea cycle disorder 
H Homocystinuria Galactosaemia

1) An 18-month-old girl is seen by the GP. Her mother is concerned by the child’s brittle hair and inability to walk. The mother reports her daughter has had two previous convulsions.

A

1) H

Homocystinuria (H) is an amino acid disorder in which there is a defi- ciency in the enzyme cystathionine synthetase. This metabolic disorder presents in childhood with characteristic features such as very fair skin and brittle hair. The condition will usually lead to developmental delay or progressive learning difficulties. Convulsions, skeletal abnormalities and thrombotic episodes have also been reported. Management options include supplementing with vitamin B6 (pyridoxine) or maintaining the child on a low-methionine diet.

Peroxisomal disorders (A) result in disordered beta-oxidation of very-long-chain fatty acids (VLCFA); these accumulate in the blood stream. In neonates, such disorders lead to seizures, dysmorphic features, severe muscular hypotonia and jaundice.

Short-chain acyl-coenzyme A dehydrogenase (SCAD) deficiency (D) is one of the four fatty acid oxidation disorders, which is unique in its neonatal presentation with failure to thrive, hypotonia, metabolic acidosis and hyperglycaemia.

Urea cycle disorders (G) arise due to deficiency in one of the six enzymes in the urea cycle, resulting in hyperammonaemia. Enzyme deficiency occurs in an autosomal recessive fashion. Symptoms depend on age of presentation, but overall encephalopathy ensues with primarily neurological features.

Galactosaemia (I) results from the deficiency in the enzyme galactose- 1-phosphate uridyl transferase (Gal-1-PUT). Symptoms occur in the infant after milk ingestion, usually poor feeding, vomiting, jaundice and hepatomegaly. A galactose-free diet is the primary management option.

42
Q

Inborn errors of metabolism

A Phenylketonuria (PKU)
B Peroxisomal disorders
C Maple syrup urine disease
D Short-chain acyl-coenzyme A dehydrogenase (SCAD) deficiency
E Von Gierke’s disease 
F Fabry’s disease
G Urea cycle disorder 
H Homocystinuria Galactosaemia

2) A fair haired 8-month-old baby, born in Syria, is seen together with his mother in the paediatric outpatient clinic. He is found to have developmental delay and a musty smell is being given off by the baby.

A

2) A

Phenylketonuria (PKU; A) is also an amino acid disorder. Children clas- sically lack the enzyme phenylalanine hydroxylase, but other co-factors may be aberrant. Since the 1960s PKU has been diagnosed at birth using the Guthrie test but in some countries the test may not be available. The child will be fair-haired and present with developmental delay between 6 and 12 months of age. Later in life, the child’s IQ will be severely impaired. Eczema and seizures have also been implicated in the disease process.

Peroxisomal disorders (A) result in disordered beta-oxidation of very-long-chain fatty acids (VLCFA); these accumulate in the blood stream. In neonates, such disorders lead to seizures, dysmorphic features, severe muscular hypotonia and jaundice.

Short-chain acyl-coenzyme A dehydrogenase (SCAD) deficiency (D) is one of the four fatty acid oxidation disorders, which is unique in its neonatal presentation with failure to thrive, hypotonia, metabolic acidosis and hyperglycaemia.

Urea cycle disorders (G) arise due to deficiency in one of the six enzymes in the urea cycle, resulting in hyperammonaemia. Enzyme deficiency occurs in an autosomal recessive fashion. Symptoms depend on age of presentation, but overall encephalopathy ensues with primarily neurological features.

Galactosaemia (I) results from the deficiency in the enzyme galactose- 1-phosphate uridyl transferase (Gal-1-PUT). Symptoms occur in the infant after milk ingestion, usually poor feeding, vomiting, jaundice and hepatomegaly. A galactose-free diet is the primary management option.

43
Q

Inborn errors of metabolism

A Phenylketonuria (PKU)
B Peroxisomal disorders
C Maple syrup urine disease
D Short-chain acyl-coenzyme A dehydrogenase (SCAD) deficiency
E Von Gierke’s disease 
F Fabry’s disease
G Urea cycle disorder 
H Homocystinuria Galactosaemia

3) A 9-month-old baby is seen in accident and emergency as her mother has reported that she has become ‘floppy’. The baby is found to be hypoglycaemic and on examination an enlarged liver and kidneys are noted.

A

3) E

Von Gierke’s disease (E) is one of nine glycogen storage disorders, in which a defect in the enzyme glucose-6-phosphate results in a failure of mobilization of glucose from glycogen. The metabolic disease presents in infancy with hypoglycaemia. The liver is usually significantly enlarged and kidney enlargement can also occur. Other glycogen storage disorders (and enzyme defects) include Pompe’s (lysosomal alpha-glucosidase), Cori’s (amylo-1,6-glucosidase) and McArdle’s (phosphorylase); each disorder presents with varying degrees of liver and muscle dysfunction.

Peroxisomal disorders (A) result in disordered beta-oxidation of very-long-chain fatty acids (VLCFA); these accumulate in the blood stream. In neonates, such disorders lead to seizures, dysmorphic features, severe muscular hypotonia and jaundice.

Short-chain acyl-coenzyme A dehydrogenase (SCAD) deficiency (D) is one of the four fatty acid oxidation disorders, which is unique in its neonatal presentation with failure to thrive, hypotonia, metabolic acidosis and hyperglycaemia.

Urea cycle disorders (G) arise due to deficiency in one of the six enzymes in the urea cycle, resulting in hyperammonaemia. Enzyme deficiency occurs in an autosomal recessive fashion. Symptoms depend on age of presentation, but overall encephalopathy ensues with primarily neurological features.

Galactosaemia (I) results from the deficiency in the enzyme galactose- 1-phosphate uridyl transferase (Gal-1-PUT). Symptoms occur in the infant after milk ingestion, usually poor feeding, vomiting, jaundice and hepatomegaly. A galactose-free diet is the primary management option.

44
Q

Inborn errors of metabolism

A Phenylketonuria (PKU)
B Peroxisomal disorders
C Maple syrup urine disease
D Short-chain acyl-coenzyme A dehydrogenase (SCAD) deficiency
E Von Gierke’s disease 
F Fabry’s disease
G Urea cycle disorder 
H Homocystinuria Galactosaemia

4) A 14-day-old girl of Jewish descent presents with lethargy, poor feeding and hypotonia. The paediatrician examining the child also notices excessively sweaty feet.

A

4)C

Maple syrup urine disease (C) is an organic aciduria, a group of disorders that represent impaired metabolism of leucine, isoleucine and valine. As a result, toxic compounds accumulate causing toxic encephalopathy which manifests as lethargy, poor feeding, hypotonia and/or seizures. Characteristic of maple syrup urine disease are a sweet odour and sweaty feet. The gold standard diagnostic test is gas chromatography with mass spectrometry. Management involves the avoidance of the causative amino acids.

Peroxisomal disorders (A) result in disordered beta-oxidation of very-long-chain fatty acids (VLCFA); these accumulate in the blood stream. In neonates, such disorders lead to seizures, dysmorphic features, severe muscular hypotonia and jaundice.

Short-chain acyl-coenzyme A dehydrogenase (SCAD) deficiency (D) is one of the four fatty acid oxidation disorders, which is unique in its neonatal presentation with failure to thrive, hypotonia, metabolic acidosis and hyperglycaemia.

Urea cycle disorders (G) arise due to deficiency in one of the six enzymes in the urea cycle, resulting in hyperammonaemia. Enzyme deficiency occurs in an autosomal recessive fashion. Symptoms depend on age of presentation, but overall encephalopathy ensues with primarily neurological features.

Galactosaemia (I) results from the deficiency in the enzyme galactose- 1-phosphate uridyl transferase (Gal-1-PUT). Symptoms occur in the infant after milk ingestion, usually poor feeding, vomiting, jaundice and hepatomegaly. A galactose-free diet is the primary management option.

45
Q

Inborn errors of metabolism

A Phenylketonuria (PKU)
B Peroxisomal disorders
C Maple syrup urine disease
D Short-chain acyl-coenzyme A dehydrogenase (SCAD) deficiency
E Von Gierke’s disease 
F Fabry’s disease
G Urea cycle disorder 
H Homocystinuria Galactosaemia

5) A 5-month-old boy is seen by the community paediatrician due to concerns of developmental delay. On examination dysmorphic features are noted, as well as a ‘cherry-red spot’ on the baby’s trunk.

A

5) F

Fabry’s disease (F) is a lysosomal storage disorder in which there is deficiency in alpha-galactosidase. Presentation is almost always a child with developmental delay together with dysmorphia. Other findings may involve movement abnormalities, seizures, deafness and/or blindness. On examination, hepatosplenomegaly, pulmonary and cardiac problems may be noted. The pathognomonic feature of lysosomal storage disorders is the presence of a ‘cherry-red spot’.

Peroxisomal disorders (A) result in disordered beta-oxidation of very-long-chain fatty acids (VLCFA); these accumulate in the blood stream. In neonates, such disorders lead to seizures, dysmorphic features, severe muscular hypotonia and jaundice.

Short-chain acyl-coenzyme A dehydrogenase (SCAD) deficiency (D) is one of the four fatty acid oxidation disorders, which is unique in its neonatal presentation with failure to thrive, hypotonia, metabolic acidosis and hyperglycaemia.

Urea cycle disorders (G) arise due to deficiency in one of the six enzymes in the urea cycle, resulting in hyperammonaemia. Enzyme deficiency occurs in an autosomal recessive fashion. Symptoms depend on age of presentation, but overall encephalopathy ensues with primarily neurological features.

Galactosaemia (I) results from the deficiency in the enzyme galactose- 1-phosphate uridyl transferase (Gal-1-PUT). Symptoms occur in the infant after milk ingestion, usually poor feeding, vomiting, jaundice and hepatomegaly. A galactose-free diet is the primary management option.

46
Q

Liver function tests

A Alcohol abuse
B Gilbert’s syndrome
C Gallstones
D Dublin–Johnson syndrome
E Non-alcoholic fatty liver disease
F Crigler–Najjar syndrome 
G Alcoholic liver disease
H Paracetamol poisoning
I Hepatocellular carcinoma

1)

AST 65 (3–35IU/L) 
ALT 72 (3–35IU/L)
GGT 82 (11–51IU/L)
ALP 829 (35–51IU/L)
Total bilirubin 234 (3–17μmol/L) 
Conjugated bilirubin 63 (1.0–5.1μmol/L)
A

1)C

Gallstones (C) may be composed of cholesterol, bilirubin or mixed in nature. The major complication of gallstones is cholestasis, whereby the flow of bile is blocked from the liver to the duodenum. This results in right upper quadrant abdominal pain, nausea and vomiting. Other causes of cholestasis include primary biliary cirrhosis, primary sclerosing cholangitis and abdominal masses compressing the biliary tree. Biochemically, cholestasis is defined by rises in GGT and ALP (obstructive picture) that are greater than the rises in AST and ALT.

Dublin–Johnson syndrome (D) is an autosomal recessive disorder that results in a raised conjugated bilirubin level due to reduced secretion of conjugated bilirubin into the bile. AST and ALT levels are normal.

Crigler–Najjar syndrome (F) is a hereditary disease resulting in either complete (type 1) or partial (type 2) reduction in the conjugating enzyme UDP glucuronosyl transferase causing an unconjugated hyperbilirubinaemia.

Alcoholic liver disease (ALD; G) occurs in three stages: alcoholic steato- sis, alcoholic hepatitis and eventually cirrhosis. GGT, AST and ALT will be markedly elevated (AST:ALT ratio >2).

Hepatocellular carcinoma (HCC; I) occurs as a result of underlying cirrhosis. Raised alpha-fetoprotein levels can be indicative of HCC. Deranged liver function tests will reflect the underlying pathology.

47
Q

Liver function tests

A Alcohol abuse
B Gilbert’s syndrome
C Gallstones
D Dublin–Johnson syndrome
E Non-alcoholic fatty liver disease
F Crigler–Najjar syndrome 
G Alcoholic liver disease
H Paracetamol poisoning
I Hepatocellular carcinoma

2)

AST 32 (3–35IU/L) 
ALT 29 (3–35IU/L)
GGT 34 (11–51IU/L)
ALP 53 (35–51IU/L)
Total bilirubin 36 (3–17μmol/L) 
Conjugated bilirubin 3.4 (1.0–5.1μmol/L)
A

2)B

Gilbert’s syndrome (B) is an autosomal dominant condition in which there is a mutation in the enzyme UDP glucuronosyl transferase which reduces conjugation of bilirubin in the liver. As a consequence patients experience mild, intermittent jaundice. Jaundice in patients with Gilbert’s syndrome may be precipitated by infection or starved states. Biochemistry will reveal that all liver function tests are normal apart from an isolated raised unconjugated bilirubin level, while conjugated bilirubin is within the normal range.

Dublin–Johnson syndrome (D) is an autosomal recessive disorder that results in a raised conjugated bilirubin level due to reduced secretion of conjugated bilirubin into the bile. AST and ALT levels are normal.

Crigler–Najjar syndrome (F) is a hereditary disease resulting in either complete (type 1) or partial (type 2) reduction in the conjugating enzyme UDP glucuronosyl transferase causing an unconjugated hyperbilirubinaemia.

Alcoholic liver disease (ALD; G) occurs in three stages: alcoholic steato- sis, alcoholic hepatitis and eventually cirrhosis. GGT, AST and ALT will be markedly elevated (AST:ALT ratio >2).

Hepatocellular carcinoma (HCC; I) occurs as a result of underlying cirrhosis. Raised alpha-fetoprotein levels can be indicative of HCC. Deranged liver function tests will reflect the underlying pathology.

48
Q

Liver function tests

A Alcohol abuse
B Gilbert’s syndrome
C Gallstones
D Dublin–Johnson syndrome
E Non-alcoholic fatty liver disease
F Crigler–Najjar syndrome 
G Alcoholic liver disease
H Paracetamol poisoning
I Hepatocellular carcinoma

3)

AST 1259 (3–35IU/L)
ALT 1563 (3–35IU/L)
GGT 73 (11–51IU/L)
ALP 46 (35–51IU/L)
Total bilirubin 15.2 (3–17μmol/L)
Conjugated bilirubin 4.2 (1.0–5.1μmol/L)
A

3)E

Non-alcoholic fatty liver disease (NAFLD; E) is due to fatty deposits in the liver (steatosis), but where the underlying cause is not due to alcohol. In such circumstances, aetiological factors include obesity, diabetes, parenteral feeding and inherited metabolic disorders (glycogen storage disease type 1). NAFLD may present with right upper quadrant pain or may be asymptomatic. Liver function tests will reveal raised AST and ALT levels (AST:ALT ratio <1) and increased GGT. Bilirubin and albumin levels are normal.

Paracetamol poisoning (H) is a common cause of acute liver failure. The clinical features of acute liver failure reflect the diminished synthetic and metabolic functioning of the liver. Characteristics include reduced blood sugar level, metabolic acidosis, increased tendency to bleed and hepatic encephalopathy. Biochemical tests will reveal AST and ALT lev- els greater than 1000IU/L. AST and ALT levels will be greater than GGT and ALP levels, reflecting the hepatic rather than obstructive picture of the pathology.

Alcohol abuse (A) can lead to deranged liver function tests. In the absence of underlying liver disease, biochemical investigation may demonstrate an isolated rise in GGT. There may also be mild elevations in AST and ALT, reflecting mild hepatic damage. Haematology results will show a macrocytic picture due to toxic effects of alcohol on the bone marrow. Isolated raised GGT levels may also occur due to the consumption of enzyme-inducing drugs such as phenytoin, carbamaz- epine and phenobarbitone.

Dublin–Johnson syndrome (D) is an autosomal recessive disorder that results in a raised conjugated bilirubin level due to reduced secretion of conjugated bilirubin into the bile. AST and ALT levels are normal.

Crigler–Najjar syndrome (F) is a hereditary disease resulting in either complete (type 1) or partial (type 2) reduction in the conjugating enzyme UDP glucuronosyl transferase causing an unconjugated hyperbilirubinaemia.

Alcoholic liver disease (ALD; G) occurs in three stages: alcoholic steato- sis, alcoholic hepatitis and eventually cirrhosis. GGT, AST and ALT will be markedly elevated (AST:ALT ratio >2).

Hepatocellular carcinoma (HCC; I) occurs as a result of underlying cirrhosis. Raised alpha-fetoprotein levels can be indicative of HCC. Deranged liver function tests will reflect the underlying pathology.

49
Q

Liver function tests

A Alcohol abuse
B Gilbert’s syndrome
C Gallstones
D Dublin–Johnson syndrome
E Non-alcoholic fatty liver disease
F Crigler–Najjar syndrome 
G Alcoholic liver disease
H Paracetamol poisoning
I Hepatocellular carcinoma

4)

AST 2321 (3–35IU/L)
ALT 2562 (3–35IU/L)
GGT 62 (11–51IU/L)
ALP 182 (35–51IU/L)
Total bilirubin 14 (3–17μmol/L)
Conjugated bilirubin 3.4 (1.0–5.1μmol/L)
A

4)H

Paracetamol poisoning (H) is a common cause of acute liver failure. The clinical features of acute liver failure reflect the diminished synthetic and metabolic functioning of the liver. Characteristics include reduced blood sugar level, metabolic acidosis, increased tendency to bleed and hepatic encephalopathy. Biochemical tests will reveal AST and ALT lev- els greater than 1000IU/L. AST and ALT levels will be greater than GGT and ALP levels, reflecting the hepatic rather than obstructive picture of the pathology.

Dublin–Johnson syndrome (D) is an autosomal recessive disorder that results in a raised conjugated bilirubin level due to reduced secretion of conjugated bilirubin into the bile. AST and ALT levels are normal.

Crigler–Najjar syndrome (F) is a hereditary disease resulting in either complete (type 1) or partial (type 2) reduction in the conjugating enzyme UDP glucuronosyl transferase causing an unconjugated hyperbilirubinaemia.

Alcoholic liver disease (ALD; G) occurs in three stages: alcoholic steato- sis, alcoholic hepatitis and eventually cirrhosis. GGT, AST and ALT will be markedly elevated (AST:ALT ratio >2).

Hepatocellular carcinoma (HCC; I) occurs as a result of underlying cirrhosis. Raised alpha-fetoprotein levels can be indicative of HCC. Deranged liver function tests will reflect the underlying pathology.

50
Q

Liver function tests

A Alcohol abuse
B Gilbert’s syndrome
C Gallstones
D Dublin–Johnson syndrome
E Non-alcoholic fatty liver disease
F Crigler–Najjar syndrome 
G Alcoholic liver disease
H Paracetamol poisoning
I Hepatocellular carcinoma

5)

AST 34 (3–35IU/L)
ALT 32 (3–35IU/L)
GGT 134 (11–51IU/L)
ALP 123 (35–51IU/L)
Total bilirubin (3–17μmol/L)
Conjugated bilirubin (1.0–5.1μmol/L)
A

5)A

Alcohol abuse (A) can lead to deranged liver function tests. In the absence of underlying liver disease, biochemical investigation may demonstrate an isolated rise in GGT. There may also be mild elevations in AST and ALT, reflecting mild hepatic damage. Haematology results will show a macrocytic picture due to toxic effects of alcohol on the bone marrow. Isolated raised GGT levels may also occur due to the consumption of enzyme-inducing drugs such as phenytoin, carbamazepine and phenobarbitone.

Dublin–Johnson syndrome (D) is an autosomal recessive disorder that results in a raised conjugated bilirubin level due to reduced secretion of conjugated bilirubin into the bile. AST and ALT levels are normal.

Crigler–Najjar syndrome (F) is a hereditary disease resulting in either complete (type 1) or partial (type 2) reduction in the conjugating enzyme UDP glucuronosyl transferase causing an unconjugated hyperbilirubinaemia.

Alcoholic liver disease (ALD; G) occurs in three stages: alcoholic steato- sis, alcoholic hepatitis and eventually cirrhosis. GGT, AST and ALT will be markedly elevated (AST:ALT ratio >2).

Hepatocellular carcinoma (HCC; I) occurs as a result of underlying cirrhosis. Raised alpha-fetoprotein levels can be indicative of HCC. Deranged liver function tests will reflect the underlying pathology

51
Q

Plasma proteins

A Bence–Jones protein
B Carcino-embryonic antigen
C Caeruloplasmin
D Fibrinogen
E Amylase
F Ferritin
G Alpha-Fetoprotein 
H Albumin
I CA125

1) A 13-year-old boy presents to his GP with parotitis with pain in his testes. His previous history reveals an incomplete childhood vaccination record.

A

1) E

Amylase (E) is an enzyme that breaks down starch into maltose. Serum amylase levels are often elevated during inflammation involving the parotid glands (parotitis) as occurs in mumps. Amylase is produced in the salivary glands, the parotid gland being the largest producer of the enzyme. Inflammation of the parotid glands cause a release of amylase into the blood stream, hence elevating levels. Raised serum amylase levels are also used in the diagnosis of pancreatitis; the pancreas is another amylase producing site.

Carcino-embryonic antigen (CEA; B) is a glycoprotein that is raised primarily in gastrointestinal cancers such as colorectal carcinoma, gastric carcinoma and pancreatic carcinoma.

Fibrinogen (D) is a glycoprotein synthesized in the liver. It has an essential role in the coagulation cascade, being converted to fibrin in the presence of thrombin, an essential process during clot formation.

alpha-Fetoprotein (G) is a tumour marker especially raised in hepatocellular carcinoma and germ cell tumours. alpha-Fetoprotein is also used antena- tally to screen for neural tube defects and Down syndrome.

Albumin (H) is synthesized in the liver. Low plasma albumin levels result in oedema (liver disease, nephrotic syndrome and malabsorption). Raised plasma albumin levels are associated with dehydration.

52
Q

Plasma proteins

A Bence–Jones protein
B Carcino-embryonic antigen
C Caeruloplasmin
D Fibrinogen
E Amylase
F Ferritin
G Alpha-Fetoprotein 
H Albumin
I CA125

2) A 50-year-old patient who has a 4-week history of tiredness undergoes a colonoscopy. Bleeding is noted in the large intestine.

A

2) F

Ferritin (F) is an intracellular protein responsible for the safe storage of iron, as free iron can be toxic to cells. Gastrointestinal bleeding may cause iron deficiency anaemia (microcytic anaemia), characterized haematologically by a reduced serum iron, raised total iron binding capacity and reduced ferritin. Ferritin levels will distinguish between other causes of microcytic anaemia: anaemia of chronic disease (raised ferritin) and thalassaemia (normal ferritin). As ferritin is an acute-phase protein, it will also be raised secondary to inflammation.

Carcino-embryonic antigen (CEA; B) is a glycoprotein that is raised primarily in gastrointestinal cancers such as colorectal carcinoma, gastric carcinoma and pancreatic carcinoma.

Fibrinogen (D) is a glycoprotein synthesized in the liver. It has an essential role in the coagulation cascade, being converted to fibrin in the presence of thrombin, an essential process during clot formation.

alpha-Fetoprotein (G) is a tumour marker especially raised in hepatocellular carcinoma and germ cell tumours. alpha-Fetoprotein is also used antena- tally to screen for neural tube defects and Down syndrome.

Albumin (H) is synthesized in the liver. Low plasma albumin levels result in oedema (liver disease, nephrotic syndrome and malabsorption). Raised plasma albumin levels are associated with dehydration.

53
Q

Plasma proteins

A Bence–Jones protein
B Carcino-embryonic antigen
C Caeruloplasmin
D Fibrinogen
E Amylase
F Ferritin
G Alpha-Fetoprotein 
H Albumin
I CA125

3) A 62-year-old smoker with a history of ulcerative colitis presents to his GP with weight loss and tiredness. The patient admits noticing fresh blood mixed in with the stool.

A

3) A

Bence–Jones proteins (A) are monoclonal globular proteins that are a diagnostic feature of multiple myeloma. Multiple myeloma is defined as the proliferation of plasma cells in the bone marrow and is commonly associated with the elderly population. Malignant plasma cells produce monoclonal antibodies and/or kappa or lamba light chains (paraproteins). The light chains appear in the urine and can be detected by electrophoresis of a urine sample as a monoclonal band. Bence–Jones proteins are also a feature of Waldenstrom’s macroglobulinaemia and amyloid light chain amyloidosis.

Carcino-embryonic antigen (CEA; B) is a glycoprotein that is raised primarily in gastrointestinal cancers such as colorectal carcinoma, gastric carcinoma and pancreatic carcinoma.

Fibrinogen (D) is a glycoprotein synthesized in the liver. It has an essential role in the coagulation cascade, being converted to fibrin in the presence of thrombin, an essential process during clot formation.

alpha-Fetoprotein (G) is a tumour marker especially raised in hepatocellular carcinoma and germ cell tumours. alpha-Fetoprotein is also used antena- tally to screen for neural tube defects and Down syndrome.

Albumin (H) is synthesized in the liver. Low plasma albumin levels result in oedema (liver disease, nephrotic syndrome and malabsorption). Raised plasma albumin levels are associated with dehydration.

54
Q

Plasma proteins

A Bence–Jones protein
B Carcino-embryonic antigen
C Caeruloplasmin
D Fibrinogen
E Amylase
F Ferritin
G Alpha-Fetoprotein 
H Albumin
I CA125

4) A 42-year-old woman presents to her GP with weight loss and abdominal pain. Bimanual examination reveals a mass in the left adnexa.

A

4) I

CA-125 (cancer antigen 125; I) is a protein encoded by the MUC16 gene that may suggest the presence of ovarian cancer. Its low sensitivity and specificity prevents it from being a diagnostic marker but it is useful when used in conjunction with imaging modalities for the diagnosis of ovarian cancer. Many ovarian cancers are coelomic epithelial carcinomas and hence will express CA-125, which is a coelomic epithelium-related glycoprotein. CA-125 may be associated with endometrial, pancreatic and breast carcinomas but plasma levels are most elevated in ovarian cancer.

Carcino-embryonic antigen (CEA; B) is a glycoprotein that is raised primarily in gastrointestinal cancers such as colorectal carcinoma, gastric carcinoma and pancreatic carcinoma.

Fibrinogen (D) is a glycoprotein synthesized in the liver. It has an essential role in the coagulation cascade, being converted to fibrin in the presence of thrombin, an essential process during clot formation.

alpha-Fetoprotein (G) is a tumour marker especially raised in hepatocellular carcinoma and germ cell tumours. alpha-Fetoprotein is also used antena- tally to screen for neural tube defects and Down syndrome.

Albumin (H) is synthesized in the liver. Low plasma albumin levels result in oedema (liver disease, nephrotic syndrome and malabsorption). Raised plasma albumin levels are associated with dehydration.

55
Q

Plasma proteins

A Bence–Jones protein
B Carcino-embryonic antigen
C Caeruloplasmin
D Fibrinogen
E Amylase
F Ferritin
G Alpha-Fetoprotein 
H Albumin
I CA125

5) A 15-year-old boy is brought in by his mother who has noted a change in his behaviour as well as a tremor. On slit lamp examination, Keiser–Fleischer rings are noted around the iris.

A

5) C

Caeruloplasmin (C) is a copper carrying protein encoded by the CP gene. Low plasma caeruloplasmin levels are associated with Wilson’s disease, an autosomal recessive condition in which there is an accumulation of copper within organs due to a defect in the copper transporter ATP7B (linking copper to caeruloplasmin). As a result caeruloplasmin is degraded in the blood stream. Clinical manifestations include neurological and psychiatric symptoms, and copper accumulation within the iris of the eyes leading to Keiser–Fleischer rings is pathognomonic.

Carcino-embryonic antigen (CEA; B) is a glycoprotein that is raised primarily in gastrointestinal cancers such as colorectal carcinoma, gastric carcinoma and pancreatic carcinoma.

Fibrinogen (D) is a glycoprotein synthesized in the liver. It has an essential role in the coagulation cascade, being converted to fibrin in the presence of thrombin, an essential process during clot formation.

alpha-Fetoprotein (G) is a tumour marker especially raised in hepatocellular carcinoma and germ cell tumours. alpha-Fetoprotein is also used antena- tally to screen for neural tube defects and Down syndrome.

Albumin (H) is synthesized in the liver. Low plasma albumin levels result in oedema (liver disease, nephrotic syndrome and malabsorption). Raised plasma albumin levels are associated with dehydration.

56
Q

Potassium handling

A Spurious sample
B Anorexia
C Diarrhoea
D Renal tubular acidosis
E Insulin overdose
F Bartter syndrome 
G Frusemide
H Renal failure
I ACE inhibitors

1) A 15-year-old boy presents to accident and emergency with loss of consciousness. His blood sugars are found to be extremely low. Blood tests demonstrate the following:

Na 138 (135–145mmol/L)
K 3.0 (3.5–5.0mmol/L)
Urea 4.2 (3.0–7.0mmol/L) 
Creatinine 74 (60–120mmol/L) 
pH 7.48 (7.35–7.45)
HCO3 31 (22–28mmol/L)
A

1)E 2)H 3)F 4)I 5)D

Insulin overdose (E) in a diabetic patient will cause a redistributive hypokalaemia and concurrent metabolic alkalosis. Insulin causes a shift of potassium ions from the extracellular space to the intracellular space, thereby lowering blood potassium levels. Metabolic alkalosis can also cause a redistributive hypokalaemia; a reduced hydrogen ion concentration in the blood causes increased intracellular hydrogen ion loss to increase extracellular levels via Na+/H+ ATPase; potassium ions therefore diffuse intracellularly to maintain the electrochemical potential. Adrenaline and re-feeding syndrome also cause redistributive hypokalaemia.

Renal failure (H) can lead to hyperkalaemia secondary to reduced distal renal delivery of sodium ions. As a consequence, there is reduced exchange of potassium ions via the Na/K ATPase pump in the collecting duct, which thereby leads to accumulation of potassium ions in the blood and hence hyperkalaemia. An increase in aldosterone release will initially cause a compensatory loss of potassium ions; as renal failure progresses, this homeostatic mechanism will become decompensated and hyperkalaemia will result. Renal failure will also be reflected in the deranged urea and creatinine levels due to reduced excretion.

Bartter syndrome (F) is an autosomal recessive condition due to a defect in the thick ascending limb of the loop of Henle. It is characterized by hypokalaemia, alkalosis and hypotension. The condition may also lead to increased calcium loss via the urine (hypercalcuria) and the kidneys (nephrocalcinosis). Various genetic defects have been discovered; neonatal Bartter syndrome is due to mutations in either the NKCC2 or ROMK genes. In the associated milder Gitelman syndrome, the potassium transporting defect is in the distal convoluted tubule of the kidney.

ACE inhibitors (I) will lead to hyperkalaemia due to reduced potassium excretion. ACE inhibitors antagonize the effect of angiotensin converting enzyme, the enzyme which catalyzes the production of angiotensin II from angiotensin I. A decreased level of angiotensin II reduces the production of aldosterone in the adrenal glands, a key hormone causing the excretion of potassium. Other causes of reduced excretion of potassium include Addison’s disease, renal failure and potassium sparing diuretics.

Renal tubular acidosis (D) occurs when there is a defect in hydrogen ion secretion into the renal tubules. Potassium secretion into the renal tubules therefore increases to balance sodium reabsorption. This results in hypokalaemia with acidosis. Renal tubular acidosis is classified according to the location of the defect: type 1 (distal tubule),type 2 (proximal tubule), type 3 (both distal and proximal tubules). Type 4 results from a defect in the adrenal glands and is included in the classification as it results in a metabolic acidosis and hyperkalaemia.

Spurious sampling (A) of blood results in hyperkalaemia. Excessive vacuuming of blood or using too fine a needle can cause haemolysis, leading to a raised potassium.

Anorexia (B) will result in reduced potassium intake and hence hypoka- laemia. Other causes of reduced potassium intake include dental problems, alcoholism and total parental nutrition deficient in potassium.

Diarrhoea (C) results in hypokalaemia due to increased gastrointestinal losses of potassium. Other causes of increased GI loss of potassium include villous adenoma and VIPoma.

Frusemide (G) intake leads to hypokalaemia secondary to increased renal loss of potassium. This occurs due to increased collecting duct permeability and hence potassium loss.

57
Q

Potassium handling

A Spurious sample
B Anorexia
C Diarrhoea
D Renal tubular acidosis
E Insulin overdose
F Bartter syndrome 
G Frusemide
H Renal failure
I ACE inhibitors

2) A 64-year-old man who is an inpatient on the Care of the Elderly ward is found to have the following blood results:


Na 136 (135–145mmol/L)

K 5.5 (3.5–5.0mmol/L)

Urea 14.4 (3.0–7.0mmol/L) 
Creatinine 165 (60–120mmol/L) 
pH 7.44 (7.35–7.45) 

HCO3 27 (22–28mmol/L)
A

2)H

Renal failure (H) can lead to hyperkalaemia secondary to reduced distal renal delivery of sodium ions. As a consequence, there is reduced exchange of potassium ions via the Na/K ATPase pump in the collecting duct, which thereby leads to accumulation of potassium ions in the blood and hence hyperkalaemia. An increase in aldosterone release will initially cause a compensatory loss of potassium ions; as renal failure progresses, this homeostatic mechanism will become decompensated and hyperkalaemia will result. Renal failure will also be reflected in the deranged urea and creatinine levels due to reduced excretion.

Spurious sampling (A) of blood results in hyperkalaemia. Excessive vacuuming of blood or using too fine a needle can cause haemolysis, leading to a raised potassium.

Anorexia (B) will result in reduced potassium intake and hence hypoka- laemia. Other causes of reduced potassium intake include dental problems, alcoholism and total parental nutrition deficient in potassium.

Diarrhoea (C) results in hypokalaemia due to increased gastrointestinal losses of potassium. Other causes of increased GI loss of potassium include villous adenoma and VIPoma.

Frusemide (G) intake leads to hypokalaemia secondary to increased renal loss of potassium. This occurs due to increased collecting duct permeability and hence potassium loss.

58
Q

Potassium handling

A Spurious sample
B Anorexia
C Diarrhoea
D Renal tubular acidosis
E Insulin overdose
F Bartter syndrome 
G Frusemide
H Renal failure
I ACE inhibitors

3) A 16-day-old baby girl is found to have low blood pressure. Urinary calcium levels are found to be elevated. Blood tests demonstrate the following results:


Na 138 (135–145mmol/L)

K 2.8 (3.5–5.0mmol/L)

Urea 3.4 (3.0–7.0mmol/L) 
Creatinine 62 (60–120mmol/L) 
pH 7.51 (7.35–7.45) 
HCO3 33 (22–28mmol/L)
A

3)F

Bartter syndrome (F) is an autosomal recessive condition due to a defect in the thick ascending limb of the loop of Henle. It is characterized by hypokalaemia, alkalosis and hypotension. The condition may also lead to increased calcium loss via the urine (hypercalcuria) and the kidneys (nephrocalcinosis). Various genetic defects have been discovered; neonatal Bartter syndrome is due to mutations in either the NKCC2 or ROMK genes. In the associated milder Gitelman syndrome, the potassium transporting defect is in the distal convoluted tubule of the kidney.

Spurious sampling (A) of blood results in hyperkalaemia. Excessive vacuuming of blood or using too fine a needle can cause haemolysis, leading to a raised potassium.

Anorexia (B) will result in reduced potassium intake and hence hypoka- laemia. Other causes of reduced potassium intake include dental problems, alcoholism and total parental nutrition deficient in potassium.

Diarrhoea (C) results in hypokalaemia due to increased gastrointestinal losses of potassium. Other causes of increased GI loss of potassium include villous adenoma and VIPoma.

Frusemide (G) intake leads to hypokalaemia secondary to increased renal loss of potassium. This occurs due to increased collecting duct permeability and hence potassium loss.

59
Q

Potassium handling

A Spurious sample
B Anorexia
C Diarrhoea
D Renal tubular acidosis
E Insulin overdose
F Bartter syndrome 
G Frusemide
H Renal failure
I ACE inhibitors

4) A 32-year-old man presents to his GP for a check-up. His serum aldosterone is found to be low. Blood tests reveal the following:

Na 140 (135–145mmol/L)

K 5.6 (3.5–5.0mmol/L)

Urea 5.3 (3.0–7.0mmol/L) 
Creatinine 92 (60–120mmol/L) 
pH 7.38 (7.35–7.45) 
HCO3 24 (22–28mmol/L) 

A

4) I

ACE inhibitors (I) will lead to hyperkalaemia due to reduced potassium excretion. ACE inhibitors antagonize the effect of angiotensin converting enzyme, the enzyme which catalyzes the production of angiotensin II from angiotensin I. A decreased level of angiotensin II reduces the production of aldosterone in the adrenal glands, a key hormone causing the excretion of potassium. Other causes of reduced excretion of potassium include Addison’s disease, renal failure and potassium sparing diuretics.

Spurious sampling (A) of blood results in hyperkalaemia. Excessive vacuuming of blood or using too fine a needle can cause haemolysis, leading to a raised potassium.

Anorexia (B) will result in reduced potassium intake and hence hypoka- laemia. Other causes of reduced potassium intake include dental problems, alcoholism and total parental nutrition deficient in potassium.

Diarrhoea (C) results in hypokalaemia due to increased gastrointestinal losses of potassium. Other causes of increased GI loss of potassium include villous adenoma and VIPoma.

Frusemide (G) intake leads to hypokalaemia secondary to increased renal loss of potassium. This occurs due to increased collecting duct permeability and hence potassium loss.

60
Q

Potassium handling

A Spurious sample
B Anorexia
C Diarrhoea
D Renal tubular acidosis
E Insulin overdose
F Bartter syndrome 
G Frusemide
H Renal failure
I ACE inhibitors

5) A 68-year-old woman on the Care of the Elderly ward is found to have the following blood results:


Na 138 (135–145mmol/L)

K 3.0 (3.5–5.0mmol/L)

Urea 4.2 (3.0–7.0mmol/L) 
Creatinine 74 (60–120mmol/L) 
pH 7.31 (7.35–7.45)

HCO3 28 (22–28mmol/L)
A

5) D

Renal tubular acidosis (D) occurs when there is a defect in hydrogen ion secretion into the renal tubules. Potassium secretion into the renal tubules therefore increases to balance sodium reabsorption. This results in hypokalaemia with acidosis. Renal tubular acidosis is classified according to the location of the defect:

  • -> type 1 (distal tubule),
  • -> type 2 (proximal tubule),
  • -> ype 3 (both distal and proximal tubules).
  • -> Type 4 results from a defect in the adrenal glands and is included in the classification as it results in a metabolic acidosis and hyperkalaemia.

Spurious sampling (A) of blood results in hyperkalaemia. Excessive vacuuming of blood or using too fine a needle can cause haemolysis, leading to a raised potassium.

Anorexia (B) will result in reduced potassium intake and hence hypoka- laemia. Other causes of reduced potassium intake include dental problems, alcoholism and total parental nutrition deficient in potassium.

Diarrhoea (C) results in hypokalaemia due to increased gastrointestinal losses of potassium. Other causes of increased GI loss of potassium include villous adenoma and VIPoma.

Frusemide (G) intake leads to hypokalaemia secondary to increased renal loss of potassium. This occurs due to increased collecting duct permeability and hence potassium loss.

61
Q

Sodium handling

A Ethanol
B SIADH
C Frusemide
D Chronic kidney disease
E Conn’s syndrome
F Diarrhoea
G Congestive cardiac failure 
H Addison’s disease
I Hyperlipidaemia

1) A 50-year-old woman with known diabetes has a routine blood test which demonstrates the following:

Na 130 (135–145mmol/L)
K 4.1 (3.5–5.0mmol/L)
Urea 4.2 (3.0–7.0mmol/L)
Glucose 3.1 (2.2–5.5mmol/L) 
Osmolality 283 (275–295mOsm/kg)
A

1)I 2)D 3)H 4)B 5)G

Pseudo-hyponatraemia can occur in patients with hyperlipidaemia (I) or hyperproteinaemia. In such states, lipids and proteins will occupy a high proportion of the total serum volume. Although the sodium concentration in serum water is in fact normal, a lower sodium concentration will be detected due to dilution by increased lipids and protein molecules. As a consequence, there is an apparent hyponatraemia. A spurious result due to the sample being taken from the drip arm can also cause pseudo-hyponatraemia.

Ethanol (A) may cause hyponatraemia in the context of a raised plasma osmolality (>295mmol/L). Other low molecular weight solutes that can cause hyponatraemia (when osmolality is raised) include mannitol and glucose.

Frusemide (C) and other diuretics cause a hypovolaemic hyponatraemia. As well as a low plasma sodium and osmolality, the urine osmolality will be greater than 20mmol/L, signifying a renal cause of hyponatraemia.

Conn’s syndrome (E), also known as primary aldosteronism, results from an aldosterone-producing adenoma producing excess aldosterone. Biochemical (and concurrent clinical) features include hypernatraemia (HTN) and hypokalaemia (paraesthesia, tetany and weakness).

Diarrhoea (F) leads to a hypovolaemic hyponatraemia (as does vomiting). Plasma sodium and osmolality will be low and urine osmolality will be <20mmol/L indicating an extra-renal cause of hyponatraemia.

62
Q

Sodium handling

A Ethanol
B SIADH
C Frusemide
D Chronic kidney disease
E Conn’s syndrome
F Diarrhoea
G Congestive cardiac failure 
H Addison’s disease
I Hyperlipidaemia

2) A 45-year-old man is seen by his specialist. His last blood and urine tests demonstrated the following:

Na 129 (135–145mmol/L)
K 5.5 (3.5–5.0mmol/L)
Urea 8.2 (3.0–7.0mmol/L)
Glucose 4.2 (2.2–5.5mmol/L) 
Osmolality 265 (275–295mOsm/kg) 
Urine osmolality 26mOsm/kg
A

2) D

A true hyponatraemic state occurs when the osmolality is simultaneously low. Chronic kidney disease (CKD; D) results in urinary protein loss and hence oedema. A reduced circulating volume causes activation of the renin–angiotensin system, thereby raising blood sodium levels. This in turn causes release of antidiuretic hormone (ADH) from the posterior pituitary leading to water retention and hypervolaemic hyponatraemia. Water reabsorption in the renal tubules increases urine osmolality (>20mmol/L indicates a renal cause of hyponatraemia). CKD is also associated with hyperkalaemia and azotaemia.

Ethanol (A) may cause hyponatraemia in the context of a raised plasma osmolality (>295mmol/L). Other low molecular weight solutes that can cause hyponatraemia (when osmolality is raised) include mannitol and glucose.

Frusemide (C) and other diuretics cause a hypovolaemic hyponatraemia. As well as a low plasma sodium and osmolality, the urine osmolality will be greater than 20mmol/L, signifying a renal cause of hyponatraemia.

Conn’s syndrome (E), also known as primary aldosteronism, results from an aldosterone-producing adenoma producing excess aldosterone. Biochemical (and concurrent clinical) features include hypernatraemia (HTN) and hypokalaemia (paraesthesia, tetany and weakness).

Diarrhoea (F) leads to a hypovolaemic hyponatraemia (as does vomiting). Plasma sodium and osmolality will be low and urine osmolality will be <20mmol/L indicating an extra-renal cause of hyponatraemia.

63
Q

Sodium handling

A Ethanol
B SIADH
C Frusemide
D Chronic kidney disease
E Conn’s syndrome
F Diarrhoea
G Congestive cardiac failure 
H Addison’s disease
I Hyperlipidaemia

3) A 30-year-old woman visits her GP due to pigmentation of her palmar creases. Two weeks later the following blood and urine tests are received:

Na 128 (135–145mmol/L)
K 5.9 (3.5–5.0mmol/L)
Urea 5.2 (3.0–7.0mmol/L)
Glucose 1.8 (2.2–5.5mmol/L) 
Osmolality 264 (275–295mOsm/kg)
Urine osmolality 24mOsm/kg
A

3) H

Addison’s disease (H) is also known as primary adrenal insufficiency (reduced aldosterone and cortisol); consequently there is a rise in the production of adrenocorticotropic hormone (ACTH). An impaired synthesis of aldosterone reduces reabsorption of sodium and increases excretion of potassium in the distal convoluted tubule and collecting ducts of the kidney; this leads to a simultaneous hyponatraemia and hyperkalaemia. Reduced cortisol production causes hypoglycaemia due to impaired gluconeogenesis. Clinical features of Addison’s disease include hyperpigmentation, postural hypotension and weight loss.

Ethanol (A) may cause hyponatraemia in the context of a raised plasma osmolality (>295mmol/L). Other low molecular weight solutes that can cause hyponatraemia (when osmolality is raised) include mannitol and glucose.

Frusemide (C) and other diuretics cause a hypovolaemic hyponatraemia. As well as a low plasma sodium and osmolality, the urine osmolality will be greater than 20mmol/L, signifying a renal cause of hyponatraemia.

Conn’s syndrome (E), also known as primary aldosteronism, results from an aldosterone-producing adenoma producing excess aldosterone. Biochemical (and concurrent clinical) features include hypernatraemia (HTN) and hypokalaemia (paraesthesia, tetany and weakness).

Diarrhoea (F) leads to a hypovolaemic hyponatraemia (as does vomiting). Plasma sodium and osmolality will be low and urine osmolality will be <20mmol/L indicating an extra-renal cause of hyponatraemia.

64
Q

Sodium handling

A Ethanol
B SIADH
C Frusemide
D Chronic kidney disease
E Conn’s syndrome
F Diarrhoea
G Congestive cardiac failure 
H Addison’s disease
I Hyperlipidaemia

4) A 30-year old woman is seen by her GP after a 5-day episode of productive cough and lethargy. The GP notes dullness on percussion of the patient’s left lower lung. Blood and urine tests reveal the following:

Na 128 (135–145mmol/L)
K 4.1 (3.5–5.0mmol/L)
Urea 3.5 (3.0–7.0mmol/L)
Glucose 3.2 (2.2–5.5mmol/L) 
Osmolality 265 (275–295mOsm/kg) 
Urine osmolality 285mOsm/kg
A

4)B

SIADH (B) results from the excess release of ADH. In this case the clinical features suggest pneumonia is the cause, but the aetiologies of SIADH are numerous, including malignancy, meningitis and drugs (carbamazepine). Criteria to diagnose SIADH include:
–> Hyponatraemia <135mmol/L
–> Plasma osmolality <270mmol/L
–> Urine osmolality >100mmol/L
–> High urine sodium >20mmol/L
–> Euvolaemia
–> No adrenal, renal or thyroid dysfunction
Characteristically the urine osmolality is inappropriately high; in normal circumstances if the plasma osmolality is low, the urine osmolality will stop rising as reduced ADH secretion prevents water retention. As a rule of thumb in SIADH, urine osmolality is greater than plasma osmolality.

Ethanol (A) may cause hyponatraemia in the context of a raised plasma osmolality (>295mmol/L). Other low molecular weight solutes that can cause hyponatraemia (when osmolality is raised) include mannitol and glucose.

Frusemide (C) and other diuretics cause a hypovolaemic hyponatraemia. As well as a low plasma sodium and osmolality, the urine osmolality will be greater than 20mmol/L, signifying a renal cause of hyponatraemia.

Conn’s syndrome (E), also known as primary aldosteronism, results from an aldosterone-producing adenoma producing excess aldosterone. Biochemical (and concurrent clinical) features include hypernatraemia (HTN) and hypokalaemia (paraesthesia, tetany and weakness).

Diarrhoea (F) leads to a hypovolaemic hyponatraemia (as does vomiting). Plasma sodium and osmolality will be low and urine osmolality will be <20mmol/L indicating an extra-renal cause of hyponatraemia.

65
Q

Sodium handling

A Ethanol
B SIADH
C Frusemide
D Chronic kidney disease
E Conn’s syndrome
F Diarrhoea
G Congestive cardiac failure 
H Addison’s disease
I Hyperlipidaemia

5) A 63-year-old man with chronic obstructive pulmonary disease (COPD) sees his GP due to oedematous ankles. His blood and urine tests show the following:

Na 130 (135–145mmol/L)
K 4.4 (3.5–5.0mmol/L)
Urea 4.2 (3.0–7.0mmol/L)
Glucose 3.1 (2.2–5.5mmol/L) 
Osmolality 268 (275–295mOsm/kg) 
Urine osmolality 16–mmol/LmOsm/kg
A

5) G

Congestive cardiac failure (G) may present with shortness of breath, pitting peripheral oedema and/or raised jugular venous pulse (JVP). In this scenario, shortness of breath may be masked by the patient’s COPD. The clinical picture together with the blood result demonstrating a low sodium and low osmolality suggest a hypervolaemic hyponatraemia. This scenario can be differentiated from hypervolaemia as a result of CKD (D) by the urine osmolality, which is less than 20mmol/L in this instance, thereby suggesting a non-renal cause for the hyponatraemia.

Ethanol (A) may cause hyponatraemia in the context of a raised plasma osmolality (>295mmol/L). Other low molecular weight solutes that can cause hyponatraemia (when osmolality is raised) include mannitol and glucose.

Frusemide (C) and other diuretics cause a hypovolaemic hyponatraemia. As well as a low plasma sodium and osmolality, the urine osmolality will be greater than 20mmol/L, signifying a renal cause of hyponatraemia.

Conn’s syndrome (E), also known as primary aldosteronism, results from an aldosterone-producing adenoma producing excess aldosterone. Biochemical (and concurrent clinical) features include hypernatraemia (HTN) and hypokalaemia (paraesthesia, tetany and weakness).

Diarrhoea (F) leads to a hypovolaemic hyponatraemia (as does vomiting). Plasma sodium and osmolality will be low and urine osmolality will be <20mmol/L indicating an extra-renal cause of hyponatraemia.

66
Q

Therapeutic drug monitoring

A Procainamide
B Lithium
C Methotrexate
D Theophylline
E Gentamicin
F Carbamazepine 
G Cyclosporine
H Phenytoin
I Digoxin

1) A 35-year-old man presents to accident and emergency with feelings of lightheadedness and slurred speech. His wife mentions that the patient has been walking around ‘like a drunk’. The man’s blood pressure is found to be low.

A

1)H 2)B 3)E 4)I 5)D

Phenytoin (H) is a commonly used anti-epileptic agent. Serum levels of phenytoin must be monitored due to its narrow therapeutic range (10–20μg/mL). Phenytoin also exhibits saturation kinetics; a small rise in dose may lead to saturation of metabolism by CYP enzymes in the liver, hence producing a large increase in drug concentration in the blood as well as associated toxic effects. Phenytoin toxicity can lead to hypotension, heart block, ventricular arrhythmias and ataxia.

Lithium (B) is a therapeutic agent used in the treatment of bipolar disorder. Drug monitoring is essential (12 hours post dose) due to its low therapeutic index as well as the potential life-threatening effects of toxicity. Lithium is excreted via the kidneys and therefore serum drug levels may increase (with potential toxicity) in states of low glomerular filtration rate, sodium depletion and diuretic use. Features of lithium toxicity include diarrhoea, vomiting, dysarthria and coarse tremor. Severe toxicity may cause convulsions, renal failure and possibly death.

Gentamicin (E) is an aminoglycoside antibiotic, particularly useful against Gram-negative bacteria. It exhibits a low therapeutic index. Factors that may potentiate toxicity include dosage, kidney function (gentamicin is excreted through the kidneys) and other medications such as vancomycin. Gentamicin is an ototoxic and nephrotoxic agent and hence toxicity can lead to deafness and renal failure. Toxic effects on the ear are not limited to hearing, as the vestibular system is also affected, which may cause problems with balance and vision.

Digoxin (I) is an anti-arrhythmic agent used in the treatment of atrial fibrillation and atrial flutter. Symptoms of under-treatment and toxicity are similar. Toxicity commonly arises due to the narrow therapeutic index of the agent. Non-specific symptoms of toxicity include tiredness, blurred vision, nausea, abdominal pain & confusion. ECG changes may include a prolonged PR interval and bradycardia. As digoxin is excreted via the kidneys, renal failure may cause accumulation of digoxin.

Theophylline (D) is a drug used in the treatment of asthma and COPD. A low therapeutic index and wide variation in metabolism between patients lead to requirement for drug monitoring. Toxicity may manifest in a number of ways including nausea, diarrhoea, tachycardia, arrhythmias & headaches. Severe toxicity may lead to seizures. The toxic effects of theophylline are potentiated by erythromycin and ciprofloxacin. Without monitoring, many patients would be undertreated.

Procainamide (A) is an anti-arrhythmic agent. Toxicity may lead to rash, fever and agranulocytosis. Drug induced lupus erythematosus may result from toxic levels.

Methotrexate (C) is an anti-folate drug used in the treatment of cancers and autoimmune conditions. Toxicity may lead to ulcerative stomatitis, leukocytopenia and rarely pulmonary fibrosis.

Carbamazepine (F) is an anti-convulsant medication. Toxic levels may commonly result in headaches, ataxia and abdominal pain. Toxicity may also cause SIADH and, rarely, aplastic anaemia.

Cyclosporine (G) is an immunosuppressant. Toxicity is associated with acute renal failure. Calcium channel antagonists and certain antibiotics such as erythromycin predispose to nephrotoxicity, whereas anti-convulsants such as phenytoin reduce blood levels of the drug.

67
Q

Therapeutic drug monitoring

A Procainamide
B Lithium
C Methotrexate
D Theophylline
E Gentamicin
F Carbamazepine 
G Cyclosporine
H Phenytoin
I Digoxin

2) A 45-year-old woman is told she may be demonstrating signs of toxicity, 12 hours after being given an initial dose of medication. She has a coarse tremor and complains of feeling nauseous.

A

1)H 2)B 3)E 4)I 5)D

Phenytoin (H) is a commonly used anti-epileptic agent. Serum levels of phenytoin must be monitored due to its narrow therapeutic range (10–20μg/mL). Phenytoin also exhibits saturation kinetics; a small rise in dose may lead to saturation of metabolism by CYP enzymes in the liver, hence producing a large increase in drug concentration in the blood as well as associated toxic effects. Phenytoin toxicity can lead to hypotension, heart block, ventricular arrhythmias and ataxia.

Lithium (B) is a therapeutic agent used in the treatment of bipolar disorder. Drug monitoring is essential (12 hours post dose) due to its low therapeutic index as well as the potential life-threatening effects of toxicity. Lithium is excreted via the kidneys and therefore serum drug levels may increase (with potential toxicity) in states of low glomerular filtration rate, sodium depletion and diuretic use. Features of lithium toxicity include diarrhoea, vomiting, dysarthria and coarse tremor. Severe toxicity may cause convulsions, renal failure and possibly death.

Gentamicin (E) is an aminoglycoside antibiotic, particularly useful against Gram-negative bacteria. It exhibits a low therapeutic index. Factors that may potentiate toxicity include dosage, kidney function (gentamicin is excreted through the kidneys) and other medications such as vancomycin. Gentamicin is an ototoxic and nephrotoxic agent and hence toxicity can lead to deafness and renal failure. Toxic effects on the ear are not limited to hearing, as the vestibular system is also affected, which may cause problems with balance and vision.

Digoxin (I) is an anti-arrhythmic agent used in the treatment of atrial fibrillation and atrial flutter. Symptoms of under-treatment and toxicity are similar. Toxicity commonly arises due to the narrow therapeutic index of the agent. Non-specific symptoms of toxicity include tiredness, blurred vision, nausea, abdominal pain & confusion. ECG changes may include a prolonged PR interval and bradycardia. As digoxin is excreted via the kidneys, renal failure may cause accumulation of digoxin.

Theophylline (D) is a drug used in the treatment of asthma and COPD. A low therapeutic index and wide variation in metabolism between patients lead to requirement for drug monitoring. Toxicity may manifest in a number of ways including nausea, diarrhoea, tachycardia, arrhythmias & headaches. Severe toxicity may lead to seizures. The toxic effects of theophylline are potentiated by erythromycin and ciprofloxacin. Without monitoring, many patients would be undertreated.

Procainamide (A) is an anti-arrhythmic agent. Toxicity may lead to rash, fever and agranulocytosis. Drug induced lupus erythematosus may result from toxic levels.

Methotrexate (C) is an anti-folate drug used in the treatment of cancers and autoimmune conditions. Toxicity may lead to ulcerative stomatitis, leukocytopenia and rarely pulmonary fibrosis.

Carbamazepine (F) is an anti-convulsant medication. Toxic levels may commonly result in headaches, ataxia and abdominal pain. Toxicity may also cause SIADH and, rarely, aplastic anaemia.

Cyclosporine (G) is an immunosuppressant. Toxicity is associated with acute renal failure. Calcium channel antagonists and certain antibiotics such as erythromycin predispose to nephrotoxicity, whereas anti-convulsants such as phenytoin reduce blood levels of the drug.

68
Q

Therapeutic drug monitoring

A Procainamide
B Lithium
C Methotrexate
D Theophylline
E Gentamicin
F Carbamazepine 
G Cyclosporine
H Phenytoin
I Digoxin

3) A 65-year-old man being treated as an inpatient develops sudden onset ‘ringing in his ears’ as well as difficulty hearing.

A

3)E

Gentamicin (E) is an aminoglycoside antibiotic, particularly useful against Gram-negative bacteria. It exhibits a low therapeutic index. Factors that may potentiate toxicity include dosage, kidney function (gentamicin is excreted through the kidneys) and other medications such as vancomycin. Gentamicin is an ototoxic and nephrotoxic agent and hence toxicity can lead to deafness and renal failure. Toxic effects on the ear are not limited to hearing, as the vestibular system is also affected, which may cause problems with balance and vision.

Procainamide (A) is an anti-arrhythmic agent. Toxicity may lead to rash, fever and agranulocytosis. Drug induced lupus erythematosus may result from toxic levels.

Methotrexate (C) is an anti-folate drug used in the treatment of cancers and autoimmune conditions. Toxicity may lead to ulcerative stomatitis, leukocytopenia and rarely pulmonary fibrosis.

Carbamazepine (F) is an anti-convulsant medication. Toxic levels may commonly result in headaches, ataxia and abdominal pain. Toxicity may also cause SIADH and, rarely, aplastic anaemia.

Cyclosporine (G) is an immunosuppressant. Toxicity is associated with acute renal failure. Calcium channel antagonists and certain antibiotics such as erythromycin predispose to nephrotoxicity, whereas anti-convulsants such as phenytoin reduce blood levels of the drug.

69
Q

Therapeutic drug monitoring

A Procainamide
B Lithium
C Methotrexate
D Theophylline
E Gentamicin
F Carbamazepine 
G Cyclosporine
H Phenytoin
I Digoxin

4) A 45-year-old woman is seen by her GP for a routine medications review. The patient complains of recent onset abdominal pain and tiredness. ECG reveals prolonged PR interval.

A

4) I

Digoxin (I) is an anti-arrhythmic agent used in the treatment of atrial fibrillation and atrial flutter. Symptoms of under-treatment and toxicity are similar. Toxicity commonly arises due to the narrow therapeutic index of the agent. Non-specific symptoms of toxicity include tiredness, blurred vision, nausea, abdominal pain & confusion. ECG changes may include a prolonged PR interval and bradycardia. As digoxin is excreted via the kidneys, renal failure may cause accumulation of digoxin.

Procainamide (A) is an anti-arrhythmic agent. Toxicity may lead to rash, fever and agranulocytosis. Drug induced lupus erythematosus may result from toxic levels.

Methotrexate (C) is an anti-folate drug used in the treatment of cancers and autoimmune conditions. Toxicity may lead to ulcerative stomatitis, leukocytopenia and rarely pulmonary fibrosis.

Carbamazepine (F) is an anti-convulsant medication. Toxic levels may commonly result in headaches, ataxia and abdominal pain. Toxicity may also cause SIADH and, rarely, aplastic anaemia.

Cyclosporine (G) is an immunosuppressant. Toxicity is associated with acute renal failure. Calcium channel antagonists and certain antibiotics such as erythromycin predispose to nephrotoxicity, whereas anti-convulsants such as phenytoin reduce blood levels of the drug.

70
Q

Therapeutic drug monitoring

A Procainamide
B Lithium
C Methotrexate
D Theophylline
E Gentamicin
F Carbamazepine 
G Cyclosporine
H Phenytoin
I Digoxin

5) A 45-year-old man presents to his GP for a routine medications review. The patient complains of recent diarrhoea and headaches. The GP notes the patient was treated with erythromycin for a community acquired pneumonia 1 week previous to the consultation.

A

1)H 2)B 3)E 4)I 5)D

Theophylline (D) is a drug used in the treatment of asthma and COPD. A low therapeutic index and wide variation in metabolism between patients lead to requirement for drug monitoring. Toxicity may manifest in a number of ways including nausea, diarrhoea, tachycardia, arrhythmias & headaches. Severe toxicity may lead to seizures. The toxic effects of theophylline are potentiated by erythromycin and ciprofloxacin. Without monitoring, many patients would be undertreated.

Procainamide (A) is an anti-arrhythmic agent. Toxicity may lead to rash, fever and agranulocytosis. Drug induced lupus erythematosus may result from toxic levels.

Methotrexate (C) is an anti-folate drug used in the treatment of cancers and autoimmune conditions. Toxicity may lead to ulcerative stomatitis, leukocytopenia and rarely pulmonary fibrosis.

Carbamazepine (F) is an anti-convulsant medication. Toxic levels may commonly result in headaches, ataxia and abdominal pain. Toxicity may also cause SIADH and, rarely, aplastic anaemia.

Cyclosporine (G) is an immunosuppressant. Toxicity is associated with acute renal failure. Calcium channel antagonists and certain antibiotics such as erythromycin predispose to nephrotoxicity, whereas anti-convulsants such as phenytoin reduce blood levels of the drug.

71
Q

Vitamin deficiencies

A Vitamin A
B Vitamin B1
C Vitamin B2
D Vitamin B6
E Vitamin B12
F Vitamin C 
G Vitamin D 
H Vitamin E 
I Vitamin K

1) A 40-year-old patient with a history of Graves’ disease presents with bilateral weakness of her legs. On examination she is Babinski sign positive and blood tests reveal a megaloblastic anaemia.

A

1)E 2)F 3)H 4)D 5)B

Vitamin B12 (cobalamin; E) deficiency may result from pathologies affecting the stomach or ileum, as well as pernicious anaemia. In perni- cious anaemia, autoantibodies exist against intrinsic factor. Pernicious anaemia is also commonly associated with other autoimmune conditions, such as Graves’ disease. Anaemia is a common manifestation of vitamin B12 deficiency, with raised mean cell volume and hypersegmented neutrophils evident. Subacute combined degeneration of the cord can also result, causing ataxia and progressive weakness in limbs and trunk; Babinski sign may be positive.

Vitamin A (A) deficiency primarily impairs the production of rods and hence causes night blindness; ocular epithelial changes also cause conjunctival Bitot’s spots. Deficiency may cause predisposition to measles and diarrhoeal illnesses.

Vitamin B2 (riboflavin; C) deficiency leads to mucosal damage and hence presents with angular stomatitis, glossitis and/or corneal ulceration.

Vitamin D (G) deficiency results from reduced dietary intake as well as inadequate sunlight exposure. Deficiency leads to bone pathology, including rickets in children and osteomalacia in adults.

Vitamin K (I) deficiency may result from reduced intestinal uptake or dietary deficiency. Presenting features may include ecchymosis, petechiae, haematomas and slow healing at wound sites.

72
Q

Vitamin deficiencies

A Vitamin A
B Vitamin B1
C Vitamin B2
D Vitamin B6
E Vitamin B12
F Vitamin C 
G Vitamin D 
H Vitamin E 
I Vitamin K

2) A 26-year-old man presents to his GP with a 5-month history of bleeding gums. Petechiae are also observed on the patient’s feet. The man admits he has had to visit his dentist recently due to poor dentition.

A

1)E 2)F 3)H 4)D 5)B

Vitamin C (F) is a water soluble vitamin, essential for the hydroxylation of collagen. When deficiency of vitamin C is present, collagen is unable to form a helical structure and hence cannot produce cross-links. As a consequence, damaged vessels and wounds are slow to heal. Vitamin C deficiency results in scurvy, which describes both bleeding (gums, skin and joints) and bone weakness (microfractures and brittle bones) tendencies. Gum disease is also a characteristic feature.

Vitamin A (A) deficiency primarily impairs the production of rods and hence causes night blindness; ocular epithelial changes also cause conjunctival Bitot’s spots. Deficiency may cause predisposition to measles and diarrhoeal illnesses.

Vitamin B2 (riboflavin; C) deficiency leads to mucosal damage and hence presents with angular stomatitis, glossitis and/or corneal ulceration.

Vitamin D (G) deficiency results from reduced dietary intake as well as inadequate sunlight exposure. Deficiency leads to bone pathology, including rickets in children and osteomalacia in adults.

Vitamin K (I) deficiency may result from reduced intestinal uptake or dietary deficiency. Presenting features may include ecchymosis, petechiae, haematomas and slow healing at wound sites.

73
Q

Vitamin deficiencies

A Vitamin A
B Vitamin B1
C Vitamin B2
D Vitamin B6
E Vitamin B12
F Vitamin C 
G Vitamin D 
H Vitamin E 
I Vitamin K

3) A 5-year-old girl who is a known cystic fibrosis sufferer is noted by her mother to have developed poor coordination of her hands and on examination her reflexes are absent. Blood tests also reveal anaemia.

A

3)H

Vitamin E (tocopherol; H) is an important anti-oxidant which acts to scavenge free radicals in the blood stream. Deficiency leads to haemolytic anaemia as red blood cells encounter oxidative damage and are consequently broken down in the spleen. Spino-cerebellar neuropathy is also a manifestation, which is characterized by ataxia and areflexia. Vitamin E deficiency has also been suggested to increase the risk of ischaemic heart disease in later life, as low-density lipoproteins become oxidized perpetuating the atherosclerotic process.

Vitamin A (A) deficiency primarily impairs the production of rods and hence causes night blindness; ocular epithelial changes also cause conjunctival Bitot’s spots. Deficiency may cause predisposition to measles and diarrhoeal illnesses.

Vitamin B2 (riboflavin; C) deficiency leads to mucosal damage and hence presents with angular stomatitis, glossitis and/or corneal ulceration.

Vitamin D (G) deficiency results from reduced dietary intake as well as inadequate sunlight exposure. Deficiency leads to bone pathology, including rickets in children and osteomalacia in adults.

Vitamin K (I) deficiency may result from reduced intestinal uptake or dietary deficiency. Presenting features may include ecchymosis, petechiae, haematomas and slow healing at wound sites.

74
Q

Vitamin deficiencies

A Vitamin A
B Vitamin B1
C Vitamin B2
D Vitamin B6
E Vitamin B12
F Vitamin C 
G Vitamin D 
H Vitamin E 
I Vitamin K

4) A 35-year-old man who is being treated for tuberculosis develops a rash on his trunk. Blood tests also reveal anaemia.

A

4)D

Vitamin B6 (pyridoxine; D) is an essential co-factor in a number of metabolic pathways including the synthesis of amino acids and neurotransmitters. Common causes of deficiency include reduced dietary intake and isoniazid use for the treatment of tuberculosis. Vitamin B6 deficiency causes blood and skin abnormalities. Haematologically, vita- min B6 deficiency causes sideroblastic anaemia; dermatologically seborrhoeic dermatitis can occur. Diagnosis is made by determining erythrocyte levels of aspartate aminotransferase.

Vitamin A (A) deficiency primarily impairs the production of rods and hence causes night blindness; ocular epithelial changes also cause conjunctival Bitot’s spots. Deficiency may cause predisposition to measles and diarrhoeal illnesses.

Vitamin B2 (riboflavin; C) deficiency leads to mucosal damage and hence presents with angular stomatitis, glossitis and/or corneal ulceration.

Vitamin D (G) deficiency results from reduced dietary intake as well as inadequate sunlight exposure. Deficiency leads to bone pathology, including rickets in children and osteomalacia in adults.

Vitamin K (I) deficiency may result from reduced intestinal uptake or dietary deficiency. Presenting features may include ecchymosis, petechiae, haematomas and slow healing at wound sites.

75
Q

Vitamin deficiencies

A Vitamin A
B Vitamin B1
C Vitamin B2
D Vitamin B6
E Vitamin B12
F Vitamin C 
G Vitamin D 
H Vitamin E 
I Vitamin K

5) A 40-year-old known alcoholic develops confusion and an unsteady gait. On examination bilateral lateral rectus palsy is noted.

A

5) B

Vitamin B1 (thiamine; B) deficiency most commonly occurs in cases of alcoholism. The acute presentation of vitamin B1 deficiency is Wernicke’s encephalopathy, characterized by the triad of confusion, ophthalmoplegia and ataxia. Chronic alcoholism can lead to Korsakoff’s syndrome (amnesia and confabulation) and peripheral neuropathy. Beriberi can also occur, classified into wet and dry beriberi. Wet beri- beri presents in a similar manner to heart failure, with cardiomegaly, oedema and dyspnoea. Dry beriberi involves an ascending impairment of nervous function involving both sensory (paraesthesia) and motor (foot drop, wrist drop and paralysis) components.

Vitamin A (A) deficiency primarily impairs the production of rods and hence causes night blindness; ocular epithelial changes also cause conjunctival Bitot’s spots. Deficiency may cause predisposition to measles and diarrhoeal illnesses.

Vitamin B2 (riboflavin; C) deficiency leads to mucosal damage and hence presents with angular stomatitis, glossitis and/or corneal ulceration.

Vitamin D (G) deficiency results from reduced dietary intake as well as inadequate sunlight exposure. Deficiency leads to bone pathology, including rickets in children and osteomalacia in adults.

Vitamin K (I) deficiency may result from reduced intestinal uptake or dietary deficiency. Presenting features may include ecchymosis, petechiae, haematomas and slow healing at wound sites.